You are on page 1of 157

Theme: Head and neck lumps

A. Branchial cyst
B. Cystic hygroma
C. Carotid body tumour
D. Lymphadenopathy
E. Adenolymphoma of the parotid
F. Pleomorphic adenoma of the parotid
G. Submandibular tumour
H. Thyroglossal cyst
I. Thoracic outlet syndrome
J. Submandibular gland calculus

Please select the most likely lesion to account for the clinical scenario given. Each option may be used
once, more than once or not at all.

1. A 60 year old Tibetan immigrant is referred to the surgical clinic with a painless neck swelling. On
examination it is located on the left side immediately anterior to the sternocleidomastoid muscle.
There are no other abnormalities to find on examination.

Carotid body tumour

Carotid body tumours typically present as painless masses. They may compress the vagus or
hypoglossal nerves with symptoms attributable to these structures. Over 90% occur spontaneously
and are more common in people living at high altitude. In familial cases up to 30% may be
bilateral. Treatment is with excision.

2. A 40 year old women presents as an emergency with a painful mass underneath her right mandible.
The mass has appeared over the previous week with the pain worsening as the lump has increased
in size. On examination there is a 4cm mass underneath her mandible, there is no associated
lymphadenopathy.

Submandibular gland calculus

The sub mandibular gland is the most common site for salivary calculi. Patients will usually
complain of pain, which is worse on eating. When the lesion is located distally the duct may be laid
open and the stone excised. Otherwise the gland will require removal.

3. A 73 year old male smoker is referred to the clinic by his GP. On examination he has a 3cm soft
mass immediately anterior to his ear. It has been present for the past five years and is otherwise
associated with no symptoms.

You answered Pleomorphic adenoma of the parotid

The correct answer is Adenolymphoma of the parotid

Warthins tumours (a.k.a. adenolymphoma) are commoner in older men (especially smokers). They
are the second commonest benign tumour of the parotid gland, they may be bilateral. They are soft
and slow growing and relatively easy to excise. Pleomorphic adenomas typically present in females
aged between 40 - 60 years.

Neck lumps

The table below gives characteristic exam question features for conditions causing neck lumps:

Reactive By far the most common cause of neck swellings. There may be a history of
lymphadenopathy local infection or a generalised viral illness
Lymphoma Rubbery, painless lymphadenopathy
The phenomenon of pain whilst drinking alcohol is very uncommon
There may be associated night sweats and splenomegaly
Thyroid swelling May be hypo-, eu- or hyperthyroid symptomatically
Moves upwards on swallowing
Thyroglossal cyst More common in patients < 20 years old
Usually midline, between the isthmus of the thyroid and the hyoid bone
Moves upwards with protrusion of the tongue
May be painful if infected
Pharyngeal pouch More common in older men
Represents a posteromedial herniation between thyropharyngeus and
cricopharyngeus muscles
Usually not seen, but if large then a midline lump in the neck that gurgles on
palpation
Typical symptoms are dysphagia, regurgitation, aspiration and chronic cough
Cystic hygroma A congenital lymphatic lesion (lymphangioma) typically found in the neck,
classically on the left side
Most are evident at birth, around 90% present before 2 years of age
Branchial cyst An oval, mobile cystic mass that develops between the sternocleidomastoid
muscle and the pharynx
Develop due to failure of obliteration of the second branchial cleft in
embryonic development
Usually present in early adulthood
Cervical rib More common in adult females
Around 10% develop thoracic outlet syndrome
Carotid aneurysm Pulsatile lateral neck mass which doesn't move on swallowing

A 12 year old child is admitted with a 12 hour history of colicky right upper quadrant pain. On
examination the child is afebrile and is jaundiced. The abdomen is soft and non tender at the time of
examination. What is the most likely cause?

A. Infectious hepatitis

B. Acute cholecystitis

C. Cholangitis

D. Hereditary spherocytosis

E. Gilberts syndrome

Theme from September 2012 Exam


The child is most likely to have hereditary spherocytosis. In these individuals there may be disease flares
precipitated by acute illness. They form small pigment stones. These may cause biliary colic and some
may require cholecystectomy.

Hereditary Spherocytosis

Most common disorder of the red cell membrane, it has an incidence of 1 in 5000. The abnormally
shaped erythrocytes are prone to splenic sequestration and destruction. This can result in
hyperbilirubinaemia, jaundice and splenomegaly. In older patients an intercurrent illness may increase
the rate of red cell destruction resulting in more acute symptoms.
Severe cases may benefit from splenectomy.

A 40 year old man presents with obstructive jaundice. Twenty years previously he underwent a right
hemicolectomy for a mucinous right sided colonic carcinoma. He was subsequently diagnosed as having
Lynch syndrome. What is the most likely cause of his jaundice?

A. Hepatocellular carcinoma

B. Liver metastasis from colonic cancer

C. Pancreatic carcinoma

D. Duodenal carcinoma
E. Gastric carcinoma

Theme from April 2013


Lynch syndrome usually results in colonic cancer which is right sided and mucinous. The next most
common site to be affected is the uterus. The stomach is at particular risk and this risk is up to 10 times
greater in HNPCC (Lynch) patients than the general population. Duodenal adenomas (and rarely
carcinoma) are usually seen in association with FAP. Whilst pancreatic carcinoma is associated with
HNPCC it is far less likely to occur than gastric cancer.

Genetics of colorectal cancer

The lifetime risk of colorectal cancer in the UK population is 5%. Up to 5% of newly diagnosed bowel
cancers will be in those individuals who have a high genetically acquired risk of bowel cancer. Cancers
arising in the low-moderate genetic risk group comprise approximately 30% of newly diagnosed bowel
cancer.

Genetics of inherited colorectal cancer syndromes


Syndrome Features Genes implicated
FAP More than 100 adenomatous polyps affecting the colon and APC (over 90%)
rectum. Duodenal and fundic glandular polyps
Gardner syndrome As FAP but with desmoid tumours and mandibular osteomas APC
Turcots syndrome Polyposis and colonic tumours and CNS tumours APC +MLH1 and
PMS2
HNPCC Colorectal cancer without extensive polyposis. Endometrial MSH2, MLH1,
cancer, renal and CNS PMS2 and GTBP
Peutz-Jeghers Hamartomatous polyps in GI tract and increased risk of GI LKB1 andSTK11 (in
syndrome malignancy up to 70%)
Cowden disease Multiple hamartomas (see below) PTEN (85%)
MYH associated Autosomal recessive, multiple adenomatous polyps in GI MYH
polyposis tract, those in colon having somatic KRAS mutations

FAP
Autosomal dominant condition, affects 1 in 12,000. Accounts for 0.5% of all CRCs. Lifetime incidence
of colorectal cancer in untreated FAP =100%. Up to 25% cases are caused by de-novo germ line
mutations and show no prior family history. The APC tumour suppressor gene is affected in most cases.

APC in non inherited colorectal cancer


Up to 80% of sporadic colorectal cancers will have somatic mutations that inactivate APC[1]. Both
alleles are usually affected. Although the APC protein more than likely has multiple critical cellular
functions, the best-established role for APC in the cancer process is as a major binding partner and
regulator of the β- catenin protein in the so-called canonical or β- catenin dependent Wnt signaling
pathway.

HNPCC
HNPCC cancers differ from conventional tumours in a number of respects. In the colon the tumours are
more likely to be right sided, histologically they are more likely to be mucinous and have dense
lymphocytic infiltrates. To be diagnosed as having HNPCC individuals must show typically HNPCC
tumours in at least three individuals, (one of whom must be a first degree relative to the other two). In at
least two successive generations. At least one cancer must be diagnosed under the age of 50. FAP must
be excluded and tumours should be verified by pathological identification (Amsterdam criteria). The
genetic changes in HNPCC stem primarily from microsatellite instability affecting DNA mismatch
repair genes. In HNPCC the mismatch repair genes most commonly implicated include; MSH2 and
MLH1 and these occur in up to 70% of people with HNPCC. The finding of microsatellite instability is
unusual in sporadic colorectal cancers. Approximately 60% of individuals who fulfill the Amsterdam
criteria will not be found to have evidence of mismatch repair gene defects on genetic testing. The risk
of developing colorectal cancer in those who do not have demonstrated mutation of the mis match repair
genes is increased if they fulfill the Amsterdam criteria, but not the the extent that it is increased in those
who fulfill the criteria AND have evidence of mis match repair gene defects.

KRAS Mutations
The RAS family of small G proteins act as molecular switches downstream of growth factor receptors.
KRAS and the other two members of the family; HRAS and NRAS, are the site of mutation in
approximately 40% of colorectal cancers. When adenomas are examined the proportion of adenomas
less than 1cm showing KRAS mutations was only 10% which contrasts with 50% in those lesions
greater than 1cm.

p53 mutations
The p53 protein functions as a key transcriptional regulator of genes that encode proteins with functions
in cell-cycle checkpoints at the G1/S and G2/M boundaries, in promoting apoptosis, and in restricting
angiogenesis . As such, selection for p53 defects at the adenoma-carcinoma transition may reflect the
fact that stresses on tumor cells activate cell-cycle arrest, apoptotic, and antiangiogenic pathways in cells
with wild-type p53 function. Many colonic tumours will demonstrate changes in the p53 gene that may
facilitate tumour progression through from adenoma to carcinoma.

Cowden syndrome
Also known as multiple hamartoma syndrome. Rare autosomal dominant condition with incidence of 1
in 200,000.. It is characterised by multiple mucocutaneous lesions, trichilemmomas, oral papillomas and
acral keratosis. Most often diagnosed in third decade of life. Breast carcinoma may occur in up to 50%
of patients and conditions such as fibrocystic disease of the breast may occur in 75% of women. Thyroid
disease occurs in 75% and may include malignancy. Endoscopic screening will identify disease in up to
85% although the small bowel is rarely involved. There is a 15-20% risk of developing colorectal cancer
and regular colonoscopic screening from age 45 is recommended.

Terminology
Oncogene Oncogenes are genes which have the potential to induce cellular proliferation and avoid
apoptosis. Oncogene mutations are general gain of function and are therefore dominant.
Increased expression of oncogenes are found in most tumours
Tumour These genes generally inhibit cellular proliferation or induce apoptosis. Mutations in
suppressor tumour suppressor genes are generally loss of function mutations, and are therefore
gene recessive. Mutations in both tumour suppressor gene alleles allow cells to proliferate
without restraint

Theme: Breast disease

A. Tuberculosis
B. Actinomycosis
C. Duct ectasia
D. Fibroadenoma
E. Fat necrosis
F. Intraductal papilloma
G. Breast abscess
H. Breast cancer

What is the most likely diagnosis for each scenario given? Each diagnosis may be used once, more than
once or not at all.

6. A 32 year old woman presents with a tender breast lump. She has a 2 month old child. Clinically
there is a tender, fluctuant mass of the breast.

Breast abscess

Theme from January 2013 Exam


Theme from September 2011 Exam
This lady is likely to be breast feeding and is at risk of mastitis. This may lead to an abscess if not
treated. Staphylococcus aureus is usually the causative organism.

7. A 53 year old lady presents with a creamy nipple discharge. On examination she has discharge
originating from multiple ducts and associated nipple inversion.

Duct ectasia

Duct ectasia is common during the period of breast involution that occurs during the menopausal
period. As the ducts shorten they may contain insipiated material. The discharge will often
discharge from several ducts.

8. A 52 year old lady presents with an episode of nipple discharge. It is usually clear in nature. On
examination the discharge is seen to originate from a single duct and although it appears clear,
when the discharge is tested with a labstix it is shown to contain blood. Imaging and examination
shows no obvious mass lesion.
Intraductal papilloma

Intraductal papilloma usually cause single duct discharge. The fluid is often clear, although it may
be blood stained. If the fluid is tested with a labstix (little point in routine practice) then it will
usually contain small amounts of blood. A microdocechtomy may be performed.

Non malignant breast disease

Duct ectasia

• Mammary duct ectasia may be seen in up to 25% of normal female breasts


• Patients usually present with nipple discharge, which may be from single or multiple ducts
(usually present age >50 years)
• The discharge is often thick and green
• Duct ectasia is a normal varient of breast involution and is not the same condition as periductal
mastitis

Periductal mastitis

• Present at younger age than duct ectasia


• May present with features of inflammation, abscess or mammary duct fistula
• Stongly associated with smoking
• Usually treated with antibiotics, abscess will require drainage

Intraductal papilloma

• Growth of papilloma in a single duct


• Usually presents with clear or blood stained discharge originating from a single duct
• No increase in risk of malignancy

Breast abscess

• Lactational mastitis is common


• Infection is usually with Staphylococcus aureus
• On examination there is usually a tender fluctuant mass
• Treatment is with antibiotics and ultrasound guided aspiration
• Overlying skin necrosis is an indication for surgical debridement, which may be complicated by
the development of a subsequent mammary duct fistula.
Tuberculosis

• Rare in western countries, usually secondary TB


• Affects women later in child bearing period
• Chronic breast or axillary sinus is present in up to 50% cases
• Diagnosis is by biopsy culture and histology

A 2 day old baby is noted to have voiding difficulties and on closer inspection is noted to have
hypospadias. Which of the following abnormalities is most commonly associated with the condition?

A. Cryptorchidism

B. Diaphragmatic hernia

C. Ventricular - septal defect

D. Bronchogenic cyst

E. Atrial septal defect

Theme from January 2012 Exam

Hypospadias most commonly occurs as an isolated disorder. Associated urological abnormalities may be
seen in up to 40% of infants, of these cryptorchidism is the most frequent (10%).

Hypospadias

The urethral meatus opens on the ventral surface of the penis. There is also a ventral deficiency of the
foreskin. The urethral meatus may open more proximally in the more severe variants. However, 75% of
the openings are distally located. The incidence is 1 in 300 male births.

Features include:

• Absent frenular artery


• Ventrally opened glans
• Skin tethering to hypoplastic urethra
• Splayed columns of spongiosum tissue distal to the meatus
• Deficiency of the foreskin ventrally

Management:

• No routine cultural circumcisions


• Urethroplasty
• Penile reconstruction

The foreskin is often utilised in the reconstructive process. In boys with very distal disease no treatment
may be needed.

Theme: Liver lesions

A. Cystadenoma
B. Hydatid cyst
C. Amoebic abscess
D. Mesenchymal hamartoma
E. Liver cell adenoma
F. Cavernous haemangioma

Please select the most likely lesion for the scenario given. Each option may be used once, more than
once or not at all.

10. A 38 year old lady presents with right upper quadrant pain and nausea. She is otherwise well and
her only medical therapy is the oral contraceptive pill which she has taken for many years with no
ill effects. Her liver function tests are normal. An ultrasound examination demonstrates a
hyperechoic well defined lesion in the left lobe of the liver which measures 14 cm in diameter.

You answered Liver cell adenoma

The correct answer is Cavernous haemangioma

Cavernous haemangioma often presents with vague symptoms and signs. They may grow to
considerable size. Liver function tests are usually normal. The lesions are typically well defined
and hyperechoic on ultrasound. A causative link between OCP use and haemangiomata has yet to
be established, but is possible.

11. A 37 year old lady presents with right upper quadrant pain and nausea. She is otherwise well and
her only medical therapy is the oral contraceptive pill which she has taken for many years with no
ill effects. Her liver function tests and serum alpha feto protein are normal. An ultrasound
examination demonstrates a 4cm non encapsulated lesion in the right lobe of the liver which has a
mixed echoity and heterogeneous texture.

You answered Cystadenoma

The correct answer is Liver cell adenoma


Liver cell adenomas are linked to OCP use and 90% of patients with liver cell adenomas have
used the OCP. Liver function tests are often normal. The lesions will typically have a mixed
echoity and heterogeneous texture.

12. A 38 year old shepherd presents to the clinic with a 3 month history of malaise and right upper
quadrant pain. On examination he is mildly jaundiced. His liver function tests demonstrate a mild
elevation in bilirubin and transaminases, his full blood count shows an elevated eosinophil level.
An abdominal x-ray is performed by the senior house officer and demonstrates a calcified lesion
in the right upper quadrant of the abdomen.

Hydatid cyst

Similar theme in September 2011 Exam

Hyatid disease is more common in those who work with sheep or dogs. Liver function tests may
be abnormal and an eosinophilia is often present. Plain radiographs may reveal a calcified cyst
wall.

Benign liver lesions

Benign liver lesions


Haemangioma • Most common benign tumours of mesenchymal origin
• Incidence in autopsy series is 8%
• Cavernous haemangiomas may be enormous
• Clinically they are reddish purple hypervascular lesions
• Lesions are normally separated from normal liver by ring of fibrous tissue
• On ultrasound they are typically hyperechoic

Liver cell • 90% develop in women in their third to fifth decade


adenoma • Linked to use of oral contraceptive pill
• Lesions are usually solitary
• They are usually sharply demarcated from normal liver although they usually
lack a fibrous capsule
• On ultrasound the appearances are of mixed echoity and heterogeneous
texture. On CT most lesions are hypodense when imaged prior to
administration of IV contrast agents
• In patients with haemorrhage or symptoms removal of the adenoma may be
required

Mesenchymal Congential and benign, usually present in infants. May compress normal liver
hamartomas
Liver abscess • Biliary sepsis is a major predisposing factor
• Structures drained by the portal venous system form the second largest
source
• Common symptoms include fever, right upper quadrant pain. Jaundice may
be seen in 50%
• Ultrasound will usually show a fluid filled cavity, hyperechoic walls may be
seen in chronic abscesses

Amoebic abscess • Liver abscess is the most common extra intestinal manifestation of
amoebiasis
• Between 75 and 90% lesions occur in the right lobe
• Presenting complaints typically include fever and right upper quadrant pain
• Ultrasonography will usually show a fluid filled structure with poorly defined
boundaries
• Aspiration yield sterile odourless fluid which has an anchovy paste
consistency
• Treatment is with metronidazole

Hyatid cysts • Seen in cases of Echinococcus infection


• Typically an intense fibrotic reaction occurs around sites of infection
• The cyst has no epithelial lining
• Cysts are commonly unilocular and may grow to 20cm in size. The cyst wall
is thick and has an external laminated hilar membrane and an internal
enucleated germinal layer
• Typically presents with malaise and right upper quadrant pain. Secondary
bacterial infection occurs in 10%.
• Liver function tests are usually abnormal and eosinophilia is present in 33%
cases
• Ultrasound may show septa and hyatid sand or daughter cysts.
• Percutaneous aspiration is contra indicated
• Treatment is by sterilisation of the cyst with mebendazole and may be
followed by surgical resection. Hypertonic swabs are packed around the cysts
during surgery

Polycystic liver • Usually occurs in association with polycystic kidney disease


disease • Autosomal dominant disorder
• Symptoms may occur as a result of capsular stretch

Cystadenoma • Rare lesions with malignant potential


• Usually solitary multiloculated lesions
• Liver function tests usually normal
• Ultrasonography typically shows a large anechoic, fluid filled area with
irregular margins. Internal echos may result from septa
• Surgical resection is indicated in all cases
A 5 year old boy presents to the clinic with short stature suggestive of achondroplasia. What is the
genetic basis of this condition?

A. X linked defect

B. Y linked defect

C. YY linked defect

D. Autosomal dominant defect

E. Autosomal recessive defect

Theme from April 2013 Exam


Achondroplasia usually occurs as a sporadic mutation. It is then transmitted in an autosomal dominant
fashion.

Achondroplasia

Achondroplasia is a common cause of dwarfism and is caused by defects in the fibroblast growth factor
receptor. In most cases (approximately 70%) it occurs as a sporadic mutation. The main risk factor is
advancing parental age at the time of conception. Once present it is typically inherited in an autosomal
dominant fashion.

Radiological features

• Large skull with narrow foramen magnum


• Short, flattened intervertebral bodies
• Narrow spinal canal
• Horizontal acetabular roof
• Broad, short metacarpals

Treatment
There is no specific therapy. However, some individuals benefit from limb lengthening procedures.
These usually involve application of Ilizarov frames and targeted bone fractures. A clearly defined need
and end point is the cornerstone of achieving success with such procedures.

A 72 year old man presents with symptoms and signs of benign prostatic hyperplasia. Which of the
following structures is most likely to be enlarged on digital rectal examination?
A. Posterior lobe of the prostate

B. Median lobe of the prostate

C. Right lateral lobe of the prostate

D. Left lateral lobe of the prostate

E. Anterior lobe of the prostate

Carcinoma of the prostate typically occurs in the posterior lobe. The median lobe is usually enlarged in
BPH. The anterior lobe has little in the way of glandular tissue and is seldom enlarged.

Benign Prostatic Hyperplasia

Benign prostatic hyperplasia occurs via an increase in the epithelial and stromal cell numbers in the peri-
urethral zone of the prostate. BPH is very common and 90% of men aged over 80 will have at least
microscopic evidence of benign prostatic hyperplasia. The causes of BPH are still not well understood,
but the importance of androgens remains appreciated even if the exact role by which they induce BPH is
elusive.

Presentation
The vast majority of men will present with lower urinary tract symptoms. These will typically be:

• Poor flow
• Nocturia
• Hesitancy
• Incomplete and double voiding
• Terminal dribbling
• Urgency
• Incontinence

Investigation

• Digital rectal examination to assess prostatic size and morphology.


• Urine dipstick for infections and haematuria.
• Uroflowmetry (a flow rate of >15ml/second helps to exclude BOO)
• Bladder pressure studies may help identify detrusor failure and whilst may not form part of first
line investigations should be included in those with atypical symptoms and prior to redo surgery.
• Bladder scanning to demonstrate residual volumes. USS if high pressure chronic retention.

Management
• Lifestyle changes such as stopping smoking and altering fluid intake may help those with mild
symptoms.
• Medical therapy includes alpha blockers and 5 α reductase inhibitors. The former work quickly
on receptor zones located at the bladder neck. Cardiovascular side effects are well documented.
The latter work on testosterone metabolising enzymes. Although they have a slower onset of
action, the 5 α reductase inhibitors may prevent acute urinary retention.
• Surgical therapy includes transurethral resection of the prostate and is the treatment of choice in
those with severe symptoms and those who fail to respond to medical therapy. More tailored
bladder neck incision procedures may be considered in those with small prostates. Retrograde
ejaculation may occur following surgery. The change in the type of irrigation solutions used has
helped to minimise the TURP syndrome of electrolyte disturbances.

A 58 year old man has been suffering from mechanical back pain for several years. One morning he
awakes from sleep and feels a sudden onset of pain in his back radiating down his left leg. Which of the
following events is most likely to account for his symptoms?

A. Prolapse of inner annulus fibrosus

B. Prolapse of outer annulus fibrosus

C. Prolapse of nucleus pulposus

D. Rupture of the ligamentum flavum

E. None of the above

Theme from 2009 Exam


Theme from September 2012 Exam
The symptoms would be most likely the result of intervertebral disk prolapse. In disk prolapse the
nucleus pulposus is the structure which usually herniates.

Intervertebral discs

• Consist of an outer annulus fibrosus and an inner nucleus pulposus.


• The anulus fibrosus consists of several layers of fibrocartilage.
• The nucleus pulposus contains loose fibres suspended in a mucoprotein gel with the consistency
of jelly. The nucleus of the disc acts as a shock absorber.
• Pressure on the disc causes posterior protrusion of the nucleus pulposus. Most commonly in the
lumbrosacral and lower cervical areas.
• The discs are separated by hyaline cartilage.
• There is one disc between each pair of vertebrae, except for C1/2 and the sacrococcygeal
vertebrae.

Theme: Paediatric neck masses


A. Cystic hygroma
B. Thyroglossal cyst
C. Rhabdomyosarcoma
D. Branchial cyst
E. Dermoid cyst

Please select the most likely underlying diagnosis for the situation that is described. Each option may be
used once, more than once, or not at all.

16. A 2 year old boy is brought to the clinic by his mother who has noticed that he has developed a
small mass. On examination a small smooth cyst is identified which is located above the hyoid
bone. On ultrasound the lesion appears to be a heterogenous and multiloculated mass.

You answered Thyroglossal cyst

The correct answer is Dermoid cyst

Dermoid cysts are usually multiloculated and heterogeneous. Most are located above the hyoid
and their appearances on imaging differentiate them from thyroglossal cysts.

17. A 22 month old baby is brought to the clinic by her mother who is concerned that she has
developed a swelling in her neck. On examination she has a soft, lesion located in the posterior
triangle that transilluminates.

Cystic hygroma

Cystic hygromas are soft and transilluminate. Most are located in the posterior triangle.

18. A 3 year old boy is brought to the clinic by his mother who has noticed a mass in his neck. On
examination he has a smooth mass located on the lateral aspect of his anterior triangle, near to the
angle of the mandible. On ultrasound it has a fluid filled, anechoic, appearance.

Branchial cyst

Branchial cysts are usually located laterally and derived from the second branchial cleft. Unless
infection has occurred they will usually have an anechoic appearance on ultrasound.

Neck Masses in Children


Thyroglossal cyst • Located in the anterior triangle, usually in the midline and below the
hyoid (65% cases)
• Derived from remnants of the thyroglossal duct
• Thin walled and anechoic on USS (echogenicity suggests infection of
cyst)

Branchial cyst • Six branchial arches separated by branchial clefts


• Incomplete obliteration of the branchial apparatus may result in cysts,
sinuses or fistulae
• 75% of branchial cysts originate from the second branchial cleft
• Usually located anterior to the sternocleidomastoid near the angle of
the mandible
• Unless infected the fluid of the cyst has a similar consistency to water
and is anechoic on USS

Dermoids • Derived from pleuripotent stem cells and are located in the midline
• Most commonly in a suprahyoid location
• They have heterogeneous appearances on imaging and contain variable
amounts of calcium and fat

Thyroid gland • True thyroid lesions are rare in children and usually represent
thyroglossal cysts or tumours like lymphoma

Lymphatic • Usually located posterior to the sternocleidomastoid


malformations • Cystic hygroma result from occlusion of lymphatic channels
• The painless, fluid filled, lesions usually present prior to the age of 2
• They are often closely linked to surrounding structures and surgical
removal is difficult
• They are typically hypoechoic on USS

Infantile haemangioma • May present in either triangle of the neck


• Grow rapidly initially and then will often spontaneously regress
• Plain x-rays will show a mass lesion, usually containing calcified
phleboliths
• As involution occurs the fat content of the lesions increases

Lymphadenopathy • Located in either triangle of the neck


• May be reactive or neoplastic
• Generalised lymphadenopathy usually secondary to infection in
children (very common)

A 34 year old man presents to the surgical clinic 8 months following a laparotomy for a ruptured spleen.
He complains of a nodule in the centre of his laparotomy wound. This is explored surgically and a stitch
granuloma is found and excised. From which of the following cell types do granulomata arise?
A. Polymorpho nucleocytes

B. Plasma cells

C. Reed- Sternberg cells

D. Platelets

E. Macrophages
Granulomas are organised collections
of macrophages

Theme from 2011 Exam

Macrophages give origin to granulomas.

Chronic inflammation

Overview
Chronic inflammation may occur secondary to acute inflammation.In most cases chronic inflammation
occurs as a primary process. These may be broadly viewed as being one of three main processes:

• Persisting infection with certain organisms such as Mycobacterium tuberculosis which results in
delayed type hypersensitivity reactions and inflammation.
• Prolonged exposure to non-biodegradable substances such as silica or suture materials which
may induce an inflammatory response.
• Autoimmune conditions involving antibodies formed against host antigens.

Acute vs. Chronic inflammation


Acute inflammation Chronic inflammation
Changes to existing vascular structure and increased Angiogenesis predominates
permeability of endothelial cells
Infiltration of neutrophils Macrophages, plasma cells and
lymphocytes predominate
Process may resolve with: Healing by fibrosis is the main result

• Suppuration
• Complete resolution
• Abscess formation
• Progression to chronic inflammation
• Healing by fibrosis
Granulomatous inflammation
A granuloma consists of a microscopic aggregation of macrophages (with epithelial type arrangement
=epitheliod). Large giant cells may be found at the periphery of granulomas.

Mediators
Growth factors released by activated macrophages include agents such as interferon and fibroblast
growth factor (plus many more). Some of these such as interferons may have systemic features resulting
in systemic symptoms and signs, which may be present in individuals with long standing chronic
inflammation.

The finding of granulomas is pathognomonic of chronic inflammation, as illustrated in this biopsy from
a patient with colonic Crohns disease

An unusually tall 43 year old lady presents to the surgical clinic with bilateral inguinal hernias. She
develops chest pain and collapses. As part of her investigations a chest x-ray shows evidence of
mediastinal widening. What is the most likely underlying diagnosis?

A. Pulmonary embolus

B. Aortic dissection

C. Tietze syndrome

D. Boerhaaves syndrome

E. Myocardial infarct

Marfans syndrome may present with a variety of connective tissue disorders such as bilateral inguinal
hernia. They are at high risk of aortic dissection, as in this case.
Aortic dissection

• More common than rupture of the abdominal aorta


• 33% of patients die within the first 24 hours, and 50% die within 48 hours if no treatment
received
• Associated with hypertension
• Features of aortic dissection: tear in the intimal layer, followed by formation and propagation of
a subintimal hematoma. Cystic medial necrosis (Marfan's)
• Most common site of dissection: 90% occurring within 10 centimetres of the aortic valve

Stanford Classification
Type Location Treatment
A Ascending aorta/ aortic root Surgery- aortic root replacement
B Descending aorta Medical therapy with antihypertensives

DeBakey classification
Type Site affected
I Ascending aorta, aortic arch, descending aorta
II Ascending aorta only
III Descending aorta distal to left subclavian artery

Clinical features

• Tearing, sudden onset chest pain (painless 10%)


• Hypertension or Hypotension
• A blood pressure difference greater than 20 mm Hg
• Neurologic deficits (20%)

Investigations

• CXR: widened mediastinum, abnormal aortic knob, ring sign, deviation trachea/oesophagus
• CT (spiral)
• MRI
• Angiography (95% of patients diagnosed)

Management

• Beta-blockers: aim HR 60-80 bpm and systolic BP 100-120 mm Hg.


• Urgent surgical intervention: type A dissections. This will usually involve aortic root
replacement.
• A 72 year old man has just undergone an emergency repair for a ruptured abdominal aortic
aneurysm. Pre operatively he was taking aspirin, clopidogrel and warfarin. Intra operatively he
received 5000 units of unfractionated heparin prior to application of the aortic cross clamp. His
blood results on admission to the critical care unit are as follows:
Full blood count
Hb 8 g/dl
Platelets 40 * 109/l
WBC 7.1 * 109/l
• His fibrin degradation products are measured and found to be markedly elevated. Which of the
following accounts for these results?

A. Anastomotic leak

B. Disseminated intravascular coagulation

C. Heparin induced thrombocytopenia

D. Adverse effect of warfarin

E. Adverse effects of antiplatelet agents



Theme from April 2012 Exam
The combination of low platelet counts and raised FDP in this setting make DIC the most likely
diagnosis.
• Disseminated intravascular coagulation - Diagnosis

Under homeostatic conditions, coagulation and fibrinolysis are coupled. The activation of the
coagulation cascade yields thrombin that converts fibrinogen to fibrin; the stable fibrin clot being
the final product of hemostasis. The fibrinolytic system breaks down fibrinogen and fibrin.
Activation of the fibrinolytic system generates plasmin (in the presence of thrombin), which is
responsible for the lysis of fibrin clots. The breakdown of fibrinogen and fibrin results in
polypeptides (fibrin degradation products). In a state of homeostasis, the presence of plasmin is
critical, as it is the central proteolytic enzyme of coagulation and is also necessary for
fibrinolysis.

In DIC, the processes of coagulation and fibrinolysis are dysregulated, and the result is
widespread clotting with resultant bleeding. Regardless of the triggering event of DIC, once
initiated, the pathophysiology of DIC is similar in all conditions. One critical mediator of DIC is
the release of a transmembrane glycoprotein (tissue factor =TF). TF is present on the surface of
many cell types (including endothelial cells, macrophages, and monocytes) and is not normally
in contact with the general circulation, but is exposed to the circulation after vascular damage.
For example, TF is released in response to exposure to cytokines (particularly interleukin 1),
tumor necrosis factor, and endotoxin. This plays a major role in the development of DIC in septic
conditions. TF is also abundant in tissues of the lungs, brain, and placenta. This helps to explain
why DIC readily develops in patients with extensive trauma. Upon activation, TF binds with
coagulation factors that then triggers the extrinsic pathway (via Factor VII) which subsequently
triggers the intrinsic pathway (XII to XI to IX) of coagulation.

Diagnosis
Fibrin degradation products are often raised.

Disorder Prothrombin time APTT Bleeding time Platelet count


Warfarin administration Prolonged Normal Normal Normal
Aspirin administration Normal Normal Prolonged Normal
Heparin Often normal (may be prolonged) Prolonged Normal Normal
DIC Prolonged Prolonged Prolonged Low

A 53 year old man from Hong Kong presents with symptoms of fatigue, weight loss and recurrent
epistaxis. Clinical examination reveals left sided cervical lymphadenopathy and oropharyngeal
examination reveals an ulcerated mass in the naso pharynx. Which of the following viral agents is most
commonly implicated in the development of this condition?

A. Cytomegalovirus

B. Epstein Barr virus

C. Coxsackie virus

D. Herpes simplex virus

E. None of the above

The clinical scenario is most typical for nasopharyngeal carcinoma. An association with previous
Epstein Barr Virus is well established. Infection with the other viruses listed is not a recognised risk
factor for the development of the condition.

Nasopharyngeal carcinoma

• Squamous cell carcinoma of the nasopharynx


• Rare in most parts of the world, apart from individuals from Southern China
• Associated with Epstein Barr virus infection

Presenting features
Systemic Local
Cervical lymphadenopathy Otalgia
Unilateral serous otitis media
Nasal obstruction, discharge and/ or epistaxis
Cranial nerve palsies e.g. III-VI

Imaging
Combined CT and MRI.

Treatment
Radiotherapy is first line therapy.

An 18 year old male presents with lethargy, night sweats and on examination is found to have left
supraclavicular lymphadenopathy. A surgical registrar performs a left supraclavicular lymph node
biopsy. The pathologist identifies Reed- Sternberg cells on the subsequent histology sections, what is the
most likely diagnosis?

A. Metastatic gastric cancer

B. Hodgkins lymphoma

C. Non Hodgkins lymphoma

D. Tuberculosis

E. None of the above

Reed-Sternberg cells are characteristic histological cell type found in Hodgkins disease.

Lymphadenopathy

• Lymphadenopathy in the neck, axillae, groins and abdomen


• Need to note: solitary/multiple, defined/indistinct, hard/rubbery/soft, tender/painless

Causes of lymphadenopathy

Mnemonic: Hodgkins disease

H aematological: Hodgkins lymphoma, NHL, Leukaemia


O ncological: metastases
D ermatopathic lympadenitis
G aucher's disease
K awasaki disease
I nfections: TB, glandular fever, Syphilis
N iemann Pick disease
S erum sickness
D rug reaction (phenytoin)
I mmunological (SLE)
S arcoidosis
E ndocrinological (Hyperthyroidism)
A ngioimmunoplastic lymphadenopathy
S LE
E osinophilic granulomatosis

Which of the following lesions is least likely to occur in the presence of severe atrophic gastritis?

A. Duodenal ulcer

B. Gastric cancer

C. Gastric polyp

D. Iron deficiency anaemia

E. Pernicious anaemia

Due the absence of acid a duodenal ulcer is unlikely to occur.

Gastric cancer

Overview
There are 700,000 new cases of gastric cancer worldwide each year. It is most common in Japan and less
common in western countries. It is more common in men and incidence rises with increasing age. The
exact cause of many sporadic cancer is not known, however, familial cases do occur in HNPCC
families. In addition, smoking and smoked or preserved foods increase the risk. Japanese migrants retain
their increased risk (decreased in subsequent generations). The distribution of the disease in western
countries is changing towards a more proximal location (perhaps due to rising obesity).

Pathology
There is some evidence of support a stepwise progression of the disease through intestinal metaplasia
progressing to atrophic gastritis and subsequent dysplasia, through to cancer. The favoured staging
system is TNM. The risk of lymph node involvement is related to size and depth of invasion; early
cancers confined to submucosa have a 20% incidence of lymph node metastasis. Tumours of the gastro-
oesophageal junction are classified as below:

Type True oesophageal cancers and may be associated with Barrett's oesophagus.
1
Type Carcinoma of the cardia, arising from cardiac type epithelium
2 or short segments with intestinal metaplasia at the oesophagogastric junction.
Type Sub cardial cancers that spread across the junction. Involve similar nodal stations to gastric
3 cancer.

Groups for close endoscopic monitoring

• Intestinal metaplasia of columnar type


• Atrophic gastritis
• Low to medium grade dysplasia
• Patients who have previously undergone resections for benign peptic ulcer disease (except highly
selective vagotomy).

Referral to endoscopy

Patients of any age with dyspepsia Patients without Worsening dyspepsia


and any of the following dyspepsia
Chronic gastrointestinal bleeding Dysphagia Barretts oesophagus
Dysphagia Unexplained abdominal Intestinal metaplasia
pain or weight loss
Weight loss Vomiting Dysplasia
Iron deficiency anaemia Upper abdominal mass Atrophic gastritis
Upper abdominal mass Jaundice Patient aged over 55 years with
unexplained or persistent dyspepsia

Upper GI endoscopy performed for dyspepsia. The addition of dye spraying (as shown in the bottom
right) may facilitate identification of smaller tumours
Image sourced from Wikipedia

Staging

• CT scanning of the chest abdomen and pelvis is the routine first line staging investigation in
most centres.
• Laparoscopy to identify occult peritoneal disease
• PET CT (particularly for junctional tumours)

Treatment

• Proximally sited disease greater than 5-10cm from the OG junction may be treated by sub total
gastrectomy
• Total gastrectomy if tumour is <5cm from OG junction
• For type 2 junctional tumours (extending into oesophagus) oesophagogastrectomy is usual
• Endoscopic sub mucosal resection may play a role in early gastric cancer confined to the mucosa
and perhaps the sub mucosa (this is debated)
• Lymphadenectomy should be performed. A D2 lymphadenectomy is widely advocated by the
Japanese, the survival advantages of extended lymphadenectomy have been debated. However,
the overall recommendation is that a D2 nodal dissection be undertaken.
• Most patients will receive chemotherapy either pre or post operatively.
Prognosis

UK Data

Disease extent Percentage 5 year survival


All RO resections 54%
Early gastric cancer 91%
Stage 1 87%
Stage 2 65%
Stage 3 18%

Operative procedure

Total Gastrectomy , lymphadenectomy and Roux en Y anastomosis

General anaesthesia
Prophylactic intravenous antibiotics
Incision: Rooftop.
Perform a thorough laparotomy to identify any occult disease.
Mobilise the left lobe of the liver off the diaphragm and place a large pack over it. Insert a large self
retaining retractor e.g. omnitract or Balfour (take time with this, the set up should be perfect). Pack the
small bowel away.
Begin by mobilising the omentum off the transverse colon.
Proceed to detach the short gastric vessels.
Mobilise the pylorus and divide it at least 2cm distally using a linear cutter stapling device.
Continue the dissection into the lesser sac taking the lesser omentum and left gastric artery flush at its
origin.
The lymph nodes should be removed en bloc with the specimen where possible.
Place 2 stay sutures either side of the distal oesophagus. Ask the anaesthetist to pull back on the
nasogastric tube. Divide the distal oesophagus and remove the stomach.
The oesphago jejunal anastomosis should be constructed. Identify the DJ flexure and bring a loop of
jejunum up to the oesophagus (to check it will reach). Divide the jejunum at this point. Bring the divided
jejunum either retrocolic or antecolic to the oesophagus. Anastamose the oesophagus to the jejunum,
using either interrupted 3/0 vicryl or a stapling device. Then create the remainder of the Roux en Y
reconstruction distally.
Place a jejunostomy feeding tube.
Wash out the abdomen and insert drains (usually the anastomosis and duodenal stump). Help the
anaesthetist insert the nasogastric tube (carefully!)
Close the abdomen and skin.
Enteral feeding may commence on the first post-operative day. However, most surgeons will leave
patients on free NG drainage for several days and keep them nil by mouth.
Question 25 of 328
A 28 year old man develops an acute paronychia and subsequent spreading sepsis. The tissue exudate
has a higher protein content than normal tissue because?

A. Breakdown of tissue cells release protein

B. Capillary walls are more permeable

C. Increased blood flow transports more protein into the area

D. Intracapillary pressure is raised

E. Plasma cells release gamma globulin

The increased permeability allows the exudation of plasma proteins.

Acute inflammation

Inflammation is the reaction of the tissue elements to injury. Vascular changes occur, resulting in the
generation of a protein rich exudate. So long as the injury does not totally destroy the existing tissue
architecture, the episode may resolve with restoration of original tissue architecture.

Vascular changes

• Vasodilation occurs and persists throughout the inflammatory phase.


• Inflammatory cells exit the circulation at the site of injury.
• The equilibrium that balances Starlings forces within capillary beds is disrupted and a protein
rich exudate will form as the vessel walls also become more permeable to proteins.
• The high fibrinogen content of the fluid may form a fibrin clot. This has several important
immunomodulatory functions.

Sequelae
Resolution • Typically occurs with minimal initial injury
• Stimulus removed and normal tissue architecture results

Organisation • Delayed removed of exudate


• Tissues undergo organisation and usually fibrosis

Suppuration • Typically formation of an abscess or an empyema


• Sequestration of large quantities of dead neutrophils

Progression to chronic • Coupled inflammatory and reparative activities


inflammation • Usually occurs when initial infection or suppuration has been
inadequately managed

Causes

• Microbacterial infections e.g. Viruses, exotoxins or endotoxins released by bacteria


• Chemical agents
• Physical agents e.g. Trauma
• Hypersensitivity reactions
• Tissue necrosis

Presence of neutrophil polymorphs is a histological diagnostic feature of acute inflammation

A 40 year old man undergoes a complex appendicectomy and the wound fails to heal satisfactorily. The
wound site itself is associated with multiple sinuses and fistulas. Pus is sent for microbiology and shows
gram positive organisms and sulphur granules. What is the most likely underlying diagnosis?

A. Infection with Staphylococcus aureus

B. Infection with Bacteroides fragilis

C. Actinomycosis

D. Crohns disease

E. Ulcerative colitis

The presence of chronic sinuses together with gram positive organisms and sulphur granules is highly
suggestive of Actinomycosis. Crohns disease is associated with multiple fistulae, but not gram positive
organisms with sulphur granules.

Actinomycosis

Chronic, progressive granulomatous disease caused by filamentous gram positive anaerobic bacteria
from the Actinomycetaceae family.

Actinomyces are commensal bacteria that become pathogenic when a mucosal barrier is breached.

The disease most commonly occurs in the head and neck, although it may also occur in the abdominal
cavity and in the thorax.

The mass will often enlarge across tissue planes with the formation of multiple sinus tracts.
Abdominopelvic actinomycosis occurs most frequently in individuals that have had appendicitis (65%)
cases.

Pathology

• On histological examination gram positive organisms and evidence of sulphur granules.


• Sulphur granules are colonies of organisms that appear as round or oval basophilic masses.
• They are also seen in other conditions such as nocardiosis.

Treatment

• Long term antibiotic therapy usually with penicillin.


• Surgical resection is indicated for extensive necrotic tissue, non healing sinus tracts, abscesses or
where biopsy is needed to exclude malignancy.
• As a busy surgical trainee on the colorectal unit you are given the unenviable task of reviewing
the unit's histopathology results for colonic polyps. Which of the polyp types described below
has the greatest risk of malignancy?

A. Hyperplastic polyp

B. Tubular adenoma

C. Villous adenoma

D. Hamartomatous polyp

E. Serrated polyp

Villous adenomas carry the highest risk of malignant transformation. Hyperplastic polyps carry
little in the way of increased risk. Although, patients with hamartomatous polyp syndromes may
have a high risk of malignancy, the polyps themselves have little malignant potential.
• Colonic polyps

Colonic Polyps
May occur in isolation of greater numbers as part of the polyposis syndromes. In FAP greater
than 100 polyps are typically present. The risk of malignancy in association with adenomas is
related to size and is the order of 10% in a 1cm adenoma. Isolated adenomas seldom give risk of
symptoms (unless large and distal). Distally sited villous lesions may produce mucous and if
very large electrolyte disturbances may occur.

Follow up of colonic polyps


Group Features Action
Low risk 1 or 2 adenomas less than 1cm No follow up or re-colonoscopy
at 5 years
Moderate 3 or 4 small adenomas or 1 adenoma greater than 1cm Re-scope at 3 years
risk
High risk More than 5 small adenomas or more than 3 with 1 of Re scope at 1 year
them greater than 1cm
• From Atkins and Saunders Gut 2002 51 (suppl V:V6-V9). It is important to stratify patients
appropriately and ensure that a complete colonoscopy with good views was performed.

Segmental resection or complete colectomy should be considered when:

1. Incomplete excision of malignant polyp


2. Malignant sessile polyp
3. Malignant pedunculated polyp with submucosal invasion
4. Polyps with poorly differentiated carcinoma
5. Familial polyposis coli
-Screening from teenager up to 40 years by 2 yearly sigmoidoscopy/colonoscopy
-Panproctocolectomy and Ileostomy or Restorative Panproctocolectomy.

Rectal polypoidal lesions may be amenable to trans anal endoscopic microsurgery.

A 23 year old man presents to the surgical clinic with an inguinal hernia. On examination he has a small
direct hernia. However, you also notice that he has pigmented spots around his mouth, on his palms and
soles. In his history he underwent a reduction of an intussusception aged 12 years. Which of the
following lesions is most likely to be identified if a colonoscopy were performed?

A. Hamartomas

B. Tubulovillous adenoma

C. Colorectal cancer

D. Crohns disease

E. Hyperplastic polyps

Theme from April 2012 Exam


Theme from January 2013 Exam
He is most likely to have Peutz-Jeghers syndrome which is associated with Hamartomas.

Peutz-Jeghers syndrome

Peutz-Jeghers syndrome is an autosomal dominant condition characterised by numerous benign


hamartomatous polyps in the gastrointestinal tract. It is also associated with pigmented freckles on the
lips, face, palms and soles. Around 50% of patients will have died from a gastrointestinal tract cancer by
the age of 60 years.

Genetics

• Autosomal dominant
• Responsible gene encodes serine threonine kinase LKB1 or STK11

Features

• Hamartomatous polyps in GI tract (mainly small bowel)


• Pigmented lesions on lips, oral mucosa, face, palms and soles
• Intestinal obstruction e.g. intussusception (which may lead to diagnosis)
• Gastrointestinal bleeding

Management

• Conservative unless complications develop

A 56 year old surgeon has been successfully operating for many years. Over the past few weeks she has
begun to notice that her hands are becoming blistering and weepy. A latex allergy is diagnosed. Which
of the following pathological processes accounts for this scenario?

A. Type 1 hypersensitivity reaction

B. Type 2 hypersensitivity reaction

C. Type 4 hypersensitivity reaction

D. Type 3 hypersensitivity reaction

E. None of the above


Hypersensitivity reactions:
ACID

type 1 --Anaphylactic
type 2 --Cytotoxic
type 3 --Immune complex
type 4 --Delayed hypersensitivity

Theme from 2012 Exam


Contact dermatitis of a chronic nature is an example of a type 4 hypersensitivity reaction. Type 4
hypersensitivity reactions are cell mediated rather than antibody mediated.

Hypersensitivity reactions

The Gell and Coombs classification divides hypersensitivity reactions into 4 types

Type I Type II Type III Type IV


Description Anaphylactic Cytotoxic Immune Delayed type
complex
Mediator IgE IgG, IgM IgG, Ig A, IgM T-cells
Antigen Exogenous Cell surface Soluble Tissues
Response Minutes Hours Hours 2-3 days
time
Examples Asthma Autoimmune haemolytic Serum sickness Graft versus host
Hay fever anaemia SLE disease
Pemphigus Aspergillosis Contact dermatitis
Goodpasture's
A 56 year old motorcyclist is involved in a road traffic accident and sustains a displaced femoral shaft
fracture. Not other injuries are identified on the primary or secondary surveys. The fracture is treated
with closed, antegrade intramedullary nailing. The following day the patient becomes increasingly
agitated and confused. On examination he is pyrexial, hypoxic SaO2 90% on 6 litres O2, tachycardic and
normotensive. Systemic examination demonstrates a non blanching petechial rash present over the torso.
What is the most likely explanation for this?

A. Pulmonary embolism with paradoxical embolus

B. Fat embolism

C. Meningococcal sepsis

D. Alcohol withdrawl

E. Chronic sub dural haematoma

This man has a recent injury and physical signs that would be concordant with fat embolism syndrome.
Meningococcal sepsis is not usually associated with hypoxia initially. Pulmonary emboli are not
typically associated with pyrexia.

Fat embolism

Diagnosis and clinical features


System Feature
Cardiothoracic • Early persistent tachycardia
• Tachypnoea, dyspnoea, hypoxia usually 72 hours following injury
• Pyrexia

Dermatological • Red/ brown impalpable petechial rash (usually only in 25-50%)


• Subconjunctival and oral haemorrhage/ petechiae

CNS • Confusion and agitation


• Retinal haemorrhages and intra-arterial fat globules on fundoscopy

Imaging

• May be normal
• Fat emboli tend to lodge distally and therefore CTPA may not show any vascular occlusion, a
ground glass appearance may be seen at the periphery

Treatment

• Prompt fixation of long bone fractures


• Some debate regarding benefit Vs. risk of medullary reaming in femoral shaft/ tibial fractures in
terms of increasing risk (probably does not).
• DVT prophylaxis
• General supportive care

A 43 year old man presents with haemoptysis and is diagnosed as having tuberculosis. Which of the cell
types listed below will usually internalise the tubercule bacullis?

A. Fibroblast

B. Neutrophil

C. Erythrocyte

D. Macrophage

E. Eosinophil

Theme from January 2013 Exam


M. Tuberculosis will reside in macrophages where it will often survive

Tuberculosis pathology
• Is a form of primary chronic inflammation, caused by the inability of macrophages to kill the
Mycobacterium tuberculosis.
• The macrophages often migrate to regional lymph nodes, the lung lesion plus affected lymph
nodes is referred to as a Ghon complex.
• This leads to the formation of a granuloma which is a collection of epithelioid histiocytes.
• There is the presence of caseous necrosis in the centre.
• The inflammatory response is mediated by a type 4 hypersensitivity reaction.
• In healthy individuals the disease may be contained, in the immunocompromised disseminated
(miliary TB) may occur.

Diagnosis

• Waxy membrane of mycobacteria prevents binding with normal stains. Ziehl - Neelsen staining
is typically used.
• Culture based methods take far longer.

Image showing acid- alcohol fast mycobacteria stained using the Ziehl- Neelsen method

Which of these tumour markers is most helpful in identifying an individual with hepatocellular
carcinoma?

A. Serum AFP

B. Serum CA19-9

C. CEA

D. Beta HCG

E. CA125
Theme from September 2011 Exam
Hepatocellular carcinoma is commonly diagnosed with imaging and an elevated alpha fetoprotein.
Biopsy may seed the tumour and should be avoided. Up to 80% of hepatocellular carcinoma arise in
cirrhotic livers.

Liver tumours

Primary liver tumours


The most common primary tumours are cholangiocarcinoma and hepatocellular carcinoma. Overall
metastatic disease accounts for 95% of all liver malignancies making the primary liver tumours
comparatively rare.

Primary liver tumours include:

• Cholangiocarcinoma
• Hepatocellular carcinoma
• Hepatoblastoma
• Sarcomas (Rare)
• Lymphomas
• Carcinoids (most often secondary although primary may occur)

Hepatocellular carcinoma
These account for the bulk of primary liver tumours (75% cases). Its worldwide incidence reflects its
propensity to occur on a background of chronic inflammatory activity. Most cases arise in cirrhotic
livers or those with chronic hepatitis B infection, especially where viral replication is actively occurring.
In the UK it accounts for less than 5% of all cancers, although in parts of Asia its incidence is 100 per
100,000.
The majority of patients (80%) present with existing liver cirrhosis, with a mass discovered on screening
ultrasound.

Diagnosis

• CT/ MRI (usually both) are the imaging modalities of choice


• a-fetoprotein is elevated in almost all cases
• Biopsy should be avoided as it seeds tumours cells through a resection plane.
• In cases of diagnostic doubt serial CT and aFP measurements are the preferred strategy.

Treatment

• Patients should be staged with liver MRI and chest, abdomen and pelvic CT scan.
• The testis should be examined in males (testicular tumours may cause raised AFP). PET CT may
be used to identify occult nodal disease.
• Surgical resection is the mainstay of treatment in operable cases. In patients with a small primary
tumour in a cirrhotic liver whose primary disease process is controlled, consideration may be
given to primary whole liver resection and transplantation.
• Liver resections are an option but since most cases occur in an already diseased liver the
operative risks and post-operative hepatic dysfunction are far greater than is seen following
metastectomy.
• These tumours are not particularly chemo or radiosensitive however, both may be used in a
palliative setting. Tumour ablation is a more popular strategy.

Survival
Poor, overall survival is 15% at 5 years.

Cholangiocarcinoma
This is the second most common type of primary liver malignancy. As its name suggests these tumours
arise in the bile ducts. Up to 80% of tumours arise in the extra hepatic biliary tree. Most patients present
with jaundice and by this stage the majority will have disease that is not resectable.
Primary scelerosing cholangitis is the main risk factor. In deprived countries typhoid and liver flukes are
also major risk factors.

Diagnosis

• Patients will typically have an obstructive picture on liver function tests.


• CA 19-9, CEA and CA 125 are often elevated
• CT/ MRI and MRCP are the imaging methods of choice.

Treatment

• Surgical resection offers the best chance of cure. Local invasion of peri hilar tumours is a
particular problem and this coupled with lobar atrophy will often contra indicate surgical
resection.
• Palliation of jaundice is important, although metallic stents should be avoided in those
considered for resection.

Survival
Is poor, approximately 15% 5 year survival.

A 39 year old man has suffered from terminal ileal Crohns disease for the past 20 years. Which
condition is he least likely to develop?

A. Gallstones
B. Malabsorption

C. Pyoderma gangrenosum

D. Amyloidosis

E. Feltys syndrome
Felteys syndrome:

• Rheumatoid disease
• Splenomegaly
• Neutropenia

Feltys syndrome is associated with rheumatoid disease. Individuals with long standing crohns disease
are at risk of gallstones because of impairment of the enterohepatic recycling of bile salts. Formation of
entero-enteric fistulation may produce malabsorption. Amyloidosis may complicate chronic
inflammatory states.

Crohns disease

Crohns disease is a chronic transmural inflammation of a segment(s) of the gastrointestinal tract and
may be associated with extra intestinal manifestations. Frequent disease patterns observed include ileal,
ileocolic and colonic disease. Peri-anal disease may occur in association with any of these. The disease
is often discontinuous in its distribution. Inflammation may cause ulceration, fissures, fistulas and
fibrosis with stricturing. Histology reveals a chronic inflammatory infiltrate that is usually patchy and
transmural.

Ulcerative colitis Vs Crohns


Crohn's disease Ulcerative colitis
Distribution Mouth to anus Rectum and colon
Macroscopic Cobblestone appearance, apthoid ulceration Contact bleeding
changes
Depth of disease Transmural inflammation Superficial inflammation
Distribution Patchy Continuous
pattern
Histological Granulomas (non caseating epithelioid cell Crypt abscesses, Inflammatory cells
features aggregates with Langhans' giant cells) in the lamina propria

Extraintestinal manifestations of Crohns


Related to disease extent Unrelated to disease extent
Aphthous ulcers (10%) Sacroiliiitis (10-15%)
Erythema nodosum (5-10%) Ankylosing spondylitis (1-2%)
Pyoderma gangrenosum (0.5%) Primary sclerosing cholangitis (Rare)
Acute arthropathy (6-12%) Gallstones (up to 30%)
Ocular complications (up to 10%) Renal calculi (up to 10%)

Theme: Renal stones

A. Calcium oxalate
B. Uric acid
C. Cystine
D. Struvite
E. Calcium phosphate

Please select the most likely stone type for each of the following urinary tract stone scenarios. Each
option may be used once, more than once or not at all.

34. A 73 year old lady is undergoing chemotherapy for treatment of acute leukaemia. She develops
symptoms of renal colic. Her urine tests positive for blood. A KUB x-ray shows no evidence of
stones.

Uric acid

Chemotherapy and cell death can increase uric acid levels. In this acute setting the uric acid
stones are unlikely to be coated with calcium and will therefore be radiolucent.

35. A 16 year old boy presents with renal colic. His parents both have a similar history of the
condition. His urine tests positive for blood. A KUB style x-ray shows a relatively radiodense
stone in the region of the mid ureter.

You answered Struvite

The correct answer is Cystine

Cystine stones are associated with an inherited metabolic disorder.

36. A 43 year old lady with episodes of recurrent urinary tract sepsis presents with a staghorn
calculus of the left kidney. Her urinary pH is 7.3. A KUB x-ray shows a faint outline of the
calculus.

You answered Calcium phosphate


The correct answer is Struvite

Theme from April 2012 Exam


Chronic infection with urease producing enzymes can produce an alkaline urine with formation of
struvite stone.

Renal stones

Type of Features Percentage of all


stones calculi
Calcium Hypercalciuria is a major risk factor (various causes) 85%
oxalate Hyperoxaluria may also increase risk
Hypocitraturia increases risk because citrate forms complexes with
calcium making it more soluble
Stones are radio-opaque (though less than calcium phosphate stones)
Hyperuricosuria may cause uric acid stones to which calcium oxalate
binds
Cystine Inherited recessive disorder of transmembrane cystine transport 1%
leading to decreased absorption of cystine from intestine and renal
tubule
Multiple stones may form
Relatively radiodense because they contain sulphur
Uric acid Uric acid is a product of purine metabolism 5-10%
May precipitate when urinary pH low
May be caused by diseases with extensive tissue breakdown e.g.
malignancy
More common in children with inborn errors of metabolism
Radiolucent
Calcium May occur in renal tubular acidosis, high urinary pH increases 10%
phosphate supersaturation of urine with calcium and phosphate
Renal tubular acidosis types 1 and 3 increase risk of stone formation
(types 2 and 4 do not)
Radio-opaque stones (composition similar to bone)
Struvite Stones formed from magnesium, ammonium and phosphate 2-20%
Occur as a result of urease producing bacteria (and are thus associated
with chronic infections)
Under the alkaline conditions produced, the crystals can precipitate
Slightly radio-opaque

Effect of urinary pH on stone formation


Urine pH will show individual variation (from pH 5-7). Post prandially the pH falls as purine
metabolism will produce uric acid. Then the urine becomes more alkaline (alkaline tide). When the
stone is not available for analysis the pH of urine may help to determine which stone was present.

Stone type Urine acidity Mean urine pH


Calcium phosphate Normal- alkaline >5.5
Calcium oxalate Variable 6
Uric acid Acid 5.5
Struvate Alkaline >7.2
Cystine Normal 6.5

A pathologist is examining a histological section and identifies Hassall's corpuscles. With what are they
most commonly associated?

A. Follicular carcinoma of the thyroid

B. Medulla of the thymus

C. Medulla of the spleen

D. Medulla of the kidney

E. Fundus of the stomach

Theme from 2010 Exam


Theme from January 2013 Exam
Theme from April 2013 Exam
Hassall's corpuscles are the concentric ring of epithelial cells seen in the medulla of the thymus.

Thymus

The thymus develops from the third and fourth pharyngeal pouches. It descends to lie in the anterior
superior mediastinum. It is encapsulated and is subdivided into lobules, these consist of a cortex and a
medulla. The cortex is composed of tightly packed lymphocytes, the medulla consists largely of
epithelial cells. The medullary epithelial cells are concentrically arranged and may surround a
keratinised centre, known as Hassall's corpuscles.
The inferior parathyroid glands also develop from the third pharyngeal pouch and may also be located
with the thymus gland.
It's arterial supply is from the internal mammary artery or pericardiophrenic arteries. Venous drainage is
to the left brachiocephalic vein.

Hassall's corpuscles stained with H+E


A 64 year old man presents to the clinic with right upper quadrant discomfort. He has never attended the
hospital previously and is usually well. He has just retired from full time employment as a machinist in a
PVC factory. CT scanning shows a large irregular tumour in the right lobe of his liver. Which of the
following lesions is the most likely?

A. Liposarcoma

B. Angiosarcoma

C. Hamartoma

D. Hyatid liver disease

E. Benign angioma

Angiosarcoma of the liver is a rare tumour. However, it is linked to working with vinyl chloride, as in
this case. Although modern factories minimise the exposure to this agent, this has not always been the
case.

Occupational cancers

Occupational cancers accounted for 5.3% cancer deaths in 2005.


In men the main cancers include:
• Mesothelioma
• Bladder cancer
• Non melanoma skin cancer
• Lung cancer
• Sino nasal cancer

Occupations with high levels of occupational tumours include:

• Construction industry
• Working with coal tar and pitch
• Mining
• Metalworkers
• Working with asbestos (accounts for 98% of all mesotheliomas)
• Working in rubber industry

Shift work has been linked to breast cancer in women (Health and safety executive report RR595).

The latency between exposure and disease is typically 15 years for solid tumours and 20 for leukaemia.

Many occupational cancers are otherwise rare. For example sino nasal cancer is an uncommon tumour,
50% will be SCC. They are linked to conditions such as wood dust exposure and unlike lung cancer is
not strongly linked to cigarette smoking. Another typical occupational tumour is angiosarcoma of the
liver which is linked to working with vinyl chloride. Again in the non occupational context this is an
extremely rare sporadic tumour.

A 32 year old man is involved in a house fire and sustains extensive partial thickness burns to his torso
and thigh. Two weeks post operatively he develops oedema of both lower legs. The most likely cause of
this is:

A. Iliofemoral deep vein thrombosis

B. Venous obstruction due to scarring

C. Hypoalbuminaemia

D. Excessive administration of intravenous fluids

E. None of the above

Theme from 2009 Exam

Loss of plasma proteins is the most common cause of oedema developing in this time frame.
Burns pathology

Extensive burns

• Haemolysis due to damage of erythrocytes by heat and microangiopathy


• Loss of capillary membrane integrity causing plasma leakage into interstitial space
• Extravasation of fluids from the burn site causing hypovolaemic shock (up to 48h after injury)-
decreased blood volume and increased haematocrit
• Protein loss
• Secondary infection e.g. Staphylococcus aureus
• ARDS
• Risk of Curlings ulcer (acute peptic stress ulcers)
• Danger of full thickness circumferential burns in an extremity as these may develop
compartment syndrome

Healing

• Superficial burns: keratinocytes migrate to form a new layer over the burn site
• Full thickness burns: dermal scarring. Usually need keratinocytes from skin grafts to provide
optimal coverage.

What is the diagnostic marker for carcinoid syndrome?

A. B-HCG

B. Histamine

C. Chromogranin A

D. 5-Hydroxyindoleacetic acid

E. 5-Hydroxytryptamine

Urinary measurement of 5- HIAA is an important part of clinical follow up.

Carcinoid syndrome

• Carcinoid tumours secrete serotonin


• Originate in neuroendocrine cells mainly in the intestine (midgut-distal ileum/appendix)
• Can occur in the rectum, bronchi
• Hormonal symptoms mainly occur when disease spreads outside the bowel
Clinical features
- Onset: years
- Flushing face
- Palpitations
- Tricuspid stenosis causing dyspnoea
- Asthma
- Severe diarrhoea (secretory, persists despite fasting)

Investigation
- 5-HIAA in a 24-hour urine collection
- Scintigraphy
- CT scan

Treatment

• Octreotide
• Surgical removal

A 42 year old man from Southern India presents with chronic swelling of both lower legs, they are
brawny and indurated with marked skin tophic changes. Which of the following organisms is the most
likely origin of this disease process?

A. Loa loa

B. Wuchereria bancrofti

C. Trypanosoma cruzi

D. Trypanosoma gambiense

E. None of the above

W. Bancrofti is the commonest cause of filariasis leading to lymphatic obstruction. Infection with Loa
loa typically occurs in the African sub continent and usually results in generalised sub cutaneous
infections without lymphatic obstruction. Trypanosomal infections would not produce this clinical
picture.

Wuchereria bancrofti

• Parasitic filarial nematode


• Accounts for 90% of cases of filariasis
• Usually diagnosed by blood smears
• Usually transmitted by mosquitos
• Treatment is with diethylcarbamazine

A 45 year old lady has recently undergone a thyroidectomy for treatment of medullary thyroid cancer.
Which of the following tumour markers is used clinically to screen for recurrence?

A. Free T3

B. Thyroglobulin

C. Calcitonin

D. Free T4

E. Thyroid stimulating hormone

Theme from 2011 Exam


Calcitonin is clinically utilised to screen for medullary thyroid cancer recurrence. Thyroid function
testing does not form part of either diagnosis or follow up from a malignancy perspective. However,
routine assessment of TSH may be needed in patients on thyroxine.

Thyroid malignancy

Papillary carcinoma

• Commonest sub-type
• Accurately diagnosed on fine needle aspiration cytology
• Histologically they may demonstrate psammoma bodies (areas of calcification) and so called
'orphan Annie' nuclei
• They typically metastasise via the lymphatics and thus laterally located apparently ectopic
thyroid tissue is usually a metastasis from a well differentiated papillary carcinoma.

Follicular carcinoma

• Are less common than papillary lesions


• Like papillary tumours they may present as a discrete nodule. Although they appear to be well
encapsulated macroscopically there invasion on microscopic evaluation.
• Lymph node metastases are uncommon and these tumours tend to spread haematogenously. This
translates into a higher mortality rate.
• Follicular lesions cannot be accurately diagnosed on fine needle aspiration cytology and thus all
follicular FNA's will require at least a hemi thyroidectomy.

Anaplastic carcinoma
• Less common and tend to occur in elderly females
• Disease is usually advanced at presentation and often only palliative decompression and
radiotherapy can be offered.

Medullary carcinoma

• These are tumours of the parafollicular cells ( C Cells) and are of neural crest origin.
• The serum calcitonin may be elevated which is of use when monitoring for recurrence.
• They may be familial and occur as part of the MEN -2A disease spectrum.
• Spread may be either lymphatic or haematogenous and as these tumours are not derived
primarily from thyroid cells they are not responsive to radioiodine.

Lymphoma

• These respond well to radiotherapy


• Radical surgery is unnecessary once the disease has been diagnosed on biopsy material. Such
biopsy material is not generated by an FNA and thus a core biopsy has to be obtained (with
care!).

A 22 year old man is kicked in the head during a rugby match. He is temporarily concussed, but then
regains consciousness. Half an hour later he develops slurred speech, ataxia and loses consciousnesses.
On arrival in hospital he is intubated and ventilated. A CT Scan is performed which shows an extradural
haematoma. What is the most likely cause?

A. Basilar artery laceration

B. Middle meningeal artery laceration

C. Laceration of the sigmoid sinus

D. Laceration of the anterior cerebral artery

E. Laceration of the middle cerebral artery

Theme based on September 2011 Exam


Theme from April 2013 Exam
The most likely vessel from those in the list to cause an acute extra dural haemorrhage is the middle
meningeal artery. The anterior and middle cerebral arteries may cause acute sub dural haemorrhage.
Acute sub dural haemorrhages usually take slightly longer to evolve than acute extra dural
haemorrhages.

Middle meningeal artery


• Middle meningeal artery is typically the third branch of the first part of the maxillary artery, one
of the two terminal branches of the external carotid artery. After branching off the maxillary
artery in the infratemporal fossa, it runs through the foramen spinosum to supply the dura mater
(the outermost meninges) .
• The middle meningeal artery is the largest of the three (paired) arteries which supply the
meninges, the others being the anterior meningeal artery and the posterior meningeal artery.
• The middle meningeal artery runs beneath the pterion. It is vulnerable to injury at this point,
where the skull is thin. Rupture of the artery may give rise to an extra dural hematoma.
• In the dry cranium, the middle meningeal, which runs within the dura mater surrounding the
brain, makes a deep indention in the calvarium.
• The middle meningeal artery is intimately associated with the auriculotemporal nerve which
wraps around the artery making the two easily identifiable in the dissection of human cadavers
and also easily damaged in surgery.

Which of the following is not characteristic of a granuloma?

A. Altered macrophages

B. Fused macrophages

C. Epithelioid cells

D. Mixture of chronic inflammatory cells

E. Polymorphnuclear leucocytes, cellular debris and fibrin

These are typical components of an abscess cavity. Polymorphonuclear leucocytes may be found in a
granuloma if there is a focus of suppuration.

Chronic inflammation

Overview
Chronic inflammation may occur secondary to acute inflammation.In most cases chronic inflammation
occurs as a primary process. These may be broadly viewed as being one of three main processes:

• Persisting infection with certain organisms such as Mycobacterium tuberculosis which results in
delayed type hypersensitivity reactions and inflammation.
• Prolonged exposure to non-biodegradable substances such as silica or suture materials which
may induce an inflammatory response.
• Autoimmune conditions involving antibodies formed against host antigens.

Acute vs. Chronic inflammation


Acute inflammation Chronic inflammation
Changes to existing vascular structure and increased Angiogenesis predominates
permeability of endothelial cells
Infiltration of neutrophils Macrophages, plasma cells and
lymphocytes predominate
Process may resolve with: Healing by fibrosis is the main result

• Suppuration
• Complete resolution
• Abscess formation
• Progression to chronic inflammation
• Healing by fibrosis

Granulomatous inflammation
A granuloma consists of a microscopic aggregation of macrophages (with epithelial type arrangement
=epitheliod). Large giant cells may be found at the periphery of granulomas.

Mediators
Growth factors released by activated macrophages include agents such as interferon and fibroblast
growth factor (plus many more). Some of these such as interferons may have systemic features resulting
in systemic symptoms and signs, which may be present in individuals with long standing chronic
inflammation.

The finding of granulomas is pathognomonic of chronic inflammation, as illustrated in this biopsy from
a patient with colonic Crohns disease

A 42 year old man presents with a painless lump in the left testicle that he noticed on self examination.
Clinically there is a firm nodule in the left testicle, ultrasound appearances show an irregular mass
lesion. His serum AFP and HCG levels are both within normal limits. What is the most likely diagnosis?

A. Yolk sack tumour


B. Seminoma

C. Testicular teratoma

D. Epididymo-orchitis

E. Adenomatoid tumour
Seminomas typically have normal AFP and HCG. These are usually raised in teratomas and yolk sac
tumours

This mans age, presenting symptoms and normal tumour markers make a seminoma the most likely
diagnosis. Epididymo-orchitis does not produce irregular mass lesions which are painless.

Testicular disorders

Testicular cancer
Testicular cancer is the most common malignancy in men aged 20-30 years. Around 95% of cases of
testicular cancer are germ-cell tumours. Germ cell tumours may essentially be divided into:

Tumour type Key features Tumour markers Pathology


Seminoma • Commonest subtype • AFP usually Sheet like lobular
(50%) normal patterns of cells with
• Average age at • HCG elevated in substantial fibrous
diagnosis = 40 10% seminomas component. Fibrous
• Even advanced • Lactate septa contain
disease associated dehydrogenase; lymphocytic inclusions
with 5 year survival
elevated in 10-20% and granulomas may
of 73% seminomas (but also be seen.
in many other
conditions)
Non seminomatous germ cell • Younger age at • AFP elevated in Heterogenous texture
tumours (42%) presentation =20-30 years up to 70% of cases with occasional
• Advanced disease carries • HCG elevated in ectopic tissue such as
• Teratoma worse prognosis (48% at 5 up to 40% of cases hair
• Yolk sac tumour years) • Other markers
• Choriocarcinoma • Retroperitoneal lymph rarely helpful
• Mixed germ cell node dissection may be
tumours (10%) needed for residual disease
after chemotherapy

Image demonstrating a classical seminoma, these tumours are typically more uniform than teratomas
Image sourced from Wikipedia

Risk factors for testicular cancer

• Cryptorchidism
• Infertility
• Family history
• Klinefelter's syndrome
• Mumps orchitis

Features
• A painless lump is the most common presenting symptom
• Pain may also be present in a minority of men
• Other possible features include hydrocele, gynaecomastia

Diagnosis

• Ultrasound is first-line
• CT scanning of the chest/ abdomen and pelvis is used for staging
• Tumour markers (see above) should be measured

Management

• Orchidectomy (Inguinal approach)


• Chemotherapy and radiotherapy may be given depending on staging
• Abdominal lesions >1cm following chemotherapy may require retroperitoneal lymph node
dissection.

Prognosis is generally excellent

• 5 year survival for seminomas is around 95% if Stage I


• 5 year survival for teratomas is around 85% if Stage I

Benign disease

Epididymo-orchitis
Acute epididymitis is an acute inflammation of the epididymis, often involving the testis and usually
caused by bacterial infection.

• Infection spreads from the urethra or bladder. In men <35 years, gonorrhoea or chlamydia are the
usual infections.
• Amiodarone is a recognised non infective cause of epididymitis, which resolves on stopping the
drug.
• Tenderness is usually confined to the epididymis, which may facilitate differentiating it from
torsion where pain usually affects the entire testis.

Testicular torsion

• Twist of the spermatic cord resulting in testicular ischaemia and necrosis.


• Most common in males aged between 10 and 30 (peak incidence 13-15 years)
• Pain is usually severe and of sudden onset.
• Cremasteric reflex is lost and elevation of the testis does not ease the pain.
• Treatment is with surgical exploration. If a torted testis is identified then both testis should be
fixed as the condition of bell clapper testis is often bilateral.

A baby is born by normal vaginal delivery at 39 weeks gestation. Initially all appears well and then the
clinical staff become concerned because the baby develops recurrent episodes of cyanosis. These are
worse during feeding and improve dramatically when the baby cries. The most likely underlying
diagnosis is:

A. Choanal atresia

B. Oesophageal reflux

C. Tetralogy of Fallot

D. Oesophageal atresia

E. Congenital diaphragmatic hernia

Theme from 2011 exam


Theme from April 2013 Exam
In Choanal atresia the episodes of cyanosis are usually worst during feeding. Improvement may be seen
when the baby cries as the oropharyngeal airway is used.

Choanal atresia

• Congenital disorder with an incidence of 1 in 7000 births.


• Posterior nasal airway occluded by soft tissue or bone.
• Associated with other congenital malformations e.g. coloboma
• Babies with unilateral disease may go unnoticed.
• Babies with bilateral disease will present early in life as they are obligate nasal breathers.
• Treatment is with fenestration procedures designed to restore patency.

A 28 year old lady presents with a pigmented lesion on her calf. Excisional biopsy confirms a diagnosis
of melanoma measuring 1cm in diameter with a Breslow thickness of 0.5mm. The lesion is close <1 mm
to all resection margins. Which of the following surgical resection margins is acceptable for this lesion?

A. 5 cm

B. 1 cm

C. 0.5 cm

D. 2 cm
E. 3 cm

Malignant melanoma

The main diagnostic features (major criteria): Secondary features (minor criteria)

• Change in size • Diameter >6mm


• Change in shape • Inflammation
• Change in colour • Oozing or bleeding
• Altered sensation

Treatment

• Suspicious lesions should undergo excision biopsy. The lesion should be removed in completely
as incision biopsy can make subsequent histopathological assessment difficult.
• Once the diagnosis is confirmed the pathology report should be reviewed to determine whether
further re-exicision of margins is required (see below):

Margins of excision-Related to Breslow thickness


Lesions 0-1mm thick 1cm
Lesions 1-2mm thick 1- 2cm (Depending upon site and pathological features)
Lesions 2-4mm thick 2-3 cm (Depending upon site and pathological features)
Lesions >4 mm thick 3cm

Further treatments such as sentinel lymph node mapping, isolated limb perfusion and block dissection of
regional lymph node groups should be selectively applied.

A 20 year old man is involved in a road traffic accident. Following the incident he is unable to extend
his wrist. However, this improves over the following weeks. Which type of injury is he most likely to
have sustained?

A. Radial nerve neurotmesis

B. Radial nerve neuropraxia

C. Axillary nerve axonotmesis

D. Ulnar nerve neuropraxia


E. Ulnar nerve axonotmesis

Theme from April 2011 Exam

Transient loss of function makes neuropraxia the most likely injury. The wrist extensors are innervated
by the radial nerve making this the most likely site of injury.

Neuropraxia

• Nerve intact but electrical conduction is affected


• Myelin sheath integrity is preserved
• Full recovery
• Autonomic function preserved
• Wallerian degeneration does not occur
• A 53 year old lady has undergone a bilateral breast augmentation procedure many years
previously. The implants are tense and uncomfortable and are removed. During their removal the
surgeon encounters a dense membrane surrounding the implants, it has a coarse granular
appearance. The tissue is sent for histology and it demonstrates fibrosis with the presence of
calcification. The underlying process responsible for these changes is:

A. Hyperplasia

B. Dysplasia

C. Metastatic calcification

D. Dystrophic calcification

E. Necrosis

Breast implants often become surrounded by a pseudocapsule and this may secondarily then be
subjected to a process of dystrophic calcification.
• Pathological calcification

Dystrophic calcification Deposition of calcium deposits in tissues that have undergone,
degeneration, damage or disease in the presence of normal serum
calcium levels
Metastatic calcification Deposition of calcium deposits in tissues that are otherwise
normal in the presence of increased serum calcium levels
A 4 year old girl presents with symptoms of right sided loin pain, lethargy and haematuria. On
examination she is pyrexial and has a large mass in the right upper quadrant. The most likely underlying
diagnosis is:

A. Perinephric abscess

B. Nephroblastoma

C. Renal cortical adenoma

D. Grawitz tumour

E. Squamous cell carcinoma of the kidney

In a child of this age, with the symptoms described a nephroblastoma is the most likely diagnosis. A
perinephric abscess is most unlikely. If an abscess were to occur it would be confined to Gertotas fascia
in the first instance, and hence anterior extension would be unlikely.

Nephroblastoma

Nephroblastoma (Wilms tumours)

• Usually present in first 4 years of life


• May often present as a mass associated with haematuria (pyrexia may occur in 50%)
• Often metastasise early (usually to lung)
• Treated by nephrectomy
• Younger children have better prognosis (<1 year of age =80% overall 5 year survival)
• Theme: Thyroid neoplasms

A. Follicular carcinoma
B. Anaplastic carcinoma
C. Medullary carcinoma
D. Papillary carcinoma
E. Lymphoma
F. Hashimotos thyroiditis
G. Graves disease

For the following histological descriptions please select the most likely underlying thyroid
neoplasm. Each option may be used once, more than once or not at all.

51. A 22 year old female undergoes a thyroidectomy. The resected specimen shows a non
encapsulated tumour with papillary projections and pale empty nuclei.
Papillary carcinoma

Theme from April 2012


The presence of papillary structures together with the cytoplasmic features described is strongly
suggestive of papillary carcinoma. They are seldom encapsulated.

52. A thyroidectomy specimen from a 43 year old lady shows a mass with prominent oxyphil cells
and scanty thyroid colloid.

You answered Medullary carcinoma

The correct answer is Follicular carcinoma

Hurthle cell tumours are a variant of follicular neoplasms in which oxyphil cells predominate.
They have a poorer prognosis than conventional follicular neoplasms

53. A 32 year old lady undergoes a thyroidectomy for a mild goitre. The resected specimen shows an
intense lymphocytic infiltrate with acinar destruction and fibrosis.

Hashimotos thyroiditis

Lymphocytic infiltrates and fibrosis are typically seen in Hashimotos thyroiditis. In Lymphoma
only dense lymphatic type tissue is usually present.

Thyroid neoplasms

Lesion Common features


Follicular • Usually present as a solitary thyroid nodule
adenoma • Malignancy can only be excluded on formal histological assessment

Papillary • Usually contain a mixture of papillary and colloidal filled follicles


carcinoma • Histologically tumour has papillary projections and pale empty nuclei
• Seldom encapsulated
• Lymph node metastasis predominate
• Haematogenous metastasis rare
• Account for 60% of thyroid cancers

Follicular • May appear macroscopically encapsulated, microscopically capsular invasion is


carcinoma seen. Without this finding the lesion is a follicular adenoma.
• Vascular invasion predominates
• Multifocal disease rare
• Account for 20% of all thyroid cancers

Anaplastic • Most common in elderly females


carcinoma • Local invasion is a common feature
• Account for 10% of thyroid cancers
• Treatment is by resection where possible, palliation may be achieved through
isthmusectomy and radiotherapy. Chemotherapy is ineffective.

Medullary • Tumours of the parafollicular cells (C Cells)


carcinoma • C cells derived from neural crest and not thyroid tissue
• Serum calcitonin levels often raised
• Familial genetic disease accounts for up to 20% cases
• Both lymphatic and haematogenous metastasis are recognised, nodal disease is
associated with a very poor prognosis.

From which of the following cell types do giant cells most commonly originate?

A. Neutrophils

B. Myofibroblasts

C. Fibroblasts

D. Macrophages

E. Goblet cells

Theme from September 2011 and 2009 Exam

Although many cell types may give rise to giant cells, macrophages remain the most common.

Giant cells

• A giant cell is a mass formed by the union of several distinct types of cells
• They are most commonly comprised of macrophages
• They are different to granulomas although causative agents may overlap

Section stained using haematoxylin and eosin showing giant cell reaction to suture material
A 43 year old lady with hypertension is suspected of having a phaeochromocytoma. Which of the
following investigations is most likely to be beneficial in this situation?

A. Dexamethasone suppression test

B. Urinary 5-Hydroxyindoleacetic Acid (5-HIAA)

C. Histamine provocation test

D. Tyramine provocation test

E. Urinary vanillymandelic acid measurements

Theme from September 2011 Exam


Theme from September 2012 Exam
Urinary VMA measurements are not completely specific but constitute first line assessment. Stimulation
tests of any sort are not justified in first line assessments.

Phaeochromocytoma and adrenal lesions

Phaeochromocytoma
Neuroendocrine tumour of the chromaffin cells of the adrenal medulla. Hypertension and
hyperglycaemia are often found.

• 10% of cases are bilateral.


• 10% occur in children.
• 11% are malignant (higher when tumour is located outside the adrenal).
• 10% will not be hypertensive.

Familial cases are usually linked to the Multiple endocrine neoplasia syndromes (considered under its
own heading).

Most tumours are unilateral (often right sided) and smaller than 10cm.

Diagnosis
Urine analysis of vanillymandelic acid (VMA) is often used (false positives may occur e.g. in patients
eating vanilla ice cream!)

Blood testing for plasma metanephrine levels.

CT and MRI scanning are both used to localise the lesion.

Treatment
Patients require medical therapy first. An irreversible alpha adrenoreceptor blocker should be given,
although minority may prefer reversible blockade(1). Labetolol may be co-administered for cardiac
chronotropic control. Isolated beta blockade should not be considered as it will lead to unopposed alpha
activity.

These patients are often volume depleted and will often require moderate volumes of intra venous
normal saline perioperatively.

Once medically optimised the phaeochromocytoma should be removed. Most adrenalectomies can now
be performed using a laparoscopic approach(2). The adrenals are highly vascular structures and removal
can be complicated by catastrophic haemorrhage in the hands of the inexperienced. This is particularly
true of right sided resections where the IVC is perilously close. Should the IVC be damaged a
laparotomy will be necessary and the defect enclosed within a Satinsky style vascular clamp and the
defect closed with prolene sutures. Attempting to interfere with the IVC using any instruments other
than vascular clamps will result in vessel trauma and make a bad situation much worse.

Incidental adrenal lesions


Adrenal lesions may be identified on CT scanning performed for other reasons(3). Factors suggesting
benign disease on CT include(4):

• Size less than 3cm


• Homogeneous texture
• Lipid rich tissue
• Thin wall to lesion

All patients with incidental lesions should be managed jointly with an endocrinologist and full work up
as described above. Patients with functioning lesions or those with adverse radiological features
(Particularly size >3cm) should proceed to surgery.
A 46 year old lady presents with symptoms of diarrhoea, weight loss of 10 Kg and a skin rash of
erythematous blisters involving the abdomen and buttocks. The blisters have an irregular border and
both intact and ruptured vesicles. What is the most likely diagnosis?

A. Colonic adenocarcinoma

B. Pancreatic adenocarcinoma

C. Tropical sprue

D. Glucagonoma

E. Insulinoma

Theme from September 2011 Exam


Theme from September 2012 Exam
Glucagonoma is strongly associated with necrolytic migratory erythema.

Glucagonoma

• Rare pancreatic tumours arising from the alpha cells of the pancreas.
• Glucagon levels markedly elevated.
• Symptoms include diarrhoea, weight loss and necrolytic migratory erythema.
• A serum level of glucagon >1000pg/ml usually suggests the diagnosis, imaging with CT
scanning is also required.
• Treatment is with surgical resection. However, careful staging is required for these tumours are
usually malignant and non resectable.

A 56 year old man presents with symptoms of neuropathic facial pain and some weakness of the
muscles of facial expression on the right side. On examination he has a hard mass approximately 6cm
anterior to the right external auditory meatus. What is the most likely diagnosis?

A. Pleomorphic adenoma

B. Adenocarcinoma

C. Mucoepidermoid carcinoma

D. Adenoid cystic carcinoma

E. Lymphoma
Theme from September 2011 Exam

The patient is most likely to have a malignant lesion within the parotid. Of the malignancies listed;
adenoid cystic carcinoma has the greatest tendency to perineural invasion.

Parotid gland malignancy

• Most parotid neoplasms (80%) are benign lesions


• Most commonly present with painless mass in cheek region
• Up to 30% may present with pain, when this is associated with a discrete mass lesion in the
parotid it usually indicates perineural invasion.
• Perineural invasion is very unlikely to occur in association with benign lesions
• 80% of patients with facial nerve weakness caused by parotid malignancies will have nodal
metastasis and a 5 year survival of 25%

Types of malignancy
Mucoepidermoid 30% of all parotid malignancies
carcinoma Usually low potential for local invasiveness and metastasis (depends mainly on
grade)
Adenoid cystic Unpredictable growth patter
carcinoma Tendency for perineural spread
Nerve growth may display skip lesions resulting in incomplete excision
Distant metastasis more common (visceral rather than nodal spread)
5 year survival 35%
Mixed tumours Often a malignancy occurring in a previously benign parotid lesion
Acinic cell carcinoma Intermediate grade malignancy
May show perineural invasion
Low potential for distant metastasis
5 year survival 80%
Adenocarcinoma Develops from secretory portion of gland
Risk of regional nodal and distant metastasis
5 year survival depends upon stage at presentation, may be up to 75% with
small lesions with no nodal involvement
Lymphoma Large rubbery lesion, may occur in association with Warthins tumours
Diagnosis should be based on regional nodal biopsy rather than parotid
resection Treatment is with chemotherapy (and radiotherapy)
A 20 year old African lady undergoes an open appendicectomy. She is reviewed for an unrelated
problem 8 months later. On abdominal inspection the wound site is covered by shiny dark protuberant
scar tissue that projects beyond the limits of the skin incision. Which of the following is the most likely
underlying process?
A. Hypertrophic scar

B. Keloid scar

C. Marjolins ulcer

D. Repeated episodes of wound sepsis

E. Mycosis fungoides

Keloid scars extend beyond the limits of the incision. Mycosis fungoides is a cutaneous T cell
lymphoma.

Wound healing

Surgical wounds are either incisional or excisional and either clean, clean contaminated or dirty.
Although the stages of wound healing are broadly similar their contributions will vary according to the
wound type.

The main stages of wound healing include:

Haemostasis

• Vasospasm in adjacent vessels, platelet plug formation and generation of fibrin rich clot.

Inflammation

• Neutrophils migrate into wound (function impaired in diabetes).


• Growth factors released, including basic fibroblast growth factor and vascular endothelial growth
factor.
• Fibroblasts replicate within the adjacent matrix and migrate into wound.
• Macrophages and fibroblasts couple matrix regeneration and clot substitution.

Regeneration

• Platelet derived growth factor and transformation growth factors stimulate fibroblasts and
epithelial cells.
• Fibroblasts produce a collagen network.
• Angiogenesis occurs and wound resembles granulation tissue.

Remodeling
• Longest phase of the healing process and may last up to one year (or longer).
• During this phase fibroblasts become differentiated (myofibroblasts) and these facilitate wound
contraction.
• Collagen fibres are remodeled.
• Microvessels regress leaving a pale scar.

The above description represents an idealised scenario. A number of diseases may distort this process. It
is obvious that one of the key events is the establishing well vascularised tissue. At a local level
angiogenesis occurs, but if arterial inflow and venous return are compromised then healing may be
impaired, or simply nor occur at all. The results of vascular compromise are all too evidence in those
with peripheral vascular disease or those poorly constructed bowel anastomoses.

Conditions such as jaundice will impair fibroblast synthetic function and overall immunity with a
detrimental effect in most parts of healing.

Problems with scars:

Hypertrophic scars
Excessive amounts of collagen within a scar. Nodules may be present histologically containing
randomly arranged fibrils within and parallel fibres on the surface. The tissue itself is confined to the
extent of the wound itself and is usually the result of a full thickness dermal injury. They may go on to
develop contractures.

Image of hypertrophic scarring. Note that it remains confined to the boundaries of the original wound:

Image sourced from Wikipedia

Keloid scars
Excessive amounts of collagen within a scar. Typically a keloid scar will pass beyond the boundaries of
the original injury. They do not contain nodules and may occur following even trivial injury. They do
not regress over time and may recur following removal.

Image of a keloid scar. Note the extension beyond the boundaries of the original incision:

Image sourced from Wikipedia

Drugs which impair wound healing:

• Non steroidal anti inflammatory drugs


• Steroids
• Immunosupressive agents
• Anti neoplastic drugs

Closure
Delayed primary closure is the anatomically precise closure that is delayed for a few days but before
granulation tissue becomes macroscopically evident.

Secondary closure refers to either spontaneous closure or to surgical closure after granulation tissue has
formed.

The pathogenicity of the tubercle bacillus is due to which of the following?

A. Necrosis caused by expanding granulomas

B. Ability to multiply within fibroblasts


C. Delayed hypersensitivity reaction against bacteria

D. Effect of antibody response

E. Direct toxic effect on host cells

Mycobacteria stimulate a specific T cell response of cell mediated immunity. This is effective in
reducing the infection, the delayed hypersensitivity also damages tissues. Necrosis occurs in TB but is
usually within the granuloma.

A 45 year old women with a thyroid carcinoma undergoes a total thyroidectomy. The post operative
histology report shows a final diagnosis of medullary type thyroid cancer. Which of the tests below is
most likely to be of clinical use in screening for disease recurrence?

A. Serum CA 19-9 Levels

B. Serum thyroglobulin levels

C. Serum PTH levels

D. Serum calcitonin levels

E. Serum TSH levels

Theme from September 2012 Exam


Medullary thyroid cancers often secrete calcitonin and monitoring the serum levels of this hormone is
useful in detecting sub clinical recurrence.

Thyroid neoplasms

Lesion Common features


Follicular • Usually present as a solitary thyroid nodule
adenoma • Malignancy can only be excluded on formal histological assessment

Papillary • Usually contain a mixture of papillary and colloidal filled follicles


carcinoma • Histologically tumour has papillary projections and pale empty nuclei
• Seldom encapsulated
• Lymph node metastasis predominate
• Haematogenous metastasis rare
• Account for 60% of thyroid cancers

Follicular • May appear macroscopically encapsulated, microscopically capsular invasion is


carcinoma seen. Without this finding the lesion is a follicular adenoma.
• Vascular invasion predominates
• Multifocal disease rare
• Account for 20% of all thyroid cancers

Anaplastic • Most common in elderly females


carcinoma • Local invasion is a common feature
• Account for 10% of thyroid cancers
• Treatment is by resection where possible, palliation may be achieved through
isthmusectomy and radiotherapy. Chemotherapy is ineffective.

Medullary • Tumours of the parafollicular cells (C Cells)


carcinoma • C cells derived from neural crest and not thyroid tissue
• Serum calcitonin levels often raised
• Familial genetic disease accounts for up to 20% cases
• Both lymphatic and haematogenous metastasis are recognised, nodal disease is
associated with a very poor prognosis.

A 15 year old boy undergoes an emergency splenectomy for trauma. He makes a full recovery and is
discharged home. Eight weeks post operatively the general practitioner performs a full blood count with
a blood film. Which of the following is most likely to be present?

A. Myofibroblasts

B. Howell-Jolly bodies

C. Multinucleate giant cells

D. Reed Sternberg Cells

E. None of the above


Post splenectomy blood film
features:
Howell- Jolly bodies
Pappenheimer bodies
Target cells
Irregular contracted erythrocytes

As the filtration function is the spleen is no longer present Howell-Jolly bodies are found.

Post splenectomy blood film changes

The loss of splenic tissue results in the inability to readily remove immature or abnormal red blood cells
from the circulation. The red cell count does not alter significantly. However, cytoplasmic inclusions
may be seen e.g. Howell-Jolly bodies.
In the first few days after splenectomy target cells, siderocytes and reticulocytes will appear in the
circulation. Immediately following splenectomy a granulocytosis (mainly composed of neutrophils) is
seen, this is replaced by a lymphocytosis and monocytosis over the following weeks.
The platelet count is usually increased and this may be persistent, oral antiplatelet agents may be needed
in some patients.

Image showing Howell Jolly bodies (arrowed)

A 43 year old women is identified as being a carrier of a BRCA 1 mutation. Apart from breast cancer,
which of the following malignancies is she at greatest risk of developing?

A. Colonic cancer

B. Ovarian cancer

C. Follicular carcinoma of the thyroid

D. Pituitary adenoma

E. Phaeochromocytoma

BRCA 1 mutation patients are 55% more likely to get ovarian cancer. Those with BRCA 2 are 25%
more likely. The risk of developing other malignancies is slightly increased but not to the same extent,
and not enough to justify screening.

Genetics and surgical disease


Some of the more commonly occurring genetic conditions occurring in surgical patients are presented
here.

Li-Fraumeni Syndrome

• Autosomal dominant
• Consists of germline mutations to p53 tumour suppressor gene
• High incidence of malignancies particularly sarcomas and leukaemias
• Diagnosed when:

*Individual develops sarcoma under 45 years


*First degree relative diagnosed with any cancer below age 45 years and another family member
develops malignancy under 45 years or sarcoma at any age

BRCA 1 and 2

• Carried on chromosome 17
• Linked to developing breast cancer (60%) risk.
• Associated risk of developing ovarian cancer (55% with BRCA 1 and 25% with BRCA 2).

Lynch Syndrome

• Autosomal dominant
• Develop colonic cancer and endometrial cancer at young age
• 80% of affected individuals with get colonic and or endometrial cancer
• High risk individuals may be identified using the Amsterdam criteria

Amsterdam criteria
Three or more family members with a confirmed diagnosis of colorectal cancer, one of whom is a first
degree (parent, child, sibling) relative of the other two.
Two successive affected generations.
One or more colon cancers diagnosed under age 50 years.
Familial adenomatous polyposis (FAP) has been excluded.

Gardners syndrome

• Autosomal dominant familial colorectal polyposis


• Multiple colonic polyps
• Extra colonic diseases include: skull osteoma, thyroid cancer and epidermoid cysts
• Desmoid tumours are seen in 15%
• Mutation of APC gene located on chromosome 5
• Due to colonic polyps most patients will undergo colectomy to reduce risk of colorectal cancer
• Now considered a variant of familial adenomatous polyposis coli
• A 53 year old man is due to undergo a splenectomy as a treatment for refractory haemolytic
anaemia. The underlying pathological basis for haemolytic anaemia is thought to be a Type 2
hypersensitivity response. Which of the following mechanisms best describes this process

A. Deposition of immune complexes

B. Cell mediated immune response

C. IgE mediated response

D. Formation of autoantibodies against cell surface antigens

E. None of the above


Mnemonic for the reactions and the
mediators involved
ACID EGG-T
Type 1 Anaphylactic
Type 2 Cytotoxic
Type 3 Immune complex
Type 4 Delayed type

EGG T (mediators)

IgE
IgG
IgG
T cells

Type 2 hypersensitivity reactions (which includes haemolytic anaemia) are associated with
formation of antibody against cell surface antigens.
• Hypersensitivity reactions

The Gell and Coombs classification divides hypersensitivity reactions into 4 types

Type I Type II Type III Type IV


Description Anaphylactic Cytotoxic Immune Delayed type
complex
Mediator IgE IgG, IgM IgG, Ig A, IgM T-cells
Antigen Exogenous Cell surface Soluble Tissues
Response Minutes Hours Hours 2-3 days
time
Examples Asthma Autoimmune haemolytic Serum sickness Graft versus host
Hay fever anaemia SLE disease
Pemphigus Aspergillosis Contact dermatitis
Goodpasture's
A 25 year old man is injured in a road traffic accident. His right tibia is fractured and is managed by
fasciotomies and application of an external fixator. Over the next 48 hours his serum creatinine rises and
urine is sent for microscopy, muddy brown casts are identified. What is the most likely underlying
diagnosis?

A. Acute interstitial nephritis

B. Acute tubular necrosis

C. Glomerulonephritis

D. IgA Nephropathy

E. Thin basement membrane disease

This patient is likely to have had compartment syndrome (tibial fracture + fasciotomies) which may
produce myoglobinuria. The presence of worsening renal function, together with muddy brown casts is
strongly suggestive of acute tubular necrosis. Acute interstitial nephritis usually arises from drug toxicity
and does not usually produce urinary muddy brown casts. Thin basement membrane disease is an
autosomal dominant condition that causes persistent microscopic haematuria, but not worsening renal
function.

Acute Renal Failure

• Final pathway is tubular cell death.


• Renal medulla is a relatively hypoxic environment making it susceptible to renal tubular
hypoxia.
• Renovascular autoregulation maintains renal blood flow across a range of arterial pressures.
• Estimates of GFR are best indices of level of renal function. Useful clinical estimates can be
obtained by considering serum creatinine, age, race, gender and body size. eGFR calculations
such as the Cockcroft and Gault equation are less reliable in populations with high GFR's.
• Nephrotoxic stimuli such as aminoglycosides and radiological contrast media induce apoptosis.
Myoglobinuria and haemolysis result in necrosis. Overlap exists and proinflammatory cytokines
play and important role in potentiating ongoing damage.
• Post-operative renal failure is more likely to occur in patients who are elderly, have peripheral
vascular disease, high BMI, have COPD, receive vasopressors, are on nephrotoxic medication or
undergo emergency surgery.
• Avoiding hypotension will reduce risk of renal tubular damage.
• There is no evidence that administration of ACE inhibitors or dopamine reduces the incidence of
post-operative renal failure.
A 56 year old man has undergone a radical nephrectomy. The pathologist bisects the kidney and
identifies a pink fleshy tumour in the renal pelvis. What is the most likely disease?

A. Renal cell carcinoma

B. Transitional cell carcinoma

C. Angiomyolipoma

D. Phaeochromocytoma

E. Renal adenoma
Most renal tumours are yellow or brown in colour. TCC's are one of the few tumours to appear pink.

Theme from April 2012


The finding of a TCC in the renal pelvis mandates a nephroureterectomy.

Renal lesions

Lesion Disease specific features Treatment


Renal cell • Most present with haematuria (50%) Usually radical or partial
carcinoma • Common renal tumour (85% cases) nephrectomy
• Paraneoplastic features include
hypertension and polycythaemia
• Most commonly has haematogenous
mestastasis

Nephroblastoma • Rare childhood tumour Surgical resection combined with


• It accounts for 80% of all chemotherapy (usually vincristine,
genitourinary malignancies in those actinomycin D and doxorubicin
under the age of 15 years
• Up to 90% will have a mass
• 50% will be hypertensive
• Diagnostic work up includes
ultrasound and CT scanning

Neuroblastoma • Most common extracranial tumour of Surgical resection, radiotherapy and


childhood chemotherapy
• 80% occur in those under 4 years of
age
• Tumour of neural crest origin (up to
50% occur in the adrenal gland)
• The tumour is usually calcified and
may be diagnosed using MIBG
scanning
• Staging is with CT

Transitional cell • Accounts for 90% of upper urinary Radical nephroureterectomy


carcinoma tract tumour, but only 10% of renal
tumours
• Males affected 3x more than females
• Occupational exposure to industrial
dyes and rubber chemicals may
increase risk
• Up to 80% present with painless
haematuria
• Diagnosis and staging is with CT IVU

Angiomyolipoma • 80% of these hamartoma type lesions 50% of patients with lesions >4cm
occur sporadically, the remainder are will have symptoms and will require
seen in those with tuberous sclerosis surgical resection
• Tumour is composed of blood vessels,
smooth muscle and fat
• Massive bleeding may occur in 10%
of cases

A 65 year old lady presents with a lesion affecting her right breast. On examination she has a weeping,
crusting lesion overlying the right nipple, the areolar region is not involved. There is no palpable mass
lesion in the breast, there is a palpable axillary lymph node. The patient's general practitioner has tried
treating the lesion with 1% hydrocortisone cream, with no success. What is the most likely diagnosis?

A. Infection with Staphylococcus aureus

B. Pagets disease of the nipple

C. Phyllodes tumour

D. Nipple eczema

E. Basal cell carcinoma

A weeping, crusty lesion such as this is most likely to represent Pagets disease of the nipple (especially
since the areolar region is spared). Although no mass lesion is palpable, a proportion of patients will still
have an underlying invasive malignancy (hence the lymphadenopathy).

Pagets disease of the nipple


Pagets disease is an eczematoid change of the nipple associated with an underlying breast malignancy
and it is present in 1-2% of patients with breast cancer. In half of these patients, it is associated with an
underlying mass lesion and 90% of such patients will have an invasive carcinoma. 30% of patients
without a mass lesion will still be found to have an underlying carcinoma. The remainder will have
carcinoma in situ.
Pagets disease differs from eczema of the nipple in that it involves the nipple primarily and only latterly
spreads to the areolar (the opposite occurs in eczema).
Diagnosis is made by punch biopsy, mammography and ultrasound of the breast.
Treatment will depend on the underlying lesion.

73 year old man presents with haemoptysis and is suspected of suffering from lung cancer. On
examination he has an enlarged supraclavicular lymph node. Which of the following features is most
likely to be present on histological examination?

A. Increased mitoses

B. Apoptosis

C. Barr Bodies

D. Multinucleate giant cells

E. Granuloma

Theme from 2011 Exam

Increased mitoses are commonly seen in association with malignant transformation of cells. Apoptosis is
not a common feature of metastatic cancer. Barr Bodies are formed during X chromosome inactivation
in female somatic cells.

Histopathology of malignancy

• Abnormal tissue architecture


• Coarse chromatin
• Invasion of basement membrane*
• Abnormal mitoses
• Angiogenesis
• De-differentiation
• Areas of necrosis
• Nuclear pleomorphism
*= Those features that distinguish invasive malignancy from in situ disease

Which of the following pathological explanations best describes the initial pathological processes
occurring in an abdominal aortic aneurysm in an otherwise well 65 year old, hypertensive male?

A. Loss of elastic fibres from the adventitia

B. Loss of collagen from the adventitia

C. Loss of collagen from the media

D. Loss of elastic fibres from the media

E. Decreased matrix metalloproteinases in the adventitia

Theme from April 2012 Exam


Theme from April 2013 Exam
In established aneurysmal disease there is dilation of all layers of the arterial wall and loss of both
elastin and collagen. The primary event is loss of elastic fibres with subsequent degradation of collagen
fibres.

Pathology of abdominal aortic aneurysm

Abdominal aortic aneurysms occur primarily as a result of the failure of elastic proteins within the
extracellular matrix. Aneurysms typically represent dilation of all layers of the arterial wall. Most
aneurysms are caused by degenerative disease. After the age of 50 years the normal diameter of the
infrarenal aorta is 1.5cm in females and 1.7cm in males. Diameters of 3cm and greater, are considered
aneurysmal. The pathophysiology involved in the development of aneurysms is complex and the
primary event is loss of the intima with loss of elastic fibres from the media. This process is associated
with, and potentiated by, increased proteolytic activity and lymphocytic infiltration.

Major risk factors for the development of aneurysms include smoking and hypertension. Rare but
important causes include syphilis and connective tissues diseases such as Ehlers Danlos type 1 and
Marfans syndrome.

Layers of arterial wall


A 28 year old lady has a malignant melanoma removed from her calf. Which of the following
pathological criteria carries the greatest prognostic weighting?

A. Vascular invasion

B. Abnormal mitoses

C. Breslow thickness

D. Perineural invasion

E. Lymphocytic infiltrates

Theme from April 2012 Exam


The Breslow thickness has considerable prognostic importance. Lymphocytic infiltrates may be
associated with an improved prognosis, but do not carry nearly the same weight as increased thickness.

Malignant melanoma

The main diagnostic features (major criteria): Secondary features (minor criteria)

• Change in size • Diameter >6mm


• Change in shape • Inflammation
• Change in colour • Oozing or bleeding
• Altered sensation

Treatment

• Suspicious lesions should undergo excision biopsy. The lesion should be removed in completely
as incision biopsy can make subsequent histopathological assessment difficult.
• Once the diagnosis is confirmed the pathology report should be reviewed to determine whether
further re-exicision of margins is required (see below):

Margins of excision-Related to Breslow thickness


Lesions 0-1mm thick 1cm
Lesions 1-2mm thick 1- 2cm (Depending upon site and pathological features)
Lesions 2-4mm thick 2-3 cm (Depending upon site and pathological features)
Lesions >4 mm thick 3cm

A 34 year old lady undergoes an elective cholecystectomy for attacks of recurrent cholecystitis due to
gallstones. Microscopic assessment of the gallbladder is most likely to show which of the following?

A. Dysplasia of the fundus

B. Widespread necrosis

C. Ashoff-Rokitansky sinuses

D. Metaplasia of the fundus

E. None of the above

Aschoff-Rokitansky sinuses are the result of hyperplasia and herniation of epithelial cells through the
fibromuscular layer of the gallbladder wall. They may be macroscopic or microscopic. Ashoff-
Rokitansky sinuses may be identified in cases of chronic cholecystitis and gallstones. Although
gallstones may predispose to the development of gallbladder cancer the actual incidence of dysplasia
and metaplastic change is rare. In the elective setting described above necrosis would be rare.

Gallbladder

• Fibromuscular sac with capacity of 50ml


• Columnar epithelium

Relations of the gallbladder


Anterior Liver
Posterior • Covered by peritoneum
• Transverse colon
• 1st part of the duodenum

Laterally Right lobe of liver


Medially Quadrate lobe of liver

Arterial supply
Cystic artery (branch of Right hepatic artery)

Venous drainage
Cystic vein

Nerve supply
Sympathetic- mid thoracic spinal cord, Parasympathetic- anterior vagal trunk

Common bile duct

Origin Confluence of cystic and common hepatic ducts


Relations at origin • Medially - Hepatic artery
• Posteriorly- Portal vein

Relations distally • Duodenum - anteriorly


• Pancreas - medially and laterally
• Right renal vein - posteriorly

Arterial supply Branches of hepatic artery and retroduodenal branches of gastroduodenal artery

Calot's triangle

Medially Common hepatic duct


Inferiorly Cystic duct
Superiorly Inferior edge of liver
Contents Cystic artery

Which of the following are not true of Li-Fraumeni syndrome?

A. It consists of mutations to the p53 tumour suppressor gene

B. Is likely to be present in a teenager presenting with a liposarcoma

C. It has an autosomal dominant inheritance pattern

D. Affected individuals are unlikely to develop acute myeloid leukaemia


E. Adrenal malignancies are more common than in normal population

They are at high risk of developing leukaemia.

Genetics and surgical disease

Some of the more commonly occurring genetic conditions occurring in surgical patients are presented
here.

Li-Fraumeni Syndrome

• Autosomal dominant
• Consists of germline mutations to p53 tumour suppressor gene
• High incidence of malignancies particularly sarcomas and leukaemias
• Diagnosed when:

*Individual develops sarcoma under 45 years


*First degree relative diagnosed with any cancer below age 45 years and another family member
develops malignancy under 45 years or sarcoma at any age

BRCA 1 and 2

• Carried on chromosome 17
• Linked to developing breast cancer (60%) risk.
• Associated risk of developing ovarian cancer (55% with BRCA 1 and 25% with BRCA 2).

Lynch Syndrome

• Autosomal dominant
• Develop colonic cancer and endometrial cancer at young age
• 80% of affected individuals with get colonic and or endometrial cancer
• High risk individuals may be identified using the Amsterdam criteria

Amsterdam criteria
Three or more family members with a confirmed diagnosis of colorectal cancer, one of whom is a first
degree (parent, child, sibling) relative of the other two.
Two successive affected generations.
One or more colon cancers diagnosed under age 50 years.
Familial adenomatous polyposis (FAP) has been excluded.

Gardners syndrome
• Autosomal dominant familial colorectal polyposis
• Multiple colonic polyps
• Extra colonic diseases include: skull osteoma, thyroid cancer and epidermoid cysts
• Desmoid tumours are seen in 15%
• Mutation of APC gene located on chromosome 5
• Due to colonic polyps most patients will undergo colectomy to reduce risk of colorectal cancer
• Now considered a variant of familial adenomatous polyposis coli

A 35 year old type 1 diabetic presents with difficulty mobilising and back pain radiating to the thigh. He
has a temperature of 39 oC and has pain on extension of the hip. He is diagnosed with an iliopsoas
abscess. Which of the following statements is false in relation to his diagnosis?

A. Staphylococcus aureus is the most likely primary cause

B. Recurrence occurs in 60% cases

C. More common in males

D. Crohn's is the most likely secondary cause

E. CT guided drainage is preferable first line management

Classical features include: a limp, back pain and fever. Recurrence rates are about 15-20%.
Staphylococcus is the commonest primary cause, others include Streptococcus and E.coli. Management
is ideally by CT guided drainage.

Iliopsoas abscess

• Collection of pus in iliopsoas compartment (iliopsoas and iliacus)


• Causes:

Primary

• Haematogenous spread of bacteria


• Staphylococcus aureus: most common

Secondary

• Crohn's (commonest cause in this category)


• Diverticulitis, Colorectal cancer
• UTI, GU cancers
• Vertebral osteomyelitis
• Femoral catheter, lithotripsy
• Endocarditis

Note the mortality rate can be up to 19-20% in secondary iliopsoas abscesses compared with 2.4% in
primary abscesses.

Clinical features

• Fever
• Back/flank pain
• Limp
• Weight loss

Clinical examination

• Patient in the supine position with the knee flexed and the hip mildly externally rotated

• Specific tests to diagnose iliopsoas inflammation:

Place hand proximal to the patient's ipsilateral knee and ask patient to lift thigh against your hand. This
will cause pain due to contraction of the psoas muscle.

Lie the patient on the normal side and hyperextend the affected hip. In inflammation this should elicit
pain as the psoas muscle is stretched.

Investigation

• CT is gold standard

Management

• Antibiotics
• Percutaneous drainage
• Surgery is indicated if:

1. Failure of percutaneous drainage


2. Presence of an another intra-abdominal pathology which requires surgery

Surgical approach
The authors technique for draining these collections is given here.
Review the CT scans and plan surgical approach. An extraperitoneal approach is important.
The collection usually extends inferiorly and can be accessed from an incision at a level of L4 on the
affected side.

GA
Transverse laterally placed incision.
Incise external oblique.
Split the subsequent muscle layers.
As you approach the peritoneum use blunt dissection to pass laterally around it.
Remember the ureter and gonadal veins lie posterior at this level.
Eventually you will enter the abscess cavity, a large amount of pus is usually released at this point.
Drain the area with suction and washout with saline.
Place a corrugated drain well into the abscess cavity.
If you have made a small skin incision it is reasonable to bring the drain up through the skin wound.
Otherwise place a lateral exit site and close the skin and external oblique. If you do this ensure that you
use interrupted sutures.

Anchor the drain with strong securely tied silk sutures (it is extremely tiresome if it falls out!)

Which of the following statements relating to parathyroid neoplasms is incorrect?

A. 15% of cases are due to parathyroid carcinoma

B. 80% of cases are due to parathyroid adenomas

C. Parathyroid adenomas are often encapsulated

D. 10% of parathyroid adenomas develop in ectopically located glands

E. 85% of cases of primary hyperparathyroidism are due to solitary


adenomas

Parathyroid carcinomas account for up to 5% of tumours. Adenomas are often encapsulated. Lesions
that are fibrotic and densely adherent to the gland may be a carcinoma. 85% cases of primary
hyperparathyroidism are due to a single adenoma and this is the reason some surgeons favour a focussed
parathyroidectomy.

Parathyroid glands and disorders of calcium metabolism

Hyperparathyroidism
Disease type Hormone profile Clinical features Cause
Primary • PTH (Elevated) • May be asymptomatic Most cases due to solitary
2+
hyperparathyroidism • Ca (Elevated) if mild adenoma (80%), multifocal
• Phosphate (Low) • Recurrent abdominal disease occurs in 10-15% and
• Serum Calcium : pain (pancreatitis, renal parathyroid carcinoma in 1%
Creatinine colic) or less
clearance ratio > • Changes to emotional
0.01 or cognitive state

Secondary • PTH (Elevated) • May have few Parathyroid gland


hyperparathyroidism • Ca2+ (Low or symptoms hyperplasia occurs as a result
normal) • Eventually may of low calcium, almost
• Phosphate develop bone disease, always in a setting of chronic
(Elevated) osteitis fibrosa cystica renal failure
• Vitamin D levels and soft tissue
(Low) calcifications

Tertiary • Ca2+ (Normal or • Metastatic Occurs as a result of ongoing


hyperparathyroidism high) calcification hyperplasia of the
• PTH (Elevated) • Bone pain and / or parathyroid glands after
• Phosphate levels fracture correction of underlying
(Decreased or • Nephrolithiasis renal disorder, hyperplasia of
Normal) • Pancreatitis all 4 glands is usually the
• Vitamin D cause
(Normal or
decreased)
• Alkaline
phosphatase
(Elevated)

Differential diagnoses
It is important to consider the rare but relatively benign condition of benign familial hypocalciuric
hypercalcaemia, caused by an autosomal dominant genetic disorder. Diagnosis is usually made by
genetic testing and concordant biochemistry (Serum Calcium : Creatinine clearance ratio <0.01-
distinguished from primary hyperparathyroidism).

Treatment

Primary hyperparathyroidism
Indications for surgery

• Elevated serum Calcium > 1mg/dL above normal


• Hypercalciuria > 400mg/day
• Creatinine clearance < 30% compared with normal
• Episode of life threatening hypercalcaemia
• Nephrolithiasis
• Age < 50 years
• Neuromuscular symptoms
• Reduction in bone mineral density of the femoral neck, lumbar spine, or distal radius of more
than 2.5 standard deviations below peak bone mass (T score lower than -2.5)

Secondary hyperparathyroidism
Usually managed with medical therapy.

Indications for surgery in secondary (renal) hyperparathyroidism:

• Bone pain
• Persistent pruritus
• Soft tissue calcifications

Tertiary hyperparathyroidism
Allow 12 months to elapse following transplant as many cases will resolve
The presence of an autonomously functioning parathyroid gland may require surgery. If the culprit gland
can be identified then it should be excised. Otherwise total parathyroidectomy and re-implantation of
part of the gland may be required.

A 20 year old girl presents with a thyroid cancer, she is otherwise well with no significant family
history. On examination she has a nodule in the left lobe of the thyroid with a small discrete mass
separate from the gland itself. Which of the following is the most likely cause?

A. Follicular carcinoma

B. Anaplastic carcinoma

C. Medullary carcinoma

D. Papillary carcinoma

E. B Cell Lymphoma

Theme from September 2011 Exam

Papillary carcinoma is the most common subtype and may cause lymph node metastasis (mass separate
from the gland itself) that is rare with follicular tumours. Anaplastic carcinoma would cause more local
symptoms and would be rare in this age group.

Thyroid malignancy

Papillary carcinoma
• Commonest sub-type
• Accurately diagnosed on fine needle aspiration cytology
• Histologically they may demonstrate psammoma bodies (areas of calcification) and so called
'orphan Annie' nuclei
• They typically metastasise via the lymphatics and thus laterally located apparently ectopic
thyroid tissue is usually a metastasis from a well differentiated papillary carcinoma.

Follicular carcinoma

• Are less common than papillary lesions


• Like papillary tumours they may present as a discrete nodule. Although they appear to be well
encapsulated macroscopically there invasion on microscopic evaluation.
• Lymph node metastases are uncommon and these tumours tend to spread haematogenously. This
translates into a higher mortality rate.
• Follicular lesions cannot be accurately diagnosed on fine needle aspiration cytology and thus all
follicular FNA's will require at least a hemi thyroidectomy.

Anaplastic carcinoma

• Less common and tend to occur in elderly females


• Disease is usually advanced at presentation and often only palliative decompression and
radiotherapy can be offered.

Medullary carcinoma

• These are tumours of the parafollicular cells ( C Cells) and are of neural crest origin.
• The serum calcitonin may be elevated which is of use when monitoring for recurrence.
• They may be familial and occur as part of the MEN -2A disease spectrum.
• Spread may be either lymphatic or haematogenous and as these tumours are not derived
primarily from thyroid cells they are not responsive to radioiodine.

Lymphoma

• These respond well to radiotherapy


• Radical surgery is unnecessary once the disease has been diagnosed on biopsy material. Such
biopsy material is not generated by an FNA and thus a core biopsy has to be obtained (with
care!).

A 28 year old lady is breast feeding her first child. She presents with discomfort of the right breast.
Clinical examination demonstrates erythema and an area that is fluctuant. Aspiration and culture of the
fluid is most likely to demonstrate infection with which of the following organisms?
A. Clostridium perfringens

B. Staphylococcus aureus

C. Streptococcus pyogenes

D. Staphylococcus epidermidis

E. Actinomycosis

Theme from 2013, 2010 and 2009 Exam

Staphylococcus aureus is the commonest cause. The infants mouth is usually the source as it damages
the nipple areolar complex allowing entry of bacteria.

Breast abscess

• In lactational women Staphylococcus aureus is the most common cause


• Typical presentation is with a tender, fluctaunt mass in a lactating women
• Diagnosis and treatment is performed using USS and associated drainage of the abscess cavity.
Antibiotics should also be administered
• Where there is necrotic skin overlying the abscess, the patient should undergo surgery

An 18 year old rock climber falls onto his left arm and sustains a large haematoma of the left upper arm.
Unfortunately the wound associated with the injury is neglected and it becomes infected. Which of these
changes is least likely to occur?

A. Axillary lymphadenopathy

B. Leucopenia

C. Tenderness

D. Mild pyrexia

E. Local formation of yellow pus

Leucopenia would be unusual and should prompt a search for another cause.

Acute inflammation

Inflammation is the reaction of the tissue elements to injury. Vascular changes occur, resulting in the
generation of a protein rich exudate. So long as the injury does not totally destroy the existing tissue
architecture, the episode may resolve with restoration of original tissue architecture.

Vascular changes

• Vasodilation occurs and persists throughout the inflammatory phase.


• Inflammatory cells exit the circulation at the site of injury.
• The equilibrium that balances Starlings forces within capillary beds is disrupted and a protein
rich exudate will form as the vessel walls also become more permeable to proteins.
• The high fibrinogen content of the fluid may form a fibrin clot. This has several important
immunomodulatory functions.

Sequelae
Resolution • Typically occurs with minimal initial injury
• Stimulus removed and normal tissue architecture results

Organisation • Delayed removed of exudate


• Tissues undergo organisation and usually fibrosis

Suppuration • Typically formation of an abscess or an empyema


• Sequestration of large quantities of dead neutrophils

Progression to chronic • Coupled inflammatory and reparative activities


inflammation • Usually occurs when initial infection or suppuration has been
inadequately managed

Causes

• Microbacterial infections e.g. Viruses, exotoxins or endotoxins released by bacteria


• Chemical agents
• Physical agents e.g. Trauma
• Hypersensitivity reactions
• Tissue necrosis

Theme: Tumour markers

A. Invasive ductal carcinoma of the breast


B. Prostate cancer
C. Gastric cancer
D. Ovarian cancer
E. Colorectal cancer
F. Pancreatic adenocarcinoma
G. Seminoma testicular cancer
H. Non-seminomatous testicular cancer
I. Hepatocellular carcinoma

For each tumour marker please select the most likely underlying malignancy. Each option may be used
once, more than once or not at all.

77. Raised beta-human chorionic gonadotropin with a raised alpha-feto protein level

Non-seminomatous testicular cancer

Theme from April 2012 Exam


A raised alpha-feto protein level excludes a seminoma

78. Elevated CA 19-9

Pancreatic adenocarcinoma

79. Raised alpha-feto protein level in a 54-year-old woman

Hepatocellular carcinoma

Tumour markers

Theme from January 2013 exam


Tumour markers may be divided into:

• monoclonal antibodies against carbohydrate or glycoprotein tumour antigens


• tumour antigens
• enzymes (alkaline phosphatase, neurone specific enolase)
• hormones (e.g. calcitonin, ADH)

It should be noted that tumour markers usually have a low specificity

Monoclonal antibodies
Tumour marker Association
CA 125 Ovarian cancer
CA 19-9 Pancreatic cancer
CA 15-3 Breast cancer
NB: The breast cancer tumour marker is not specific or sensitive enough to be used routinely.

Tumour antigens
Tumour marker Association
Prostate specific antigen (PSA) Prostatic carcinoma
Alpha-feto protein (AFP) Hepatocellular carcinoma, teratoma
Carcinoembryonic antigen (CEA) Colorectal cancer

Theme: Benign breast lesions

A. Fibroadenoma
B. Breast abscess
C. Cyst of Montgomery's gland
D. Galactocele
E. Lipoma
F. Duct ectasia
G. Intraductal papilloma
H. Fat necrosis

What is the most likely diagnosis for the scenario given? Each option may be used once, more than once
or not at all.

80. A 64 year old obese female presents with a breast lump. She was hit on the breast by a cricket ball
when playing with her grandson.

Fat necrosis

An obese, post menopausal woman, with a history of trauma points towards fat necrosis. Trauma
causes inflammation of fat cells, leading to formation of a lump. Mammography will be needed to
differentiate it from breast disease.

81. A 21 year old female notices a bloody discharge from the nipple. She is otherwise well. On
examination there are no discrete lesions to feel and mammography shows dense breast tissue but
no mass lesion.

Intraductal papilloma
Intraductal papillomata are the commonest cause of blood stained nipple discharge in younger
women. There is seldom any palpable mass. An ultrasound is required and possibly a
galactogram.

82. A 18 year old female notices a non tender mobile breast lump. Clinically there is a smooth lump
which is not tethered to the skin.

Fibroadenoma

Also called a breast 'mouse' due to its mobility. It is a benign condition arising from the breast
lobule. May enlarge in pregnancy.

Benign Breast lesions

Benign Breast Diseases


Lesion Features Treatment
Fibroadenoma • Develop from a whole lobule If >3cm surgical excision is usual,
• Mobile, firm breast lumps Phyllodes tumours should be widely
• 12% of all breast masses excised (mastectomy if the lesion is
• Over a 2 year period up to 30% large)
will get smaller
• No increase in risk of
malignancy

Breast cyst • 7% of all Western females will Cysts should be aspirated, those
present with a breast cyst which are blood stained or
• Usually presents as a smooth persistently refill should be biopsied
discrete lump (may be fluctuant) or excised
• Small increased risk of breast
cancer (especially if younger)

Sclerosing adenosis, • Usually presents as a breast Lesions should be biopsied, excision


(radial scars and lump or breast pain is not mandatory
complex sclerosing • Causes mammographic changes
lesions) which may mimic carcinoma
• Cause distortion of the distal
lobular unit, without hyperplasia
(complex lesions will show
hyperplasia)
• Considered a disorder of
involution, no increase in
malignancy risk

Epithelial hyperplasia • Variable clinical presentation If no atypical features then


ranging from generalised conservative, those with atypical
lumpiness through to discrete features require either close
lump monitoring or surgical resection
• Disorder consists of increased
cellularity of terminal lobular
unit, atypical features may be
present
• Atypical features and family
history of breast cancer confers
greatly increased risk of
malignancy

Fat necrosis • Up to 40% cases usually have a Imaging and core biopsy
traumatic aetiology
• Physical features usually mimic
carcinoma
• Mass may increase in size
initially

Duct papilloma • Usually present with nipple Microdochectomy


discharge
• Large papillomas may present
with a mass
• The discharge usually originates
from a single duct
• No increase risk of malignancy

A 17 year old man is identified as having a Meckels diverticulum. From which of the following
embryological structures is it derived?

A. Foregut

B. Hindgut

C. Uranchus

D. Cloaca

E. Vitello-intestinal duct
Rule of 2's
2% of population
2 inches (5cm) long
2 feet (60 cm) from the ileocaecal
valve
2 x's more common in men
2 tissue types involved

The Meckels diverticulum is a persistence of the vitello-intestinal duct.

Meckel's diverticulum

• Congenital abnormality resulting in incomplete obliteration of the vitello-intestinal duct


• Normally, in the foetus, there is an attachment between the vitello-intestinal duct and the yolk
sac.This disappears at 6 weeks gestation.
• The tip is free in majority of cases.
• Associated with enterocystomas, umbilical sinuses, and omphaloileal fistulas.
• Arterial supply: omphalomesenteric artery.
• 2% of population, 2 inches long, 2 feet from the ileocaecal valve.
• Typically lined by ileal mucosa but ectopic gastric mucosa can occur, with the risk of peptic
ulceration. Pancreatic and jejunal mucosa can also occur.

Clinical

• Normally asymptomatic and an incidental finding.


• Complications are the result of obstruction, ectopic tissue, or inflammation.
• Removal if narrow neck or symptomatic. Options are between wedge excision or formal small
bowel resection and anastomosis.

Theme: Rectal bleeding

A. Solitary rectal ulcer syndrome


B. Haemorroidal disease
C. Fissure in ano
D. Fistula in ano
E. Anal cancer
F. Ulcerative colitis

Please select the most likely diagnosis for the scenario given. Each option may be used once, more than
once or not at all.

84. A 22 year old man presents with a 6 day history of passage of bloody diarrhoea with passage of
mucous and slime. He is passing an average of 8 to 9 bowel movements per day. On digital rectal
examination there is no discrete abnormality to feel, but there is some blood stained mucous on
the glove.

You answered Solitary rectal ulcer syndrome

The correct answer is Ulcerative colitis

The passage of bloody diarrhoea together with mucous and a short history makes this a likely first
presentation of inflammatory bowel disease.

85. A 17 year old man presents with a 2 week history of significant pain on defecation accompanied
by the presence of a small amount of blood which is noticed on toilet paper.

Fissure in ano

Young patients with painful rectal bleeding may have a fissure. Treatment is with stool softeners
and either GTN or Diltiazem initially.

86. A 24 year old women presents with a long history of obstructed defecation and chronic
constipation. She often strains to open her bowels for long periods and occasionally notices that
she has passed a small amount of blood. On examination she has an indurated area located
anteriorly approximately 3cm proximal to the anal verge.

Solitary rectal ulcer syndrome

Solitary rectal ulcers are associated with chronic constipation and straining. It will need to be
biopsied to exclude malignancy (the histological appearances are characteristic). Diagnostic work
up should include endoscopy and probably defecating proctogram and ano-rectal manometry
studies.

Rectal bleeding

Rectal bleeding is a common cause for patients to be referred to the surgical clinic. In the clinical history
it is useful to try and localise the anatomical source of the blood. Bright red blood is usually of rectal
anal canal origin, whilst dark red blood is more suggestive of a proximally sited bleeding source. Blood
which has entered the GI tract from a gastro-duodenal source will typically resemble malaena due to the
effects of the digestive enzymes on the blood itself.

In the table below we give some typical bleeding scenarios together with physical examination findings
and causation.
Cause Type of Features in history Examination findings
bleeding
Fissure in Bright red Painful bleeding that occurs post Muco-epithelial defect usually in the
ano rectal defecation in small volumes. Usually midline posteriorly (anterior fissures
bleeding antecedent features of constipation more likely to be due to underlying
disease)
Haemorroids Bright red Post defecation bleeding noted both Normal colon and rectum.
rectal on toilet paper and drips into pan. Proctoscopy may show internal
bleeding May be alteration of bowel habit and haemorrhoids. Internal haemorrhoids
history of straining. No blood mixed are usually impalpable.
with stool. No local pain.
Crohns Bright red or Bleeding that is accompanied by Perineal inspection may show
disease mixed blood other symptoms such as altered fissures or fistulae. Proctoscopy may
bowel habit, malaise, history of demonstrate indurated mucosa and
fissures (especially anterior) and possibly strictures. Skip lesions may
abscesses. be noted at colonoscopy.
Ulcerative Bright red Diarrhoea, weight loss, nocturnal Proctitis is the most marked finding.
colitis bleeding incontinence, passage of mucous PR. Peri anal disease is usually absent.
often mixed Colonoscopy will show continuous
with stool mucosal lesion.
Rectal cancer Bright red Alteration of bowel habit. Tenesmus Usually obvious mucosal
blood mixed may be present. Symptoms of abnormality. Lesion may be fixed or
volumes metastatic disease. mobile depending upon disease
extent. Surrounding mucosa often
normal, although polyps may be
present.

Image showing a fissure in ano. Typically these are located posteriorly and in the midline. Fissures at
other sites may be associated with underlying disease.
Image sourced from Wikipedia

Colonoscopic image of internal haemorroids. Note these may often be impalpable.

Image sourced from Wikipedia

Investigation

• All patients presenting with rectal bleeding require digital rectal examination and procto-
sigmoidoscopy as a minimal baseline.
• Remember that haemorrhoids are typically impalpable and to attribute bleeding to these in the
absence of accurate internal inspection is unsatisfactory.
• In young patients with no other concerning features in the history a carefully performed
sigmoidoscopy that demonstrates clear haemorrhoidal disease may be sufficient. If clear views
cannot be obtained then patients require bowel preparation with an enema and a flexible
sigmoidscopy performed.
• In those presenting with features of altered bowel habit or suspicion of inflammatory bowel
disease a colonoscopy is the best test.
• Patients with excessive pain who are suspected of having a fissure may require an examination
under general or local anaesthesia.
• In young patients with external stigmata of fissure and a compatible history it is acceptable to
treat medically and defer internal examination until the fissure is healed. If the fissure fails to
heal then internal examination becomes necessary along the lines suggested above to exclude
internal disease.

Special tests

• In patients with a malignancy of the rectum the staging investigations comprise an MRI of the
rectum to identify circumferential resection margin compromise and to identify mesorectal nodal
disease. In addition to this CT scanning of the chest abdomen and pelvis is necessary to stage for
more distant disease. Some centres will still stage the mesorectum with endo rectal ultrasound
but this is becoming far less common.

• Patients with fissure in ano who are being considered for surgical sphincterotomy and are
females who have an obstetric history should probably have ano rectal manometry testing
performed together with endo anal ultrasound. As this service is not universally available it is not
mandatory but in the absence of such information there are continence issues that may arise
following sphincterotomy.

Management

Disease Management
Fissure in ano GTN ointment 0.2% or diltiazem cream applied topically is the usual first line
treatment. Botulinum toxin for those who fail to respond. Internal sphincterotomy for
those who fail with botox, can be considered at the botox stage in males.
Haemorroids Lifestyle advice, for small internal haemorrhoids can consider injection sclerotherapy
or rubber band ligation. For external haemorrhoids consider haemorrhoidectomy.
Modern options include HALO procedure and stapled haemorrhoidectomy.
Inflammatory Medical management- although surgery may be needed for fistulating Crohns
bowel disease (setons).
Rectal cancer Anterior resection or abdomino-perineal excision of the colon and rectum. Total
mesorectal excision is now standard of care. Most resections below the peritoneal
reflection will require defunctioning ileostomy. Most patients will require
preoperative radiotherapy.

Which of the following is the most common childhood brain tumour?

A. Glioblastoma multiforme

B. Astrocytoma

C. Medulloblastoma

D. Ependymoma

E. Meningioma

Glioblastoma multiforme is rare in childhood. In contrast, astrocytoma is the commonest brain tumour in
children. Medulloblastoma is no longer the commonest CNS tumour in children (Cancer research UK)

CNS tumours

• 60% = Glioma and metastatic disease


• 20% = Meningioma
• 10% = Pituitary lesions

In paediatric practice medulloblastomas (neuroectodermal tumours) are the commonest lesion, they are
very rare in adults.
Tumours arising in right temporal and frontal lobe may reach considerable size before becoming
symptomatic. Whereas tumours in the speech and visual areas will typically produce early symptoms.

Diagnosis
MRI Scanning provides the best resolution.

Treatment
Usually surgery, even if tumour cannot be completely resected conditions such as rising ICP can be
addressed with tumour debulking and survival and quality of life prolonged.
Curative surgery can usually be undertaken with lesions such as meningiomas. Gliomas have a marked
propensity to invade normal brain and resection of these lesions is nearly always incomplete.
A keen surgical trainee is about to embark on her first hemi arthroplasty for a fractured neck of femur. In
the anaesthetic room the patient is given 1.2g intravenous co-amoxiclav. There is a possible history of
penicillin allergy but the patient is demented and the history is not checked. The patient then develops
severe respiratory compromise and haemodynamic collapse. Which pathological process accounts for
this event?

A. Binding of the drug to circulating IgG class antibodies

B. Recognition of the drug by IgE receptors on mast cells

C. Drug initiated formation of hapten-protein complexes

D. Binding of the drug to circulating IgM class antibodies

E. None of the above


Anaphylactic shock:
Antigen recognised by IgE molecules on the surface of mast cells resulting in rapid degranulation with
release of histamine and other inflammatory cytokines.

This is a case of anaphylactic shock. In anaphylaxis the mast cells degranulate.

Anaphylactic shock

• Suspect if there has been exposure to an allergen

Management

- Remove allergen
- ABCD
- Drugs:

Adrenaline 1:1000 0.5ml INTRAMUSCULARLY (not IV). Repeat after 5 mins if no response.
Then Chlorpheniramine 10mg IV
Then Hydrocortisone 100-200mg IV

Which of the following hepatobiliary disorders are most classically associated with ulcerative colitis?

A. Gallstones

B. Primary sclerosing cholangitis

C. Bile duct stones

D. Liver hamartomas
E. Hepatocellular carcinoma

Primary sclerosing cholangitis is an idiopathic inflammation of the bile ducts. It may result in episodes
of cholestasis and cholangitis and ultimately result in the need for liver transplantation. It carries a 10%
risk of malignant transformation. Crohns disease is associated with gallstones due to impaired entero-
hepatic circulation. Apart from PSC, ulcerative colitis does not increase the risk of other liver lesions.

Ulcerative colitis

Ulcerative colitis is a form of inflammatory bowel disease. Inflammation always starts at rectum, does
not spread beyond ileocaecal valve (although backwash ileitis may occur) and is continuous. The peak
incidence of ulcerative colitis is in people aged 15-25 years and in those aged 55-65 years. It is less
common in smokers.

The initial presentation is usually following insidious and intermittent symptoms. Features include:

• bloody diarrhoea
• urgency
• tenesmus
• abdominal pain, particularly in the left lower quadrant
• extra-intestinal features (see below)

Questions regarding the 'extra-intestinal' features of inflammatory bowel disease are common. Extra-
intestinal features include sclerosing cholangitis, iritis and ankylosing spondylitis.

Common to both Crohn's disease (CD) Notes


and Ulcerative colitis (UC)
Related to disease Arthritis: pauciarticular, asymmetric Arthritis is the most common extra-
activity Erythema nodosum intestinal feature in both CD and UC
Episcleritis Episcleritis is more common in Crohns
Osteoporosis disease
Unrelated to Arthritis: polyarticular, symmetric Primary sclerosing cholangitis is much
disease activity Uveitis more common in UC
Pyoderma gangrenosum Uveitis is more common in UC
Clubbing
Primary sclerosing cholangitis

Pathology

• red, raw mucosa, bleeds easily


• no inflammation beyond submucosa (unless fulminant disease)
• widespread superficial ulceration with preservation of adjacent mucosa which has the appearance
of polyps ('pseudopolyps')
• inflammatory cell infiltrate in lamina propria
• neutrophils migrate through the walls of glands to form crypt abscesses
• depletion of goblet cells and mucin from gland epithelium
• granulomas are infrequent

Barium enema

• loss of haustrations
• superficial ulceration, 'pseudopolyps'
• long standing disease: colon is narrow and short -'drainpipe colon'

Endoscopy

• Superficial inflammation of the colonic and rectal mucosa


• Continuous disease from rectum proximally
• Superifical ulceration, mucosal islands, loss of vascular definition and continuous ulceration
pattern.

Management

• Patients with long term disease are at increased risk of development of malignancy
• Acute exacerbations are generally managed with steroids, in chronic patients agents such as
azathioprine and infliximab may be used
• Individuals with medically unresponsive disease usually require surgery- in the acute phase a sub
total colectomy and end ileostomy. In the longer term a proctectomy will be required. An
ileoanal pouch is an option for selected patients
• Which of the following is not associated with thrombosis?

A. Endothelial cell damage

B. Use of tourniquets in surgery

C. Formation of platelet aggregates

D. Thrombocytopenia

E. Carcinoma of the stomach



All the other options either act directly to promote thrombosis e.g. endothelial cell damage or via
changes in consistency or flow of blood.
• Abnormal coagulation

Cause Factors affected
Heparin Prevents activation factors 2,9,10,11
Warfarin Affects synthesis of factors 2,7,9,10
DIC Factors 1,2,5,8,11
Liver disease Factors 1,2,5,7,9,10

Interpretation blood clotting test results
Disorder APTT PT Bleeding time
Haemophilia Increased Normal Normal
von Willebrand's disease Increased Normal Increased
Vitamin K deficiency Increased Increased Normal

A 16 year old boy develops a painful swelling of his distal femur. An osteoblastic sarcoma is diagnosed.
To which of the following sites is this lesion most likely to metastasise?

A. Inguinal lymph nodes

B. Common iliac lymph nodes

C. Liver

D. Brain

E. Lung
Sarcomas in which Lymphatic
Metastasis is seen:

'RACE For MS'

R: Rhabdomyosarcoma
A: Angiosarcoma
C: Clear cell sarcoma
E: Epithelial cell sarcoma

For: Fibrosarcoma

M: Malignant fibrous histiocytoma


S: Synovial cell sarcoma

Or

'SCARE'
Synovial sarcoma
Clear cell sarcoma
Angiosarcoma
Rhabdomyosarcoma
Epithelioid sarcoma

Sarcomas often metastasise via the haematogenous route and the lung is a common site for sarcoma
metastasis. The liver and brain are often spared (at least initially). A smaller number may develop
lymphatic metastasis (see above).

Sarcomas

• Malignant tumours of mesenchymal origin

Types
May be either bone or soft tissue in origin.
Bone sarcoma include:

• Osteosarcoma
• Ewings sarcoma (although non bony sites recognised)
• Chrondrosarcoma - originate from Chondrocytes

Soft tissue sarcoma are a far more heterogeneous group and include:

• Liposarcoma-adipocytes
• Rhabdomyosarcoma-striated muscle
• Leiomyosarcoma-smooth muscle
• Synovial sarcomas- close to joints (cell of origin not known but not synovium)

Malignant fibrous histiocytoma is a sarcoma that may arise in both soft tissue and bone.

Features
Certain features of a mass or swelling should raise suspicion for a sarcoma these include:

• Large >5cm soft tissue mass


• Deep tissue location or intra muscular location
• Rapid growth
• Painful lump

Assessment
Imaging of suspicious masses should utilise a combination of MRI, CT and USS. Blind biopsy should
not be performed prior to imaging and where required should be done in such a way that the biopsy tract
can be subsequently included in any resection.

Ewings sarcoma

• Commoner in males
• Incidence of 0.3 / 1, 000, 000
• Onset typically between 10 and 20 years of age
• Location by femoral diaphysis is commonest site
• Histologically it is a small round tumour
• Blood borne metastasis is common and chemotherapy is often combined with surgery

Osteosarcoma

• Mesenchymal cells with osteoblastic differentiation


• 20% of all primary bone tumours
• Incidence of 5 per 1,000,000
• Peak age 15-30, commoner in males
• Limb preserving surgery may be possible and many patients will receive chemotherapy

Liposarcoma

• Malignancy of adipocytes
• Rare approximately 2.5 per 1,000,000. They are the second most common soft tissue sarcoma
• Typically located in deep locations such as retroperitoneum
• Affect older age group usually >40 years of age
• May be well differentiated and thus slow growing although may undergo dedifferentiation and
disease progression
• Many tumours will have a pseudocapsule that can misleadingly allow surgeons to feel that they
can 'shell out' these lesions. In reality tumour may invade at the edge of the pseudocapsule and
result in local recurrence if this strategy is adopted
• Usually resistant to radiotherapy although this is often used in a palliative setting

Malignant Fibrous Histiocytoma

• Tumour with large number of histiocytes


• Most common sarcoma in adults
• Also described as undifferentiated pleomorphic sarcoma NOS (i.e. Cell of origin is not known)
• Four major subtypes are recognised: storiform-pleomorphic (70% cases), myxoid (less
aggressive), giant cell and inflammatory
• Treatment is usually with surgical resection and adjuvant radiotherapy as this reduces the
likelihood of local recurrence
nfection with which of the following micro-organisms may result in a clinical picture resembling
achalasia of the oesphagus?

A. Epstein Barr virus

B. Wuchereria Bancrofti

C. Candida Spp

D. Trypanosoma Cruzi

E. Helicobacter Pylori

Infection with Trypanosoma Cruzi may result in destruction of the ganglion cells of the myenteric
plexus, resulting in a clinical picture similar to achalasia.

Trypanosoma Cruzi

• Protozoan
• Causes Chagas disease
• Carried by bugs which infect the skin whilst feeding
• Penetrate through open wounds and mucous membranes
• Intracellular proliferation
• Major infective sites include CNS, intestinal myenteric plexus, spleen, lymph nodes and cardiac
muscle
• Chronic disease is irreversible, nifurtimox is used to treat acute infection

A 45-year-old man presents to surgical outpatients with a long history of recurrent abdominal pain and
vomiting. He is noted to have a peripheral motor neuropathy on examination. What is the most likely
diagnosis?

A. Huntington's disease

B. Myeloma

C. Acute intermittent porphyria

D. Lawrence-Moon-Biedl syndrome

E. Friedreich's ataxia

Neurological signs combined with abdominal pain is acute intermittent porphyria or lead poisoning until
proven otherwise.
Acute intermittent porphyria

Acute intermittent porphyria (AIP) is a rare autosomal dominant condition caused by a defect in
porphobilinogen deaminase, an enzyme involved in the biosynthesis of haem. The results in the toxic
accumulation of delta aminolaevulinic acid and porphobilinogen. It characteristically presents with
abdominal and neuropsychiatric symptoms in 20-40 year olds. AIP is more common in females (5:1)

Features

• abdominal: abdominal pain, vomiting


• neurological: motor neuropathy
• psychiatric: e.g. depression
• hypertension and tachycardia common

Diagnosis

• classically urine turns deep red on standing


• raised urinary porphobilinogen (elevated between attacks and to a greater extent during acute
attacks)
• assay of red cells for porphobilinogen deaminase
• raised serum levels of delta aminolaevulinic acid and porphobilinogen

A 56 year old man presents with episodic facial pain and discomfort whilst eating. He has suffered from
halitosis recently and he frequently complains of a dry mouth. He has a smooth swelling underneath his
right mandible. What is the most likely underlying diagnosis?

A. Stone impacted in Whartons duct

B. Stone impacted in Stensens duct

C. Benign adenoma of the submandibular gland

D. Adenocarcinoma of the submandibular gland

E. Squamous cell carcinoma of the submandibular gland

The symptoms are typical for sialolithiasis. The stones most commonly form in the submandibular gland
and therefore may occlude Whartons duct. Stensens duct drains the parotid gland.

Submandibular glands- disease

Physiology
The submandibular glands secrete approximately 800- 1000ml saliva per day. They typically produce
mixed seromucinous secretions. When parasympathetic activity is dominant the secretions will be more
serous. The parasympathetic fibres are derived from the chorda tympani nerves and the submandibular
ganglion, they travel to the glands via the lingual nerves.

Sialolithiasis

• 80% of all salivary gland calculi occur in the submandibular gland


• 70% of the these calculi are radio-opaque
• Stones are usually composed of calcium phosphate or calcium carbonate
• Patients typically develop colicky pain and post prandial swelling of the gland
• Investigation involves sialography to demonstrate the site of obstruction and associated other
stones
• Stones impacted in the distal aspect of Whartons duct may be removed orally, other stones and
chronic inflammation will usually require gland excision

Sialadenitis

• Usually occurs as a result of Staphylococcus aureus infection


• Pus may be seen leaking from the duct, erythema may also be noted
• Development of a sub mandibular abscess is a serious complication as it may spread through the
other deep fascial spaces and occlude the airway

Submandibular tumours

• Only 8% of salivary gland tumours affect the sub mandibular gland


• Of these 50% are malignant (usually adenoid cystic carcinoma)
• Diagnosis usually involves fine needle aspiration cytology
• Imaging is with CT and MRI
• In view of the high prevalence of malignancy, all masses of the submandibular glands should
generally be excised.

Which of the following cellular types or features is not seen in sarcoidosis?

A. Reed Sternberg Cells

B. T lymphocytes

C. Macrophages

D. Asteroid bodies

E. B lymphocytes

Reed Sternberg cells are seen in Hodgkins disease. All of the other cell types are seen in sarcoid.
Chronic inflammation

Overview
Chronic inflammation may occur secondary to acute inflammation.In most cases chronic inflammation
occurs as a primary process. These may be broadly viewed as being one of three main processes:

• Persisting infection with certain organisms such as Mycobacterium tuberculosis which results in
delayed type hypersensitivity reactions and inflammation.
• Prolonged exposure to non-biodegradable substances such as silica or suture materials which
may induce an inflammatory response.
• Autoimmune conditions involving antibodies formed against host antigens.

Acute vs. Chronic inflammation


Acute inflammation Chronic inflammation
Changes to existing vascular structure and increased Angiogenesis predominates
permeability of endothelial cells
Infiltration of neutrophils Macrophages, plasma cells and
lymphocytes predominate
Process may resolve with: Healing by fibrosis is the main result

• Suppuration
• Complete resolution
• Abscess formation
• Progression to chronic inflammation
• Healing by fibrosis

Granulomatous inflammation
A granuloma consists of a microscopic aggregation of macrophages (with epithelial type arrangement
=epitheliod). Large giant cells may be found at the periphery of granulomas.

Mediators
Growth factors released by activated macrophages include agents such as interferon and fibroblast
growth factor (plus many more). Some of these such as interferons may have systemic features resulting
in systemic symptoms and signs, which may be present in individuals with long standing chronic
inflammation.

Which of the following diseases is not considered a risk factor for gastric cancer?

A. Polya gastrectomy for antral ulcer

B. Atrophic gastritis
C. Intestinal metaplasia of columnar type at the gastric cardia

D. Patient with polyp showing medium grade dysplasia

E. Long term therapy with H2 blockers

Although some acid lowering procedures increase the risk of gastric cancer the use of H2 blockers does
not, at the present time, seem to increase the risk.

Gastric cancer

Overview
There are 700,000 new cases of gastric cancer worldwide each year. It is most common in Japan and less
common in western countries. It is more common in men and incidence rises with increasing age. The
exact cause of many sporadic cancer is not known, however, familial cases do occur in HNPCC
families. In addition, smoking and smoked or preserved foods increase the risk. Japanese migrants retain
their increased risk (decreased in subsequent generations). The distribution of the disease in western
countries is changing towards a more proximal location (perhaps due to rising obesity).

Pathology
There is some evidence of support a stepwise progression of the disease through intestinal metaplasia
progressing to atrophic gastritis and subsequent dysplasia, through to cancer. The favoured staging
system is TNM. The risk of lymph node involvement is related to size and depth of invasion; early
cancers confined to submucosa have a 20% incidence of lymph node metastasis. Tumours of the gastro-
oesophageal junction are classified as below:

Type True oesophageal cancers and may be associated with Barrett's oesophagus.
1
Type Carcinoma of the cardia, arising from cardiac type epithelium
2 or short segments with intestinal metaplasia at the oesophagogastric junction.
Type Sub cardial cancers that spread across the junction. Involve similar nodal stations to gastric
3 cancer.

Groups for close endoscopic monitoring

• Intestinal metaplasia of columnar type


• Atrophic gastritis
• Low to medium grade dysplasia
• Patients who have previously undergone resections for benign peptic ulcer disease (except highly
selective vagotomy).

Referral to endoscopy

Patients of any age with dyspepsia Patients without Worsening dyspepsia


and any of the following dyspepsia
Chronic gastrointestinal bleeding Dysphagia Barretts oesophagus
Dysphagia Unexplained abdominal Intestinal metaplasia
pain or weight loss
Weight loss Vomiting Dysplasia
Iron deficiency anaemia Upper abdominal mass Atrophic gastritis
Upper abdominal mass Jaundice Patient aged over 55 years with
unexplained or persistent dyspepsia

Upper GI endoscopy performed for dyspepsia. The addition of dye spraying (as shown in the bottom
right) may facilitate identification of smaller tumours

Image sourced from Wikipedia

Staging

• CT scanning of the chest abdomen and pelvis is the routine first line staging investigation in
most centres.
• Laparoscopy to identify occult peritoneal disease
• PET CT (particularly for junctional tumours)

Treatment
• Proximally sited disease greater than 5-10cm from the OG junction may be treated by sub total
gastrectomy
• Total gastrectomy if tumour is <5cm from OG junction
• For type 2 junctional tumours (extending into oesophagus) oesophagogastrectomy is usual
• Endoscopic sub mucosal resection may play a role in early gastric cancer confined to the mucosa
and perhaps the sub mucosa (this is debated)
• Lymphadenectomy should be performed. A D2 lymphadenectomy is widely advocated by the
Japanese, the survival advantages of extended lymphadenectomy have been debated. However,
the overall recommendation is that a D2 nodal dissection be undertaken.
• Most patients will receive chemotherapy either pre or post operatively.

Prognosis

UK Data

Disease extent Percentage 5 year survival


All RO resections 54%
Early gastric cancer 91%
Stage 1 87%
Stage 2 65%
Stage 3 18%

Operative procedure

Total Gastrectomy , lymphadenectomy and Roux en Y anastomosis

General anaesthesia
Prophylactic intravenous antibiotics
Incision: Rooftop.
Perform a thorough laparotomy to identify any occult disease.
Mobilise the left lobe of the liver off the diaphragm and place a large pack over it. Insert a large self
retaining retractor e.g. omnitract or Balfour (take time with this, the set up should be perfect). Pack the
small bowel away.
Begin by mobilising the omentum off the transverse colon.
Proceed to detach the short gastric vessels.
Mobilise the pylorus and divide it at least 2cm distally using a linear cutter stapling device.
Continue the dissection into the lesser sac taking the lesser omentum and left gastric artery flush at its
origin.
The lymph nodes should be removed en bloc with the specimen where possible.
Place 2 stay sutures either side of the distal oesophagus. Ask the anaesthetist to pull back on the
nasogastric tube. Divide the distal oesophagus and remove the stomach.
The oesphago jejunal anastomosis should be constructed. Identify the DJ flexure and bring a loop of
jejunum up to the oesophagus (to check it will reach). Divide the jejunum at this point. Bring the divided
jejunum either retrocolic or antecolic to the oesophagus. Anastamose the oesophagus to the jejunum,
using either interrupted 3/0 vicryl or a stapling device. Then create the remainder of the Roux en Y
reconstruction distally.
Place a jejunostomy feeding tube.
Wash out the abdomen and insert drains (usually the anastomosis and duodenal stump). Help the
anaesthetist insert the nasogastric tube (carefully!)
Close the abdomen and skin.
Enteral feeding may commence on the first post-operative day. However, most surgeons will leave
patients on free NG drainage for several days and keep them nil by mouth.

A 56 year old man is diagnosed as having a glioma. From which of the following cell types do these
tumours usually originate?

A. Astrocytes

B. Oligodendrocytes

C. Ependymal cells

D. Squamous cells

E. Neuroglial cells

Theme from January 2012 Exam


Gliomas originate from glial (otherwise known as neuroglial) cells. These serve a structural function in
the CNS. The tumours produced may resemble a number of CNS cell types. Tumours are therefore
named according to the cells they resemble rather than the origin. Where this is not possible they are
termed gliomas.

Glioma

Glioma is a tumour that is typically found in the CNS. These tumours arise from glial cells. They are sub
categorised according to the cell type they most closely resemble.

Glioma sub types

• Ependymomas- Ependymal cells


• Astocytomas- Astrocytes (including glioblastoma)
• Oligodendrogliomas- Oligodendrocytes
• Mixed- e.g. oligoastrocytomas

Gliomas are categorised as being either high or low grade lesions (the former has the worse prognosis).
They may be either supra or infra tentorial. Their symptoms will typically reflect their site of origin.
Glioblastoma multiforme has the worst prognosis and few patients will survive beyond 12 months.

A 78 year old man presents with unilateral deafness which has been present for the past 3 months. On
examination Webers test localises to the contralateral side and a CT scan of his head shows a thickened
calvarium with areas of sclerosis and radiolucency. His blood tests show an elevated alkaline
phosphatase, normal serum calcium and normal PTH levels. Which of the following is the most likely
underlying diagnosis?

A. Multiple myeloma with skull involvement

B. Osteoporosis

C. Pagets disease with skull involvement

D. Lung cancer with skull metastasis

E. Osteopetrosis with skull involvement

Of the conditions listed Pagets disease is the most likely diagnosis (skull vault expansion and
sensorineural hearing loss). Multiple myeloma would typically result in multiple areas of radiolucency
and usually raised calcium in this setting. Osteopetrosis is a recognised cause of the features described.
However, it is a rare inherited disorder and usually presents in children in young adults. Presentation at
this stage with no prior symptoms would be extremely rare and therefore this is not the most likely
diagnosis.

Pagets disease

Paget's disease is a disease of increased but uncontrolled bone turnover and is characterised by
architecturally abnormal bones. It is thought to be primarily a disorder of osteoclasts, with excessive
osteoclastic resorption followed by increased osteoblastic activity causing areas of sclerosis and
deformity. Paget's disease is common (UK prevalence 5%) but symptomatic in only 1 in 20 patients

Predisposing factors

• increasing age
• male sex
• northern latitude
• family history

Clinical features

• bone pain (e.g. pelvis, lumbar spine, femur)


• classical, untreated features: bowing of tibia, bossing of skull
• raised alkaline phosphatase (ALP) - calcium* and phosphate are typically normal
• skull x-ray: thickened vault, osteoporosis circumscripta

Indications for treatment include bone pain, skull or long bone deformity, fracture, periarticular Paget's

• bisphosphonate (either oral risedronate or IV zoledronate)


• calcitonin is less commonly used now

Complications

• deafness (cranial nerve entrapment)


• bone sarcoma (1% if affected for > 10 years)
• fractures
• skull thickening
• high-output cardiac failure

*usually normal in this condition but hypercalcaemia may occur with prolonged immobilisation

Theme: Genetic causes of cancer

A. Multiple endocrine neoplasia type I


B. Multiple endocrine neoplasia type II
C. Gardner's syndrome
D. Lynch Syndrome
E. Kartagener's syndrome
F. Von Recklinghausen's disease

Please select the most likely condition for the disease process described. Each option may be used once,
more than once or not at all

99. A 5 year old boy presents with recurrent episodes of sinusitis. The casualty staff are surprised to
find his liver lying in the left upper quadrant of the abdomen

Kartagener's syndrome

This is a case of Kartagener's syndrome. The primary problem is of immotile cilia syndrome.
When associated with situs inversus Kartagener's syndrome is diagnosed.
100. A 22 year old man presents with carcinoma of the caecum. His brother died from colorectal
cancer aged 25 and his father died from the disease aged 30.

You answered Gardner's syndrome

The correct answer is Lynch Syndrome

This is a case of Lynch syndrome HNPCC. It is transmitted in an autosomal dominant fashion.

101. A tall 32 year old lady presents with a diffuse neck swelling a carcinoma of the thyroid
medullary type is diagnosed.

You answered Multiple endocrine neoplasia type I

The correct answer is Multiple endocrine neoplasia type II

This is a case MEN type IIb. It is associated with phaeochromocytomas and is transmitted in an
autosomal dominant pattern if inherited. All MEN II tend to have medullary carcinoma of the
thyroid as a presenting feature

Genetics and surgical disease

Some of the more commonly occurring genetic conditions occurring in surgical patients are presented
here.

Li-Fraumeni Syndrome

• Autosomal dominant
• Consists of germline mutations to p53 tumour suppressor gene
• High incidence of malignancies particularly sarcomas and leukaemias
• Diagnosed when:

*Individual develops sarcoma under 45 years


*First degree relative diagnosed with any cancer below age 45 years and another family member
develops malignancy under 45 years or sarcoma at any age

BRCA 1 and 2

• Carried on chromosome 17
• Linked to developing breast cancer (60%) risk.
• Associated risk of developing ovarian cancer (55% with BRCA 1 and 25% with BRCA 2).

Lynch Syndrome

• Autosomal dominant
• Develop colonic cancer and endometrial cancer at young age
• 80% of affected individuals with get colonic and or endometrial cancer
• High risk individuals may be identified using the Amsterdam criteria

Amsterdam criteria
Three or more family members with a confirmed diagnosis of colorectal cancer, one of whom is a first
degree (parent, child, sibling) relative of the other two.
Two successive affected generations.
One or more colon cancers diagnosed under age 50 years.
Familial adenomatous polyposis (FAP) has been excluded.

Gardners syndrome

• Autosomal dominant familial colorectal polyposis


• Multiple colonic polyps
• Extra colonic diseases include: skull osteoma, thyroid cancer and epidermoid cysts
• Desmoid tumours are seen in 15%
• Mutation of APC gene located on chromosome 5
• Due to colonic polyps most patients will undergo colectomy to reduce risk of colorectal cancer
• Now considered a variant of familial adenomatous polyposis coli

A 45 year old man presents with symptoms of urinary colic. In the history he has suffered from recurrent
episodes of frank haematuria over the past week or so. On examination he has a left loin mass and a
varicocele. The most likely diagnosis is:

A. Renal adenocarcinoma

B. Renal cortical adenoma

C. Squamous cell carcinoma of the renal pelvis

D. Retroperitoneal fibrosis

E. Nephroblastoma

• Renal adenocarcinoma are the most common renal malignancy and account for 75% cases.
• Patients may develop frank haematuria and have episodes of clot colic.
• A Grawitz tumour is an eponymous name for Renal Adenocarcinoma.
• May metastasise to bone.

Renal tumours

Renal cell carcinoma


Renal cell carcinoma is an adenocarcinoma of the renal cortex and is believed to arise from the proximal
convoluted tubule. They are usually solid lesions, up to 20% may be multifocal, 20% may be calcified
and 20% may have either a cystic component or be wholly cystic. They are often circumscribed by a
pseudocapsule of compressed normal renal tissue. Spread may occur either by direct extension into the
adrenal gland, renal vein or surrounding fascia. More distant disease usually occurs via the
haematogenous route to lung, bone or brain.
Renal cell carcinoma comprise up to 85% of all renal malignancies. Males are more commonly affected
than females and sporadic tumours typically affect patients in their sixth decade.
Patients may present with a variety of symptoms including; haematuria (50%), loin pain (40%), mass
(30%) and up to 25% may have symptoms of metastasis.Less than 10% have the classic triad of
haematuria, pain and mass.

Investigation
Many cases will present as haematuria and be discovered during diagnostic work up. Benign renal
tumours are rare, so renal masses should be investigated with multislice CT scanning. Some units will
add and arterial and venous phase to the scan to demonstrate vascularity and evidence of caval ingrowth.

CT scanning of the chest and abdomen to detect distant disease should also be undertaken.

Routine bone scanning is not indicated in the absence of symptoms.

Biopsy should not be performed when a nephrectomy is planned but is mandatory before any ablative
therapies are undertaken.

Assessment of the functioning of the contra lateral kidney.

Management
T1 lesions may be managed by partial nephrectomy and this gives equivalent oncological results to total
radical nephrectomy. Partial nephrectomy may also be performed when there is inadequate reserve in
the remaining kidney.

For T2 lesions and above a radical nephrectomy is standard practice and this may be performed via a
laparoscopic or open approach. Preoperative embolisation is not indicated nor is resection of uninvolved
adrenal glands. During surgery early venous control is mandatory to avoid shedding of tumour cells into
the circulation.

Patients with completely resected disease do not benefit from adjuvant therapy with either
chemotherapy or biological agents. These should not be administered outside the setting of clinical
trials.
Patients with transitional cell cancer will require a nephroureterectomy with disconnection of the ureter
at the bladder.

A 63 year old man finds that he has to stop walking after 100 yards due to bilateral calf pain. He finds
that bending forwards and walking up hill helps. He is able to ride a bike without any pain. What is the
most likely underlying cause?

A. Lumbar canal stenosis

B. Diabetic neuropathy

C. Aorto-iliac occlusion

D. Occlusion of the superficial femoral artery

E. Pelvic rheumatoid arthritis

Theme from April 2012 Exam


Theme from April 2013 Exam
The positional nature of the pain and the fact that improves with walking uphill makes an underlying
vascular aetiology far less likely.

Lumbar spinal stenosis

Lumbar spinal stenosis is a condition in which the central canal is narrowed by tumour, disk prolapse or
other similar degenerative changes.
Patients may present with a combination of back pain, neuropathic pain and symptoms mimicking
claudication. One of the main features that may help to differentiate it from true claudication in the
history is the positional element to the pain. Sitting is better than standing and patients may find it easier
to walk uphill rather than downhill. The neurogenic claudication type history makes lumbar spinal
stenosis a likely underlying diagnosis, the absence of such symptoms makes it far less likely.

Pathology
Degenerative disease is the commonest underlying cause. Degeneration is believed to begin in the
intervertebral disk where biochemical changes such as cell death and loss of proteoglycan and water
content lead to progressive disk bulging and collapse. This process leads to an increased stress transfer
to the posterior facet joints, which accelerates cartilaginous degeneration, hypertrophy, and osteophyte
formation; this is associated with thickening and distortion of the ligamentum flavum. The combination
of the ventral disk bulging, osteophyte formation at the dorsal facet, and ligamentum flavum
hyptertrophy combine to circumferentially narrow the spinal canal and the space available for the neural
elements. The compression of the nerve roots of the cauda equina leads to the characteristic clinical
signs and symptoms of lumbar spinal stenosis.
Diagnosis
MRI scanning is the best modality for demonstrating the canal narrowing. Historically a bicycle test was
used as true vascular claudicants could not complete the test.

Treatment
Laminectomy

A 73 year old lady is admitted for a laparoscopic cholecystectomy. During her pre-operative assessment
it is noted that she is receiving furosemide for the treatment of hypertension. Approximately what
proportion of the sodium that is filtered at the glomerulus will be subsequently excreted?

A. Up to 25%

B. Upt to 75%

C. Between 3 and 5%

D. <2%

E. Between 1 and 2%

Theme from 2010 Exam


The loop diuretics can lead to marked increases in the amount of sodium excreted. They act in the
medullary and cortical aspects of the thick ascending limb of the loop of Henle. This results in a
decreased medullary osmolal gradient and increases free water excretion (as well as loss of sodium).
Because loop diuretics result in the loss of both sodium and water they are less frequently associated
with hyponatraemia than thiazide diuretics (these latter agents act in the cortex and do not affect urine
concentrating ability).

Diuretic agents

The diuretic drugs are divided into three major classes, which are distinguished according to the site at
which they impair sodium reabsorption: loop diuretics in the thick ascending loop of Henle, thiazide
type diuretics in the distal tubule and connecting segment; and potassium sparing diuretics in the
aldosterone - sensitive principal cells in the cortical collecting tubule.
In the kidney, sodium is reabsorbed through Na+/ K+ ATPase pumps located on the basolateral
membrane. These pumps return reabsorbed sodium to the circulation and maintain low intracellular
sodium levels. This latter effect ensures a constant concentration gradient.

Physiological effects of commonly used diuretics


Site of action Diuretic Carrier or channel Percentage of filtered sodium
inhibited excreted
Ascending limb of loop of Frusemide Na+/K+ 2Cl - carrier Upt to 25%
Henle
Distal tubule and connecting Thiazides Na+Cl- carrier Between 3 and 5%
segment
Cortical collecting tubule Spironolactone Na+ channel Between 1 and 2%

A 59 year old man presents with recurrent episodes of urinary sepsis. In his history he mentions that he
has suffered from recurrent attacks of left iliac fossa pain over the past few months. He has also notices
bubbles in his urine. He undergoes a CT scan which shows a large inflammatory mass in the left iliac
fossa. No other abnormality is detected. The most likely diagnosis is:

A. Ulcerative colitis

B. Crohns disease

C. Mesenteric ischaemia

D. Diverticular disease

E. Rectal cancer

Theme from 2009 and 2011 Exam

Recurrent attacks of diverticulitis may cause the development of local abscesses which may erode into
the bladder resulting in urinary sepsis and pneumaturia. This would be an unusual presentation from
Crohns disease and rectal cancer would be more distally sited and generally evidence of extra colonic
disease would be present if the case were malignant and this advanced.

Diverticular disease

Diverticular disease is a common surgical problem. It consists of herniation of colonic mucosa through
the muscular wall of the colon. The usual site is between the taenia coli which vessels pierce the muscle
to supply the mucosa.

Symptoms

• Altered bowel habit


• Bleeding
• Abdominal pain

Complications

• Diverticulitis
• Haemorrhage
• Development of fistula
• Perforation and faecal peritonitis
• Perforation and development of abscess
• Development of diverticular phlegmon

Diagnosis
Patients presenting in clinic will typically undergo either a colonoscopy or barium enema as part of their
diagnostic work up. Both tests will identify diverticular disease. It can be far more difficult to
confidently exclude cancer, particularly in diverticular strictures.

Acutely unwell surgical patients should be investigated in a systematic way. Plain abdominal films and
an erect chest x-ray will identify perforation. An abdominal CT scan with oral and intravenous contrast
will help to identify whether acute inflammation is present but also the presence of local complications
such as abscess formation.

Severity Classification- Hinchey


I Para-colonic abscess
II Pelvic abscess
III Purulent peritonitis
IV Faecal peritonitis

Treatment

• Increase dietary fibre intake.


• Mild attacks of diverticulitis may be managed conservatively with antibiotics.
• Peri colonic abscesses should be drained either surgically or radiologically.
• Recurrent episodes of acute diverticulitis requiring hospitalisation are a relative indication for a
segmental resection.
• Hinchey IV perforations (generalised faecal peritonitis) will require a resection and usually a
stoma. This group have a very high risk of post operative complications and usually require
HDU admission. Less severe perforations may be managed by laparoscopic washout and drain
insertion.

A 78 year old man is referred to the clinic by his general practitioner. For many years he noticed a
smooth swelling approximately 5cm anterior to the tragus of his right ear. Apart from being a heavy
smoker he has no co-morbidities. What is the most likely diagnosis?

A. Pleomorphic adenoma

B. Liposarcoma

C. Warthins tumour

D. Adenocarcinoma
E. None of the above

Warthins tumours are most common in elderly smokers. They have a relatively benign and indolent
course. They are usually well circumscribed as illustrated below:

Image sourced from Wikipedia

Parotid gland clinical

Causes of bilateral parotid enlargement

• Mumps: Associated with meningoencephalitis, pancreatitis, orchitis, or deafness


• Parotitis
• Sialectasis - especially if related to eating
• Sjogren's syndrome: dry eyes or mouth, connective tissue disease
• Sarcoidosis
• Tuberculosis
• Alcoholism
• Myxoedema
• Cushing's disease
• Diabetes/insulin resistance
• Liver cirrhosis
• Gout
• Bulimia nervosa
• Drugs
• Severe dehydration
• Malnutrition

Causes of unilateral parotid enlargement

• Salivary calculus
• Tumour

Parotid gland tumours

• Pleomorphic adenomas are the most common.


• Incisional biopsy of parotid masses is not recommended, so superficial parotidectomy is the
usual procedure of choice.
• Signs of facial nerve palsy and a parotid mass should raise suspicion of malignancy.
• Warthins tumours are relatively benign lesions that are slow growing and occur most commonly
in elderly male smokers.
• Adenoid cystic carcinoma have a tendency for perineural invasion.

Theme: Lung cancer

A. Adenocarcinoma
B. Small cell lung cancer
C. Large cell lung cancer
D. Squamous cell carcinoma

Please select the most likely lung cancer variant for the scenario described. Each option may be used
once, more than once or not at all.

107. A 73 year old heavy smoker presents with haemoptysis. On examination he is cachectic and
shows evidence of clubbing. Imaging shows a main bronchial tumour with massive mediastinal
lymphadenopathy together with widespread visceral metastases.

Small cell lung cancer

Theme from April 2012


Small cell carcinoma is associated with disseminated disease at presentation in the majority of
cases. Most cases occur in the main airways and paraneoplastic features are common.

108. A 68 year old female who has never smoked presents with a mass at the periphery of her right
lung.

Adenocarcinoma

Adenocarcinomas are the most common tumour type present in never smokers. They are usually
located at the periphery.

109. An 85 year old man presents with a cough and haemoptysis. He has a modest smoking history of
15 pack years. He is found to have a tumour located in the right main bronchus, with no
evidence of metastatic disease. He decides no undergo any treatment and he remains well for a
further 12 months before developing symptomatic metastasis.

Squamous cell carcinoma

Squamous cell carcinomas are reported to be more slow growing and are typically centrally
located. Small cell carcinomas are usually centrally located. However, small cell carcinomas
would seldom be associated with a survival of a year without treatment.

Lung cancer

Lung cancers may be classified according to histological subtypes. The main distinction is between
small cell and non small cell lung cancer. Non small cell lung cancer is the most common variant and
accounts for 80% of all lung cancers.

Non small cell lung cancer


These share common features of prognosis and management. They comprise the following tumours:

• Squamous cell carcinoma (25% cases)


• Adenocarcinoma (40% cases)
• Large cell carcinoma (10% cases)

Paraneoplastic features and early disease dissemination are less likely than with small cell lung
carcinoma. Adenocarcinoma is the most common lung cancer type encountered in never smokers.

Small cell lung carcinoma


Small cell lung carcinomas are comprised of cells with a neuro endocrine differentiation. The
neuroendocrine hormones may be released from these cells with a wide range of paraneoplastic
associations. These tumours are strongly associated with smoking and will typically arise in the larger
airways. They disseminate early in the course of the disease and although they are usually
chemosensitive this seldom results in long lasting remissions.
Which of the following cells is not found on a blood film post splenectomy?

A. Pappenheimer bodies

B. Stipple cells

C. Erythrocyte containing siderotic granules

D. Howell-Jolly bodies

E. Target cells

Stipple cells are found in lead poisoning/haemoglobinopathies.

Blood film in hyposplenism:

Howell-Jolly bodies
Pappenheimer bodies
Poikilocytes (Target cells)
Erythrocyte containing siderotic granules
Heinz bodies

Post splenectomy blood film changes

The loss of splenic tissue results in the inability to readily remove immature or abnormal red blood cells
from the circulation. The red cell count does not alter significantly. However, cytoplasmic inclusions
may be seen e.g. Howell-Jolly bodies.
In the first few days after splenectomy target cells, siderocytes and reticulocytes will appear in the
circulation. Immediately following splenectomy a granulocytosis (mainly composed of neutrophils) is
seen, this is replaced by a lymphocytosis and monocytosis over the following weeks.
The platelet count is usually increased and this may be persistent, oral antiplatelet agents may be needed
in some patients.

Image showing Howell Jolly bodies (arrowed)


A 45 year old man with long standing ulcerative colitis and rectal dysplasia presents with a DALM
lesion in the rectum. What is the most appropriate management option?

A. Snare polypectomy

B. Repeat endoscopy in 2 years

C. Discharge

D. Anterior resection

E. Panproctocolectomy

DALM lesions complicating ulcerative colitis should be managed with panproctocolectomy. An anterior
resection is inadequate since it will only remove the rectum and ulcerative colitis affects the entire
colon. Since many will be associated with invasion a snare polypectomy is not sufficient either.

Colonic lesions - DALM

• The term DALM lesion refers to a Dysplasia Associated Lesion or Mass.


• They may complicate dysplasia occurring in patients with longstanding ulcerative colitis.
• They have a high incidence of invasive foci.
• When they complicate longstanding ulcerative colitis, they should be treated by
panproctocolectomy.
• Which of the metastatic bone tumours described below is at the greatest risk of pathological
fracture ?
A. Proximal humeral lesion from a prostate cancer

B. Vertebral body lesions from a prostate cancer

C. Peritrochanteric lesion from a carcinoma of the breast

D. Proximal humeral lesion from a carcinoma of the breast

E. Peritrochanteric lesion from a prostate cancer



Peritrochanteric lesions have the greatest risks of fracture (due to loading). The lesions from
breast cancer are usually lytic and therefore at higher risk rather than the sclerotic lesions from
prostate cancer.
• Metastatic bone disease- risk of fracture

Metastatic bone tumours may be described as blastic, lytic or mixed. Osteoblastic metastatic
disease has the lowest risk of spontaneous fracture when compared to osteolytic lesions of a
similar size.
Lesions affecting the peritrochanteric region are most prone to spontaneous fracture (because of
loading forces at that site).
The factors are incorporated into the Mirel Scoring system to stratify the risk of spontaneous
fracture for bone metastasis of varying types.

Mirel Scoring system

Score Site Radiographic Width of bone Pain


points appearance involved
1 Upper extremity Blastic Less than 1/3 Mild
2 Lower Mixed 1/3 to 2/3 Moderate
extremity
3 Peritrochanteric Lytic More than 2/3 Aggravated by
function

Depending upon the score the treatment should be as follows:

Score Risk of fracture Treatment


9 or greater Impending (33%) Prophylactic fixation
8 Borderline Consider fixation
7 or less Not impending (4%) Non operative management

A 63 year old male presents with several episodes of haematuria. He suffers from COPD secondary to
long term smoking. Which is the most likely underlying cause?
A. Renal cortical adenoma

B. Renal adenocarcinoma

C. Nephroblastoma

D. Transitional cell carcinoma of the bladder

E. Adenocarcinoma of the bladder

Theme from 2009 Exam


TCC is the most common subtype and is strongly linked to smoking. The important point to note in this
question is the term most likely as renal adenocarcinoma may produce similar symptoms but is less
likely.

Bladder cancer

Bladder cancer is the second most common urological cancer. It most commonly affects males aged
between 50 and 80 years of age. Those who are current, or previous (within 20 years), smokers have a 2-
5 fold increased risk of the disease. Occupational exposure to hydrocarbons such as alanine increases the
risk. Although rare in the UK, chronic bladder inflammation arising from Schistosomiasis infection
remains a common cause of squamous cell carcinomas, in those countries where the disease is endemic.

Benign tumours
Benign tumours of the bladder including inverted urothelial papilloma and nephrogenic adenoma are
uncommon.

Bladder malignancies

• Transitional cell carcinoma (>90% of cases)


• Squamous cell carcinoma ( 1-7% -except in regions affected by schistosomiasis)
• Adenocarcinoma (2%)

Transitional cell carcinomas may arise as solitary lesions, or may be multifocal, owing to the effect of
"field change" within the urothelium. Up to 70% of TCC's will have a papillary growth pattern. These
tumours are usually superficial in location and accordingly have a better prognosis. The remaining
tumours show either mixed papillary and solid growth or pure solid growths. These tumours are
typically more prone to local invasion and may be of higher grade, the prognosis is therefore worse.
Those with T3 disease or worse have a 30% (or higher) risk of regional or distant lymph node
metastasis.

TNM Staging
Stage Description
T0 No evidence of tumour
Ta Non invasive papillary carcinoma
T1 Tumour invades sub epithelial connective tissue
T2a Inner half of detrusor invaded
T2b Outer half of detrusor invaded
T3 Tumour extends to perivesical fat
T4a Invasion of uterus, prostate or bowel
T4b Invasion of other abdominal organs
N0 No nodal disease
N1 Single lymph node metastasis (up to 2cm)
N2 Single node >2cm or multiple nodes up to 5cm
N3 Nodes over 5cm
M1 Distant disease

Presentation
Most patients (85%) will present with painless, macroscopic haematuria. In those patients with
incidental microscopic haematuria, up to 10% of females aged over 50 will be found to have a
malignancy (once infection excluded).

Staging
Most will undergo a cystoscopy and biopsies or TURBT, this provides histological diagnosis and
information relating to depth of invasion. Locoregional spread is best determined using pelvic MRI and
distant disease CT scanning. Nodes of uncertain significance may be investigated using PET CT.

Treatment
Those with superficial lesions may be managed using TURBT in isolation. Those with recurrences or
higher grade/ risk on histology may be offered intravesical chemotherapy. Those with T2 disease are
usually offered either surgery (radical cystectomy and ileal conduit) or radical radiotherapy.

Prognosis
T1 90%
T2 60%
T3 35%
T4a 10-25%
Any T, N1-N2 30%

heme: Neck lumps

A. Cystic hygroma
B. Bartonella infection
C. Mycobacterium tuberculosis infection
D. Branchial cyst
E. Thyroglossal cyst
F. Pharyngeal pouch
G. Follicular thyroid cyst
H. Parathyroid adenoma
I. None of the above

Please select the most likely underlying disease process for the scenario given. Each option may be used
once, more than once or not at all.

114. A 25 year old cat lover presents with symptoms of abdominal pain, lethargy and sweats. These
have been present for the past two weeks. On examination she has lymphadenopathy in the
posterior triangle.

Bartonella infection

Bartonella infection may occur following a cat scratch. The organism is intracellular.
Generalised systemic symptoms may occur for a week or so prior to clinical presentation.

115. A 25 year old lady presents with an swelling located at the anterior border of the
sternocleidomastoid muscle. The swelling is intermittent and on examination it is soft and
fluctuant.

Branchial cyst

Branchial cysts are remnants of the branchial cleft. They may become infected.

116. A 38 year old lady presents with a mass in the midline of the neck immediately below the hyoid
bone. It moves upwards on tongue protrusion.

Thyroglossal cyst

Thyroglossal cysts are usually located in the midline and are linked to the foramen caecum and
will thus move upwards on tongue protrusion.

Neck lumps

The table below gives characteristic exam question features for conditions causing neck lumps:

Reactive By far the most common cause of neck swellings. There may be a history of
lymphadenopathy local infection or a generalised viral illness
Lymphoma Rubbery, painless lymphadenopathy
The phenomenon of pain whilst drinking alcohol is very uncommon
There may be associated night sweats and splenomegaly
Thyroid swelling May be hypo-, eu- or hyperthyroid symptomatically
Moves upwards on swallowing
Thyroglossal cyst More common in patients < 20 years old
Usually midline, between the isthmus of the thyroid and the hyoid bone
Moves upwards with protrusion of the tongue
May be painful if infected
Pharyngeal pouch More common in older men
Represents a posteromedial herniation between thyropharyngeus and
cricopharyngeus muscles
Usually not seen, but if large then a midline lump in the neck that gurgles on
palpation
Typical symptoms are dysphagia, regurgitation, aspiration and chronic cough
Cystic hygroma A congenital lymphatic lesion (lymphangioma) typically found in the neck,
classically on the left side
Most are evident at birth, around 90% present before 2 years of age
Branchial cyst An oval, mobile cystic mass that develops between the sternocleidomastoid
muscle and the pharynx
Develop due to failure of obliteration of the second branchial cleft in
embryonic development
Usually present in early adulthood
Cervical rib More common in adult females
Around 10% develop thoracic outlet syndrome
Carotid aneurysm Pulsatile lateral neck mass which doesn't move on swallowing

A 22 year old man presents with a discharging area on his lower back. On examination there is an
epithelial defect located 6cm proximal to the tip of his coccyx and located in the midline. There are two
further defects located about 2cm superiorly in the same position. He is extremely hirsute. What is the
most likely diagnosis?

A. Pre sacral tumour

B. Sacrococcygeal teratoma

C. Pilonidal sinus

D. Fistula in ano

E. Occult spina bifida


Pilonidal sinuses are extremely common in hirsute individuals and typically present as midline sinuses
in the natal cleft.

Pilonidal sinus

• Occur as a result of hair debris creating sinuses in the skin (Bascom theory).
• Usually in the natal cleft of male patients after puberty.
• It is more common in Caucasians related to their hair type and growth patterns.
• The opening of the sinus is lined by squamous epithelium, but most of its wall consists of
granulation tissue. Up to 50 cases of squamous cell carcinoma have been described in patients
with chronic pilonidal sinus disease.
• Hairs become trapped within the sinus.
• Clinically the sinus presents when acute inflammation occurs, leading to an abscess. Patients
may describe cycles of being asymptomatic and periods of pain and discharge from the sinus.
• Treatment is difficult and opinions differ. Definitive treatment should never be undertaken when
acute infection or abscess is present as this will result in failure.
• Definitive treatments include the Bascom procedure with excision of the pits and obliteration of
the underlying cavity. The Karydakis procedure involves wide excision of the natal cleft such
that the surface is recontoured once the wound is closed. This avoids the shearing forces that
break off the hairs and has reasonable results.

Pilonidal sinuses are most commonly located in the midline of the natal cleft, as illustrated below
A 43 year old man from Greece presents with colicky right upper quadrant pain, jaundice and a pruritic,
erythematous rash over his body. He is initially treated with ciprofloxacin, but does not improve. What
is the most likely diagnosis?

A. Infection with Wucheria bancrofti

B. Infection with Echinococcus granulosus

C. Type III hypersensitivity reaction

D. Allergy to ciprofloxacin

E. Common bile duct stones

Infection with Echinococcus granulosus will typically produce a type I hypersensitivity reaction which
is characterised by a urticarial rash. With biliary rupture a classical triad of biliary colic, jaundice and
urticaria occurs. Whilst jaundice and biliary colic may be a feature of CBD stones they do not produce
an urticarial rash. Antibiotic sensitivity with ciprofloxacin may produce jaundice and a rash, however it
was not present at the outset and does not cause biliary colic.

Hydatid cysts
Hydatid cysts are endemic in Mediterranean and Middle Eastern countries. They are caused by the
tapeworm parasite Echinococcus granulosus. An outer fibrous capsule is formed containing multiple
small daughter cysts. These cysts are allergens which precipitate a type 1 hypersensitivity reaction.

Clinical features are as follows:

• Up to 90% cysts occur in the liver and lungs


• Can be asymtomatic, or symptomatic if cysts > 5cm in diameter
• Morbidity caused by cyst bursting, infection and organ dysfunction (biliary, bronchial, renal and
cerebrospinal fluid outflow obstruction)
• In biliary ruputure there may be the classical triad of; biliary colic, jaundice, and urticaria

CT is the best investigation to differentiate hydatid cysts from amoebic and pyogenic cysts.
Surgery is the mainstay of treatment (the cyst walls must not be ruptured during removal and the
contents sterilised first).

A 22 year old lady presents with an episode of renal colic and following investigation is suspected of
suffering from MEN IIa. Which of the following abnormalities of the parathyroid glands are most often
found in this condition?

A. Hypertrophy

B. Hyperplasia

C. Adenoma

D. Carcinoma

E. Metaplasia
MEN IIa

• Medullary thyroid cancer


• Hyperparathyroidism (usually
hyperplasia)
• Phaeochromocytoma

In MEN IIa the commonest lesion is medullary thyroid cancer, with regards to the parathyroid glands
the most common lesion is hyperplasia. In MEN I a parathyroid adenoma is the most common lesion.

Multiple Endocrine Neoplasia


Multiple endocrine neoplasia (MEN) is inherited as an autosomal dominant disorder.

The table below summarises the three main types of MEN:

MEN type I MEN type IIa MEN type IIb


Mnemonic 'three P's': Phaeochromocytoma Same as MEN IIa with
Medullary thyroid cancer addition of:
Parathyroid (95%): Parathyroid adenoma (70%) Marfanoid body habitus
Pituitary (70%): Prolactinoma/ACTH/Growth Hyperparathyroidism Mucosal neuromas
Hormone secreting adenoma (60%)
Pancreas (50%): Islet cell tumours/Zollinger
Ellison syndrome

also: Adrenal (adenoma) and thyroid (adenoma)


MENIN gene (chromosome 11) RET oncogene RET oncogene
(chromosome 10) (chromosome 10)
Most common presentation = hypercalcaemia

A male infant is born prematurely at 34 weeks gestation by emergency cesarean section. He initially
appears to be stable. However, over the ensuing 24 hours he develops worsening neurological function.
Which of the following processes is most likely to have occurred?

A. Extra dural haemorrhage

B. Sub dural haemorrhage

C. Sub arachnoid haemorrhage

D. Intraventricular haemorrhage

E. Arteriovenous malformation

Theme from April 2012 Exam

Intraventricular haemorrhage

Intraventricular haemorrhage is a haemorrhage that occurs into the ventricular system of the brain. It is
relatively rare in adult surgical practice and when it does occur, it is typically associated with severe
head injuries. In premature neonates it may occur spontaneously. The blood may clot and occlude CSF
flow, hydrocephalus may result.
In neonatal practice the vast majority of IVH occur in the first 72 hours after birth, the aetiology is not
well understood and it is suggested to occur as a result of birth trauma combined with cellular hypoxia,
together the with the delicate neonatal CNS.
Treatment
Is largely supportive, therapies such as intraventricular thrombolysis and prophylactic CSF drainage
have been trialled and not demonstrated to show benefit. Hydrocephalus and rising ICP is an indication
for shunting.

A 22 year old man is admitted to hospital with a lower respiratory chest infection. He had a splenectomy
after being involved in a car accident. What is the most likely infective organism?

A. Haemophilus influenzae

B. Staphylococcus aureus

C. Rhinovirus

D. Mycobacterium tuberculosis

E. Moraxella catarrhalis
Organisms causing post
splenectomy sepsis:
Streptococcus pneumoniae
Haemophilus influenzae
Meningococci

Encapsulated organisms carry the greatest pathogenic risk following splenectomy. The effects of sepsis
following splenectomy are variable. This may be the result of small isolated fragments of splenic tissue
that retain some function following splenectomy. These may implant spontaneously following splenic
rupture (in trauma) or be surgically implanted at the time of splenectomy.

Post splenectomy sepsis

The loss of splenic function renders individuals at increased risk of fulminant sepsis. Young children are
at the highest risk, especially in the first 2 years following surgery. Surgery for trauma is associated with
a lower risk than when splenectomy is performed as a treatment for haematological disorders.
Infection with encapsulated organisms poses the greatest risk, these organisms may be opsonised, but
this then goes undetected at an immunological level due to loss of the spleen.
Prophylactic vaccinations are usually administered to reduce the risk of pneumococcal septicaemia.
Since the vaccine only covers up to 80% of pneumococcal infections, patients will usually recieve long
term, low dose penicillin prophylaxis in addition to vaccination.

A 24 year old man presents with symptoms of malaise, weight loss and lymphadenopathy. A lymph
node biopsy is performed and the subsequent histology report states that there is evidence of granuloma
formation and central necrosis. What is the most likely underlying cause?
A. Non Hodgkins lymphoma

B. Churg Strauss syndrome

C. Epstein Barr Virus infection

D. Rheumatoid nodule

E. Infection with Mycobacterium tuberculosis

These histological features are typically seen in TB. Necrosis occurring in granulomas is usually
indicative of an underlying infective cause. Churg Strauss syndrome is a form of vasculitis, which is the
usual histological finding. Granulomas are reported in the condition, but it is rare for them to
demonstrate necrosis.

Tuberculosis pathology

• Is a form of primary chronic inflammation, caused by the inability of macrophages to kill the
Mycobacterium tuberculosis.
• The macrophages often migrate to regional lymph nodes, the lung lesion plus affected lymph
nodes is referred to as a Ghon complex.
• This leads to the formation of a granuloma which is a collection of epithelioid histiocytes.
• There is the presence of caseous necrosis in the centre.
• The inflammatory response is mediated by a type 4 hypersensitivity reaction.
• In healthy individuals the disease may be contained, in the immunocompromised disseminated
(miliary TB) may occur.

Diagnosis

• Waxy membrane of mycobacteria prevents binding with normal stains. Ziehl - Neelsen staining
is typically used.
• Culture based methods take far longer.

A 20 year old man develops acute appendicitis, his appendix is removed and he makes a full recovery.
Which of the following pathological processes is least likely to be present in the acutely inflamed
tissues?

A. Altered Starlings forces.

B. Seqestration of neurophils

C. Formation of fluid exudate


D. Formation of granulomas

E. None of the above


Neutrophil polymorphs=Acute
inflammation.
Granuloma = Chronic inflammation.

Acute inflammation:

• 3 phases:

1. Changes in blood vessel and flow: flush, flare, wheal


2. Fluid exudates (rich in protein i.e. Ig, coagulation factors) produced via increased vascular
permeability
3. Cellular exudates mainly containing neutrophil polymorphs pass into extravascular space.

Neutrophils are then transported to tissues via:

a. Margination of neutrophils to the peripheral plasmatic of the vessel rather than the central axial
stream
b. Pavementing: Adhesion of neutrophils to endothelial cells in venules at site of acute inflammation
c. Emigration: neutrophils pass between endothelial cells into the tissue

Acute inflammation

Inflammation is the reaction of the tissue elements to injury. Vascular changes occur, resulting in the
generation of a protein rich exudate. So long as the injury does not totally destroy the existing tissue
architecture, the episode may resolve with restoration of original tissue architecture.

Vascular changes

• Vasodilation occurs and persists throughout the inflammatory phase.


• Inflammatory cells exit the circulation at the site of injury.
• The equilibrium that balances Starlings forces within capillary beds is disrupted and a protein
rich exudate will form as the vessel walls also become more permeable to proteins.
• The high fibrinogen content of the fluid may form a fibrin clot. This has several important
immunomodulatory functions.

Sequelae
Resolution • Typically occurs with minimal initial injury
• Stimulus removed and normal tissue architecture results

Organisation • Delayed removed of exudate


• Tissues undergo organisation and usually fibrosis

Suppuration • Typically formation of an abscess or an empyema


• Sequestration of large quantities of dead neutrophils

Progression to chronic • Coupled inflammatory and reparative activities


inflammation • Usually occurs when initial infection or suppuration has been
inadequately managed

Causes

• Microbacterial infections e.g. Viruses, exotoxins or endotoxins released by bacteria


• Chemical agents
• Physical agents e.g. Trauma
• Hypersensitivity reactions
• Tissue necrosis

Theme: Liver lesions

A. Haemangioma
B. Hepatocellular carcinoma
C. Hepatic metastasis
D. Polycystic liver disease
E. Simple liver cyst
F. Hyatid cyst
G. Amoebic abscess
H. Mesenchymal hamartoma

Please select the most likely liver lesion for the scenario given. Each option may be used once, more
than once or not at all.

124. A 42 year old lady has suffered from hepatitis C for many years and has also developed
cirrhosis. On routine follow up, an ultrasound has demonstrated a 2.5cm lesion in the right lobe
of the liver.

Hepatocellular carcinoma

In patients with cirrhosis the presence of a lesion >2cm is highly suggestive of malignancy. The
diagnosis is virtually confirmed if the AFP is >400ng/mL.
125. A 25 year old man from the far east presents with a fever and right upper quadrant pain. As part
of his investigations a CT scan shows an ill defined lesion in the right lobe of the liver.

Amoebic abscess

Amoebic abscesses will tend to present in a similar fashion to other pyogenic liver abscesses.
They should be considered in any individual presenting from a region where Entamoeba
histiolytica is endemic. Treatment with metronidazole usually produces a marked clinical
response.

126. A 42 year old lady presents with right upper quadrant pain and a sensation of abdominal
fullness. An ultrasound scan demonstrates a 6.5 cm hyperechoic lesion in the right lobe of the
liver. Serum AFP is normal.

Haemangioma

A large hyperechoic lesion in the presence of normal AFP is likely to be a haemangioma. An


HCC of equivalent size will almost always result in rise in AFP.

Benign liver lesions

Benign liver lesions


Haemangioma • Most common benign tumours of mesenchymal origin
• Incidence in autopsy series is 8%
• Cavernous haemangiomas may be enormous
• Clinically they are reddish purple hypervascular lesions
• Lesions are normally separated from normal liver by ring of fibrous tissue
• On ultrasound they are typically hyperechoic

Liver cell • 90% develop in women in their third to fifth decade


adenoma • Linked to use of oral contraceptive pill
• Lesions are usually solitary
• They are usually sharply demarcated from normal liver although they usually
lack a fibrous capsule
• On ultrasound the appearances are of mixed echoity and heterogeneous
texture. On CT most lesions are hypodense when imaged prior to
administration of IV contrast agents
• In patients with haemorrhage or symptoms removal of the adenoma may be
required

Mesenchymal Congential and benign, usually present in infants. May compress normal liver
hamartomas
Liver abscess • Biliary sepsis is a major predisposing factor
• Structures drained by the portal venous system form the second largest
source
• Common symptoms include fever, right upper quadrant pain. Jaundice may
be seen in 50%
• Ultrasound will usually show a fluid filled cavity, hyperechoic walls may be
seen in chronic abscesses

Amoebic abscess • Liver abscess is the most common extra intestinal manifestation of
amoebiasis
• Between 75 and 90% lesions occur in the right lobe
• Presenting complaints typically include fever and right upper quadrant pain
• Ultrasonography will usually show a fluid filled structure with poorly defined
boundaries
• Aspiration yield sterile odourless fluid which has an anchovy paste
consistency
• Treatment is with metronidazole

Hyatid cysts • Seen in cases of Echinococcus infection


• Typically an intense fibrotic reaction occurs around sites of infection
• The cyst has no epithelial lining
• Cysts are commonly unilocular and may grow to 20cm in size. The cyst wall
is thick and has an external laminated hilar membrane and an internal
enucleated germinal layer
• Typically presents with malaise and right upper quadrant pain. Secondary
bacterial infection occurs in 10%.
• Liver function tests are usually abnormal and eosinophilia is present in 33%
cases
• Ultrasound may show septa and hyatid sand or daughter cysts.
• Percutaneous aspiration is contra indicated
• Treatment is by sterilisation of the cyst with mebendazole and may be
followed by surgical resection. Hypertonic swabs are packed around the cysts
during surgery

Polycystic liver • Usually occurs in association with polycystic kidney disease


disease • Autosomal dominant disorder
• Symptoms may occur as a result of capsular stretch

Cystadenoma • Rare lesions with malignant potential


• Usually solitary multiloculated lesions
• Liver function tests usually normal
• Ultrasonography typically shows a large anechoic, fluid filled area with
irregular margins. Internal echos may result from septa
• Surgical resection is indicated in all cases

Which of the following disorders is associated with massive splenomegaly?

A. Acute lymphoblastic leukaemia

B. Acute myeloblastic leukaemia

C. Acute myelomonocytic leukaemia

D. Acute monoblastic leukaemia

E. Chronic granulocytic leukaemia

Chronic leukaemia is more likely to be associated with splenomegaly than acute leukaemia.

Spleen

• Embryology: derived from mesenchymal tissue


• Shape: orange segment
• Position: below 9th-12th ribs
• Weight: 75-150g

Relations

• Superiorly- diaphragm
• Anteriorly- gastric impression
• Posteriorly- kidney
• Inferiorly- colon
• Hilum: tail of pancreas and splenic vessels (splenic artery divides here, branches pass to the
white pulp transporting plasma)
• Forms apex of lesser sac (containing short gastric vessels)

Contents
- White pulp: immune function. Contains central trabecular artery. The germinal centres are supplied by
arterioles called penicilliary radicles.
- Red pulp: filters abnormal red blood cells

Function

• Filtration of abnormal blood cells and foreign bodies such as bacteria.


• Immunity: IgM. Production of properdin, and tuftsin which help target fungi and bacteria for
phagocytosis.
• Haematopoiesis: up to 5th month gestation or in haematological disorders.
• Pooling: storage of 40% platelets.
• Iron reutilisation
• Storage red blood cells-animals, not humans.
• Storage monocytes

Disorders of the spleen


Massive splenomegaly

• Myelofibrosis
• Chronic myeloid leukaemia
• Visceral leishmaniasis (kala-azar)
• Malaria
• Gaucher's syndrome

Other causes (as above plus)

• Portal hypertension e.g. secondary to cirrhosis


• Lymphoproliferative disease e.g. CLL, Hodgkin's
• Haemolytic anaemia
• Infection: hepatitis, glandular fever
• Infective endocarditis
• Sickle-cell*, thalassaemia
• Rheumatoid arthritis (Felty's syndrome)

*the majority of adults patients with sickle-cell will have an atrophied spleen due to repeated infarction

Causes of primary chronic inflammation do not include which of the following?

A. Sarcoidosis

B. Tuberculosis

C. Ulcerative colitis

D. Prostheses

E. Chronic cholecystitis
Chronic cholecystitis is caused by recurrent episodes of acute inflammation.
Prosthetic implants may be the site of primary chronic inflammation. A common example clinically is
breast implants which may become encapsulated. The subsequent fibrosis then results in distortion and
may be painful.

A 30 year old man is trapped in a house fire and sustains 30% partial and full thickness burns to his
torso and limbs. Three days following admission he has a brisk haematemesis. Which of the following is
the most likely explanation for this event?

A. Dieulafoy lesion

B. Curlings ulcers

C. Mallory Weiss tear

D. Depletion of platelets

E. Depletion of clotting factors

Stress ulcers in burns patients are referred to as Curlings ulcers and may cause haematemesis.

Burns

Types of burn

Type of burn Skin layers affected Skin Blanching Management


appearance
Epidermal/Superficial Epidermis Red, moist Yes
Superficial partial Epidermis and part of Pale, dry Yes Normally heals with no
thickness papillary dermis affected intervention
Deep partial thickness Epidermis, whole Mottled red No Needs surgical
papillary dermis affected colour intervention (depending
on site)
Full thickness Whole skin layer and Dry, leathery No Burns centre
subcutaneous tissue hard wound
affected

Depth of burn assessment

• Bleeding on needle prick


• Sensation
• Appearance
• Blanching to pressure

Percentage burn estimation


Lund Browder chart: most accurate even in children
Wallace rule of nines
Palmar surface: surface area palm = 0.8% burn

>15% body surface area burns in adults needs urgent burn fluid resuscitation

Transfer to burn centre if:

• Need burn shock resuscitation


• Face/hands/genitals affected
• Deep partial thickness or full thickness burns
• Significant electrical/chemical burns

Escharotomies

• Indicated in circumferential full thickness burns to the torso or limbs.


• Careful division of the encasing band of burn tissue will potentially improve ventilation (if the
burn involves the torso), or relieve compartment syndrome and oedema (where a limb is
involved)

Theme: Adrenal gland disorders

A. Nelsons syndrome
B. Conns syndrome
C. Cushings syndrome
D. Benign incidental adenoma
E. Malignant adrenal adenoma
F. Waterhouse- Friderichsen syndrome
G. Metastatic lesion
H. Walker - Warburg syndrome
I. Phaeochromocytoma

Please select the most appropriate adrenal disorder for the scenario given. Each disorder may be selected
once, more than once or not at all.

130. A 19 year old lady is admitted to ITU with severe meningococcal sepsis. She is on maximal
inotropic support and a CT scan of her chest and abdomen is performed. The adrenal glands
show evidence of diffuse haemorrhage.
Waterhouse- Friderichsen syndrome

This is often a pre-terminal event and is associated with profound sepsis and coagulopathy.

131. A 34 year old lady is admitted with recurrent episodes of non-specific abdominal pain. On each
admission all blood investigations are normal, as are her observations. On this admission a CT
scan was performed. This demonstrates a 1.5cm nodule in the right adrenal gland. This is
associated with a lipid rich core. Urinary VMA is within normal limits. Other hormonal studies
are normal.

You answered Metastatic lesion

The correct answer is Benign incidental adenoma

This is typical for a benign adenoma.Benign adenomas often have a lipid rich core that is readily
identifiable on CT scanning. In addition the nodules are often well circumscribed.

132. A 38 year old man is noted to have a blood pressure of 175/110 on routine screening. On
examination there are no physical abnormalities of note. CT scanning shows a left sided adrenal
mass. Plasma metanephrines are elevated.

Phaeochromocytoma

Hypertension in a young patient without any obvious cause should be investigated. Urinary
VMA and plasma metanephrines are typically elevated.

Phaeochromocytoma and adrenal lesions

Phaeochromocytoma
Neuroendocrine tumour of the chromaffin cells of the adrenal medulla. Hypertension and
hyperglycaemia are often found.

• 10% of cases are bilateral.


• 10% occur in children.
• 11% are malignant (higher when tumour is located outside the adrenal).
• 10% will not be hypertensive.

Familial cases are usually linked to the Multiple endocrine neoplasia syndromes (considered under its
own heading).
Most tumours are unilateral (often right sided) and smaller than 10cm.

Diagnosis
Urine analysis of vanillymandelic acid (VMA) is often used (false positives may occur e.g. in patients
eating vanilla ice cream!)

Blood testing for plasma metanephrine levels.

CT and MRI scanning are both used to localise the lesion.

Treatment
Patients require medical therapy first. An irreversible alpha adrenoreceptor blocker should be given,
although minority may prefer reversible blockade(1). Labetolol may be co-administered for cardiac
chronotropic control. Isolated beta blockade should not be considered as it will lead to unopposed alpha
activity.

These patients are often volume depleted and will often require moderate volumes of intra venous
normal saline perioperatively.

Once medically optimised the phaeochromocytoma should be removed. Most adrenalectomies can now
be performed using a laparoscopic approach(2). The adrenals are highly vascular structures and removal
can be complicated by catastrophic haemorrhage in the hands of the inexperienced. This is particularly
true of right sided resections where the IVC is perilously close. Should the IVC be damaged a
laparotomy will be necessary and the defect enclosed within a Satinsky style vascular clamp and the
defect closed with prolene sutures. Attempting to interfere with the IVC using any instruments other
than vascular clamps will result in vessel trauma and make a bad situation much worse.

Incidental adrenal lesions


Adrenal lesions may be identified on CT scanning performed for other reasons(3). Factors suggesting
benign disease on CT include(4):

• Size less than 3cm


• Homogeneous texture
• Lipid rich tissue
• Thin wall to lesion

All patients with incidental lesions should be managed jointly with an endocrinologist and full work up
as described above. Patients with functioning lesions or those with adverse radiological features
(Particularly size >3cm) should proceed to surgery.
A 43 year old man presents with dyspepsia and undergoes an upper GI endoscopy. During the procedure
diffuse gastric and duodenal ulcers are identified. A Clo test confirms the presence of Helicobacter
pylori infection. What is the most likely explanation for the ulcers?
A. Decreased gastric motility

B. Increased urease activity

C. Decreased release of mucous and bicarbonate

D. Decreased gastrin levels

E. Increased acid production

Theme from April 2011 Exam


H-Pylori has a number of pathological effects. In this question the main issue is by what mechanism the
organism is able to induce both gastric and duodenal ulceration. Without modestly elevated acid levels,
the duodenum would not undergo gastric metaplasia. H-Pylori cannot colonise duodenal mucosa and
therefore the development of ulcers at this site can only occur in those who have undergone metaplastic
transformation (mediated by increased acidity).

Helicobacter Pylori

Infection with Helicobacter Pylori is implicated in many cases of duodenal ulceration and up to 60% of
patients with gastric ulceration.
It is a gram negative, helical shaped rod with microaerophillic requirements. It has the ability to produce
a urease enzyme that will hydrolyse urea resulting in the production of ammonia. The effect of ammonia
on antral G cells is to cause release of gastrin via a negative feedback loop.
Once infection is established the organism releases enzymes that disrupt the gastric mucous layer.
Certain subtypes release cytotoxins cag A and vac A gene products. The organism incites a classical
chronic inflammatory process of the gastric epithelium. This accounts for the development of gastric
ulcers. The mildly increased acidity may induce a process of duodenal gastric metaplasia. Whilst
duodenal mucosa cannot be colonised by H-Pylori, mucosa that has undergone metaplastic change to the
gastric epithelial type may be colonised by H- Pylori with subsequent inflammation and development of
duodenitis and ulcers.
In patients who are colonised there is a 10-20% risk of peptic ulcer, 1-2% risk gastric cancer and <1%
risk MALT lymphoma.

A 15 year old boy is admitted with colicky abdominal pain of 6 hours duration. On examination he has a
soft abdomen, on systemic examination he has brownish spots around his mouth, feet and hands. His
mother underwent surgery for intussusception, aged 12, and has similar lesions. What is the most likely
underlying diagnosis?

A. Li Fraumeni syndrome

B. Peutz-Jeghers syndrome
C. Addisons disease

D. McCune -Albright syndrome

E. Appendicitis

This is most likely to be Peutz-Jeghers syndrome. Addisons and McCune Albright syndrome may
produce similar skin changes but the intussusception resulting from polyps combined with the autosomal
inheritance pattern makes this the most likely diagnosis.

Peutz-Jeghers syndrome

Peutz-Jeghers syndrome is an autosomal dominant condition characterised by numerous benign


hamartomatous polyps in the gastrointestinal tract. It is also associated with pigmented freckles on the
lips, face, palms and soles. Around 50% of patients will have died from a gastrointestinal tract cancer by
the age of 60 years.

Genetics

• Autosomal dominant
• Responsible gene encodes serine threonine kinase LKB1 or STK11

Features

• Hamartomatous polyps in GI tract (mainly small bowel)


• Pigmented lesions on lips, oral mucosa, face, palms and soles
• Intestinal obstruction e.g. intussusception (which may lead to diagnosis)
• Gastrointestinal bleeding

Management

• Conservative unless complications develop

Which of the following is not included in Multiple Endocrine Neoplasia Type 2b?

A. Phaeochromocytoma

B. Visceral ganglioneuromas

C. Thyroid medullary carcinoma

D. Zollinger Ellison syndrome


E. Marfanoid features

MEN IIB

• Medullary thyroid cancer


• Phaeochromocytoma
• Mucosal neuroma
• Marfanoid appearance

Multiple Endocrine Neoplasia

Multiple endocrine neoplasia (MEN) is inherited as an autosomal dominant disorder.

The table below summarises the three main types of MEN:

MEN type I MEN type IIa MEN type IIb


Mnemonic 'three P's': Phaeochromocytoma Same as MEN IIa with
Medullary thyroid cancer addition of:
Parathyroid (95%): Parathyroid adenoma (70%) Marfanoid body habitus
Pituitary (70%): Prolactinoma/ACTH/Growth Hyperparathyroidism Mucosal neuromas
Hormone secreting adenoma (60%)
Pancreas (50%): Islet cell tumours/Zollinger
Ellison syndrome

also: Adrenal (adenoma) and thyroid (adenoma)


MENIN gene (chromosome 11) RET oncogene RET oncogene
(chromosome 10) (chromosome 10)
Most common presentation = hypercalcaemia

A 25 year old male pedestrian is involved in a road traffic accident. He sustains multiple injuries and is
admitted to the intensive care unit, intubated and ventilated. Over the next week he develops adult
respiratory distress syndrome. What is the main reason for hypoxaemia in this condition?

A. Increased lung compliance

B. Reduced diffusion

C. Reduced surfactant

D. Reduced elastase

E. Left to right shunt


Theme from 2011 Exam
Theme from January 2013 Exam
The diffuse lung injury, which is associated with loss of surfactant and increased elastase release from
neutrophils, results in fluid accumulation. This leads to reduced diffusion, which is the main reason for
hypoxaemia.

Adult respiratory distress syndrome

Defined as an acute condition characterized by bilateral pulmonary infiltrates and severe hypoxemia
(PaO2/FiO2 ratio < 200) in the absence of evidence for cardiogenic pulmonary oedema (clinically or
pulmonary capillary wedge pressure of less than 18 mm Hg).
In is subdivided into two stages. Early stages consist of an exudative phase of injury with associated
oedema. The later stage is one of repair and consists of fibroproliferative changes. Subsequent scarring
may result in poor lung function.

Causes

• Sepsis
• Direct lung injury
• Trauma
• Acute pancreatitis
• Long bone fracture or multiple fractures (through fat embolism)
• Head injury (causes sympathetic nervous stimulation which leads to acute pulmonary
hypertension)

Clinical features

• Acute dyspnoea and hypoxaemia hours/days after event


• Multi organ failure

Management

• Treat the underlying cause


• Antibiotics
• Negative fluid balance i.e. Diuretics
• Mechanical ventilation strategy using low tidal volumes as conventional tidal volumes may
cause lung injury (only treatment found to improve survival rates)

Which virus is associated with Kaposi's sarcoma?

A. Human herpes virus 8


B. Human papillomavirus 16

C. Human T-lymphotropic virus 1

D. Epstein-Barr virus

E. Human papillomavirus 18

Oncoviruses

• Viruses which cause cancer


• These may be detected on blood test and prevented by vaccine

These are the main types of oncoviruses and their diseases:

Oncovirus Cancer
Epstein-Barr virus Burkitt's lymphoma
Hodgkin's lymphoma
Post transfusion lymphoma
Nasopharyngeal carcinoma
Human papillomavirus 16/18 Cervical cancer
Anal cancer
Penile cancer
Vulval cancer
Oropharyneal cancer
Human herpes virus 8 Kaposi's sarcoma
Hepatitis B virus Hepatocellular carcinoma
Hepatitis C virus Hepatocellular carcinoma
Human T-lymphotropic virus 1 Tropical spastic paraparesis
Adult T cell leukaemia
Which of the following is not a feature of Wallerian Degeneration?

A. May result from an axonotmesis

B. May occur in either the central or peripheral nervous systems

C. The axon remains excitable throughout the whole process

D. The distal neuronal stump is affected


E. Is a component of the healing process following neuronal injury

The axon loses its excitability once the process is established.

Wallerian degeneration

- Is the process that occurs when a nerve is cut or crushed.


- It occurs when the part of the axon separated from the neuron's cell nucleus degenerates.
- It usually begins 24 hours following neuronal injury and the distal axon remains excitable up until this
time.
- The degeneration of the axon is following by breakdown of the myelin sheath, a process that occurs by
infiltration of the site with macrophages.
- Eventually regeneration of the nerve may occur although recovery will depend on the extent and
manner of injury

A 45 year old woman complains of painful tingling in her fingers. The pain is relieved by hanging the
arm over the side of the bed. She has a positive Tinel's sign. Which of the following is most likely to
contribute to her diagnosis?

A. Methotrexate use

B. Crohn's disease

C. Hyperthyroidism

D. Tuberculosis

E. Rheumatoid arthritis

This woman has a diagnosis of carpal tunnel syndrome. Rheumatological disorders are a common cause.
Clinical examination should focus on identifying stigmata of rheumatoid arthritis, such as rheumatoid
nodules, vasculitic lesions and metacarpophalangeal joint arthritis.

Carpal tunnel syndome

Carpal tunnel syndrome is caused by compression of median nerve in the carpal tunnel

History

• pain/pins and needles in thumb, index, middle finger e.g. at night


• patient flicks hand to obtain relief
Examination

• weakness of thumb abduction


• wasting of thenar eminence (NOT hypothenar)
• Tinel's sign: tapping causes paraesthesia
• Phalen's sign: flexion of wrist causes symptoms

Causes of carpal tunnel syndrome

MEDIAN TRAP Mnemonic

• Myxoedema
• Edema premenstrually
• Diabetes
• Idiopathic
• Agromegaly
• Neoplasm
• Trauma
• Rheumatoid arthritis
• Amyloidosis
• Pregnancy

Management
Non surgical treatment Surgery
May resolve Complete division of the flexor retinaculum and decompression of the tunnel
spontaneously (successful in approximately 80% of patients)
Avoid precipitants and
reassurance
Night-time splints
Local steroid injections

Which of the following is not an oncogene?

A. ras

B. myc

C. sis

D. Ki 67
E. erb-B

Ki 67 is a nuclear proliferation marker (used in immunohistochemistry). Although, Ki67 positivity is a


marker of malignancy, it is not itself, an oncogene.

Oncogenes

Oncogenes are cancer promoting genes that are derived from normal genes (proto-oncogenes). Proto-
oncogenes play an important physiological role in cellular growth. They are implicated in the
development of up to 20% of human cancers.

Proto-oncogenes may become oncogenes via the following processes:

• Mutation (point mutation)


• Chromosomal translocation
• Increased protein expression

Only one mutated copy of the gene is needed for cancer to occur - a dominant effect

Classification of oncogenes

• Growth factors e.g. Sis


• Transcription factors e.g. Myc
• Receptor tyrosine kinase e.g. RET
• Cytoplasmic tyrosine kinase e.g. Src
• Regulatory GTPases e.g. Ras

Tumour supressor genes


Tumour supressor genes restrict or repress cellular proliferation in normal cells. Their inactivation
through mutation or germ line incorporation is implicated in renal, colonic, breast, bladder and many
other cancers. One of the best known tumour supressor genes is p53. p53 gene offers protection by
causing apoptosis of damaged cells. Other well known genes include BRCA 1 and 2.

A 30 year old male presents with a painless swelling of the testis. Histologically the stroma has
lymphocytic infiltrate. The most likely diagnosis is :

A. Differentiated teratoma

B. Malignant undifferentiated teratoma

C. Classical seminoma
D. Spermatocytic seminoma

E. Anaplastic seminoma

Seminoma is the commonest type of testicular tumour and is more common in males aged between 30-
40 years. Classical seminoma is the commonest subtype and histology shows lymphocytic stromal
infiltrate. Other subtypes include:
1. Spermatocytic: tumour cells resemble spermatocytes. Excellent prognosis.
2. Anaplastic
3. Syncytiotrophoblast giant cells: beta hCG present in cells
A teratoma is common in males aged 20-30 years.

Testicular disorders

Testicular cancer
Testicular cancer is the most common malignancy in men aged 20-30 years. Around 95% of cases of
testicular cancer are germ-cell tumours. Germ cell tumours may essentially be divided into:

Tumour type Key features Tumour markers Pathology


Seminoma • Commonest subtype • AFP usually Sheet like lobular
(50%) normal patterns of cells with
• Average age at • HCG elevated in substantial fibrous
diagnosis = 40 10% seminomas component. Fibrous
• Even advanced • Lactate septa contain
disease associated dehydrogenase; lymphocytic inclusions
with 5 year survival
elevated in 10-20% and granulomas may
of 73% seminomas (but also be seen.
in many other
conditions)
Non seminomatous germ cell • Younger age at • AFP elevated in Heterogenous texture
tumours (42%) presentation =20-30 years up to 70% of cases with occasional
• Advanced disease carries • HCG elevated in ectopic tissue such as
• Teratoma worse prognosis (48% at 5 up to 40% of cases hair
• Yolk sac tumour years) • Other markers
• Choriocarcinoma • Retroperitoneal lymph rarely helpful
• Mixed germ cell node dissection may be
tumours (10%) needed for residual disease
after chemotherapy

Image demonstrating a classical seminoma, these tumours are typically more uniform than teratomas
Image sourced from Wikipedia

Risk factors for testicular cancer

• Cryptorchidism
• Infertility
• Family history
• Klinefelter's syndrome
• Mumps orchitis

Features
• A painless lump is the most common presenting symptom
• Pain may also be present in a minority of men
• Other possible features include hydrocele, gynaecomastia

Diagnosis

• Ultrasound is first-line
• CT scanning of the chest/ abdomen and pelvis is used for staging
• Tumour markers (see above) should be measured

Management

• Orchidectomy (Inguinal approach)


• Chemotherapy and radiotherapy may be given depending on staging
• Abdominal lesions >1cm following chemotherapy may require retroperitoneal lymph node
dissection.

Prognosis is generally excellent

• 5 year survival for seminomas is around 95% if Stage I


• 5 year survival for teratomas is around 85% if Stage I

Benign disease

Epididymo-orchitis
Acute epididymitis is an acute inflammation of the epididymis, often involving the testis and usually
caused by bacterial infection.

• Infection spreads from the urethra or bladder. In men <35 years, gonorrhoea or chlamydia are the
usual infections.
• Amiodarone is a recognised non infective cause of epididymitis, which resolves on stopping the
drug.
• Tenderness is usually confined to the epididymis, which may facilitate differentiating it from
torsion where pain usually affects the entire testis.

Testicular torsion

• Twist of the spermatic cord resulting in testicular ischaemia and necrosis.


• Most common in males aged between 10 and 30 (peak incidence 13-15 years)
• Pain is usually severe and of sudden onset.
• Cremasteric reflex is lost and elevation of the testis does not ease the pain.
• Treatment is with surgical exploration. If a torted testis is identified then both testis should be
fixed as the condition of bell clapper testis is often bilateral.

You might also like